Federal Courts

Ace your homework & exams now with Quizwiz!

United States v. 93.970 Acres of Land

the U.S. Army, having leased an airfield to a private company, wished to use the property for military purposes. The lease was revocable by the government in specified circumstances, but the company disputed the government's right to revoke on the facts presented. In order to obtain immediate use of the land rather than awaiting an adjudication of its right to revoke, the government filed a federal court action to condemn whatever interest the company might have. The lower courts applied the state's election-of-remedies law, which treated the United States, in electing to condemn, as having abandoned its claim of a lawful right to revoke. Thus interpreted, state law required recognition of a remaining property interest in the company, whether or not in the circumstances involved the government in fact had the right to revoke. The Supreme Court unanimously reversed, holding that the government had the right to revoke under the lease and owed no compensation. To follow the state's election-of-remedies law would put the government to the "Hobson's choice" of giving up either its right to immediate. possession under condemnation law or its right to revoke the lease; under governing federal law, no such election was to be imputed

Samuels v. Mackell

the Younger doctrine applies not only to injunctive but also to declaratory relief against a pending state criminal prosecution -> a declaratory judgment would "result in precisely the same interference with" state proceedings as an injunction, especially since a declaratory judgment could be enforced, if necessary, with a subsequent injunction.

Revisions by the Executive Branch

cannot revise when they are acting as a court. In general, a judge is not acting judicially when his decision is reviewable by an executive official. This raises some of the same concerns as advisory opinions.

Marbury Q: Statutory Q

clearly according to the court-> if not here then where? But it does not seem that obvious. Does say that there is appellate jurisdiction

Verizon Maryland Inc. v. Public Service Commission (2002)

confirmed that the core of the Young doctrine is still alive and well, even in cases involving federal statutory rights. Verizon had filed a federal court action against a state agency, its members (in their official capacity), and others, seeking declaratory and injunctive relief on the basis that an agency decision violated both a federal statute and a federal agency ruling. A unanimous Supreme Court, in an opinion by Justice Scalia, held that the Young doctrine allowed Verizon to proceed against the individual commissioners.

Cohens v. Virginia

defendants, convicted in Virginia state court for violating Virginia law by selling District of Columbia lottery tickets, sought review in the Supreme Court, asserting that under the Supremacy Clause they were immune from prosecution because the lottery was authorized by Congress. Although the Court affirmed the convictions on the merits, it rejected the state's contention that it was being sued without its consent. Because the defendants' petition for the writ of error was entirely defensive and sought no affirmative relief, Chief Justice Marshall concluded that it was not a "suit" within the meaning of the Amendment. Alternative reason for why no 11th amendment issue here? They are not citizens of another state but of the same state which is outside of the text of the Amendment. Note that the Hans court would state this is dictum and not really address it

Crosby v. Nat'l Foreign Trade Council

displaced MA law restricting the power of state agencies to purchase goods or services from companies doing business with Myanmar. H: the state statute was preempted by federal statute that imposed similar sanctions on Myanmar; did not rely on federal common law and focus on foreign affairs

Wallis v. Pan American Petroleum Corp. (1966)

distinguished in Boyle. The questions in Wallis were the validity of an oral contract, and the interpretation of a written contract, allegedly assigning a share in an oil and gas lease issued by the United States under the Mineral Leasing Act of 1920. The Court held that these questions were governed by state law—under which, the district court had ruled, the oral contract was not valid and the written contract did not effect an assignment: "In deciding whether rules of federal common law should be fashioned, normally the guiding principle is that a significant conflict between some federal policy or interest and the use of state law must first be specifically shown. We find nothing in the Mineral Leasing Act of 1920 expressing policies inconsistent with state law in the area that concerns us here."

Political Question Doctrine:

ederal courts should not rule on certain allegations of unconstitutional gov. conduct even if other justiciability requirements are met. These questions should be left to the politically accountable branches of gov. Typical political question subject areas [Baker]: foreign relations, questions involving dates of duration of hostilities, formal validity of legislative enactments, status of Native American tribes, and questions about whether a republican form of gov. exists in the states

Federal Common Law Definition:

federal rules of decision whose content cannot be traced directly by traditional methods of interpretation to federal statutory or constitutional commands.

Chafin v. Chafin

found a respondent's argument in a child custody battle confused merits and mootness; rejected respondent's argument that improbability of foreign court respecting the District Court's order gave rise to mootness

Howell v. Mississippi

the petitioner argued that he had presented his federal claim in state court by implication, because the state-law rule on which he had relied was identical to the federal rule. The Court assumed without deciding that identical standards might overcome a failure to have identified as federal a claim pressed in state court, but ruled that state and federal law in fact differed and hence dismissed the petition.

Alden Distinguishing precedent:

ilton v. S. Carolina Public Ry. Comm'n (1991): Court held that injured employee of a state-owned RR could sue his employer, an arm of the state, under FELA. This decision was made under Union Gas, which is no longer good law, so this case is irrelevant. Nevada v. Hall (1979): California could subject Nevada to suit in California state court. Constitution did not reflect an agreement between the states to respect sovereign immunity of one another Reich v. Collins (1994): Despite its immunity from suit in federal court, a state that holds out what appears to be a clear and certain post-deprivation remedy for taxes collected in violation of federal law may not later declare that the remedy does not exist after the fact.

Extension of Syke

over time, the approach in Sykes has been extended, through a series of smaller brush strokes, to cover virtually all state court procedural defaults. Defaults at trial Failure to include a claim among challenges on appeal-Including failure to include claims in petition seeking discretionary state high court appellate review Failure to perfect a timely appeal

Younger and the remed

- the core is the limits on equity and the role of equitable relief- how does that apply with respect to things with other than injunctions. SCOTUS has told us that it raises the same concerns with something like a declaratory judgment What about damages? Different resonance - would probably stay it

Prudential requirements:

1) No raising another person's (third party's) legal rights. 2) No adjudicating generalized grievances more appropriately presented to the other branches. 3) P's claim must fall w/in the zone of interests protected by the law invoked [But see Lexmark 4) Desire not to interfere with family relations structured by state law

Two exceptions to mootness

1) Voluntary Cessation and 2) Capable of repetition yet evading review

Potential benefits of advisory opinion

1. Help to frame issues from the outset. 2. Avoid costly litigation by deciding issues in advance. 3. Allow the branches to engage in a "constitutional dialogue."

What about Declaratory Judgments

28 U.S.C. § 2201(a): In a case of actual controversy within its jurisdiction, . . . any court of the United States, upon the filing of an appropriate pleading, may declare the rights and other legal relations of any interested party seeking such declaration, whether or not further relief is or could be sought. Any such declaration shall have the force and effect of a final judgment or decree and shall be reviewable. Declaratory Judgments are not advisory, but they are subject to justiciability requirements. 1) dispute must be "definite and concrete," not "hypothetical" or "abstract;"; and 2) party seeking DJ must have a concrete interest in the outcome of the DJ.

Factors of stern

: 1) parties- is it the gov 2) source of the claim 3) tied in with a federal regulatory scheme?

Customary International Law and Federal Common Law

A distinct question concerns the status in the American legal system of customary international law (CIL)—which is found not in treaties but rather in the consistent practice of nations, followed from a sense of legal obligation. ' Modern Position: Many international law scholars argue that CIL is presumptively incorporated into the American legal system and given effect as federal law. Revisionist: disagree with treating CIL as federal common law. Prior to Erie the Court had never suggested that CIL was federal common law, and that Sabbatino did not alter that result. Furthermore treating CIL as federal common law is inconsistent with Erie, noting Erie's positivist insistence that law be associated with a particular sovereign and its realist recognition that judicial decision making is a form of lawmaking. Intermediate Approach: CIL should be viewed as neither state nor federal law

Reach of Skelly

A narrow reading of Skelly Oil would permit the exercise of jurisdiction over a declaratory action only if jurisdiction would also exist in a hypothetical nondeclaratory action brought by the declaratory judgment plaintiff. ****A broader reading of Skelly Oil would uphold jurisdiction over a declaratory action if jurisdiction would exist in a hypothetical nondeclaratory (coercive) action brought by either party against the other. The broader interpretation is in some tension with a premise of Skelly Oil, because it would permit defendants to sue in federal court when they could not have done so before enactment of the Declaratory Judgment Act. The Supreme Court has nonetheless endorsed the broader view. In Franchise Tax Board v. Construction Laborers Vacation Trust:

Just Compensation Claims.

A similar equivocation about whether the Constitution mandates judicial remedies has occurred in Takings Clause cases: First English Evangelical Lutheran Church of Glendale v. Los Angeles County: the Court stated that "in the event of a taking, the compensation remedy is required by the Constitution". City of Monterey v. Del Monte Dunes at Monterey, Ltd.: Justice Kennedy's plurality opinion treated it as an open question whether "the sovereign immunity rationale retains its vitality" with respect to just compensation claims and offered a "cf." citation to First English

Wainwright v. Sykes

A state statute required that a defendant make an objection at the time a piece of evidence was offered for submission during trial. Sykes (defendant) failed to comply with this state contemporaneous-objection rule. He claimed that testimony was admitted at trial in violation of rights under Miranda. Sykes was convicted at trial. At his appeal, Sykes failed to raise the issue of the admissibility of the statements taken in violation of Miranda. Only after the appeal did he file a motion in the trial court to vacate the conviction because of the inculpatory statements. "Cause and prejudice" standard: If there is procedural default in the state court, then federal habeas review is barred absent a showing of cause and prejudice attendant to a state procedural waiver.

Protective Jurisdiction.

A third theory rests on the concept of protective jurisdiction. In its broadest form, the theory of protective jurisdiction suggests that Article III permits Congress to vest jurisdiction in the federal courts to prevent discrimination against or hostility toward federal instrumentalities or interests—even when a particular case involves only state substantive law. Congress and courts don't trust state courts to deal with bankruptcy. Very strong federal interest even if the government has not stepped on it. Wechsler: contended whenever Congress possessed authority to create federal rules of decision over a subject matter—which in turn would permit it to confer federal question jurisdiction based on those rules of decision—Congress also possessed the lesser power simply to confer federal jurisdiction over the subject matter, without enacting any federal rules of decision, permitting the federal courts to adjudicate a matter that might be governed entirely by state law. Professor Mishkin: Mishkin contended that "where there is an articulated and active federal policy regulating a field, the 'arising under' clause of Article III apparently permits the conferring of jurisdiction on the national courts of all cases in the area—including those substantively governed by state law." In such circumstances, the protection offered was not to a particular party (as in diversity cases) but rather to the legislative program, and the case can be said to arise under the law establishing that program. In one respect Mishkin's theory is broader than Wechsler's: Mishkin would not require that Congress possess legislative authority to regulate the subject matter presented in a particular case. In another respect, however, Mishkin's theory is narrower than Wechsler's: Mishkin would require that Congress have already enacted legislation that establishes an "articulated and active federal policy" in the field

What about the presumption against pre-emption?

Altria Group, Inc. v. Good (2008): "When the text of a pre-emption clause is susceptible of more than one plausible reading, courts ordinarily 'accept the reading that disfavors preemption.'" Puerto Rico v. Franklin California Tax-Free Trust (2016): when a statute has an express preemption clause, we don't invoke any presumption against preemption but focus on the plain wording of the clause, which necessarily has the best evidence of Congress's preemptive intent.

Lincoln Continued

Analyzing the legislative history, he reasoned that under the Act, the enforceability of the agreement to arbitrate grievances was the quid pro quo for an agreement not to strike and that the purpose of § 301 was to promote labor peace. Thus, in suits under § 301(a), federal courts should apply federal law, fashioned from national labor policy What was Congress trying to do? And were the interpretative approaches the court used?-> Very open to purposive approach Apart from jurisdiction there was a strong argument that federal law, which fosters the negotiation of collective bargaining agreements and the use of arbitration to forestall industrial strife, would be undermined if, as was true under the law of some states, agreements to arbitrate could not be enforced. Applies in state court too: Local 174 v. Lucas Flour Co., 369 U.S. 95 (1962), the Court held that federal common law governs in suits within the scope of § 301 that are brought in state court. This provides more evidence that this was not enough

SCOTUS entering judgment

Another means by which the Supreme Court can deal with state recalcitrance is to enter judgment, as in Martin v. Hunter's Lessee. However, this is extremely rare and the court is usually reluctant to do so: NAACP v. Alabama ex rel. Flowers (1964): after eight years of litigation (including four considerations by the Supreme Court) and obvious state court recalcitrance, the Court still refused the request to formulate its own decree for entry in the state courts. Accepting that it "undoubtedly" has the power to enter judgment, the Court "prefer[red] to follow our usual practice and remand the case to the Supreme Court of Alabama for further proceedings not inconsistent with this opinion."

Claims Against the Government

Any payment for claims against the US requires a specific or general appropriation from Congress, which raises issues of legislative revision. As a constitutional matter, no money out of treasury except under appropriations. AI, §9, cl. 7: "[n]o Money shall be drawn from the Treasury, but in Consequence of Appropriations made by Law" -> Payment of any judgment against U.S. requires a specific or general appropriation by Congress.

Successive petitions

As amended in 1996, § 2244(b)(1) requires dismissal (without exception) of a claim that was presented in a prior petition. Section 2244(b)(2) requires dismissal of a claim not previously presented unless: "(A) The claim relies on a new rule of constitutional law, made retroactive to cases on collateral review by the Supreme Court, that was previously unavailable; or (note: Tyler v. Cain (2001): "made" means "held") "(B)(i) the factual predicate for the claim could not have been discovered previously through the exercise of due diligence; and (ii) the facts underlying the claim, if proven and viewed in light of the evidence as a whole, would be sufficient to establish by clear and convincing evidence that, but for constitutional error, no reasonable factfinder would have found the applicant guilty of the underlying offense."

Rule 59(e) motion to alter or amend a habeas judgment a successive petition.'

Banister v. Davis (2020): Gregory Dean Banister was convicted by a jury of aggravated assault with a deadly weapon and sentenced to thirty years' imprisonment. He filed a habeas petition asserting numerous constitutional violations, which the district court denied on the merits on May 15, 2017. He also requested a certificate of appealability (COA), which the district court also denied in the same order. On June 12, 2017, Banister filed a motion to "amend or alter" the judgment of the district court pursuant to Rule 59(e) of the Federal Rules of Civil Procedure, which the court denied on the merits on June 20, 2017. A Rule 59(e) motion to alter or amend a habeas court's judgment is not a second or successive habeas petition under 28 U.S.C. § 2244(b), so Banister's appeal was timely.

Curtis v. Loether

But in Curtis, the Court held that Jones & Laughlin did not govern, as the action—one between private parties, under a federal civil rights law—was brought in a regular Article III court.

Butler v. McKellar

Butler's claim was similar to that in Roberson, but Butler's conviction had become final in 1982, before Roberson was decided. The Supreme Court ruled, 5-4, that Butler could not obtain habeas relief because the Roberson decision, on which Butler's claim depended, had established a new rule. Chief Justice Rehnquist wrote that "[t]he 'new rule' principle * * * validates reasonable, good-faith interpretations of existing precedents made by state courts even though they are shown to be contrary to later decisions."

Stern and arguments about efficiency?

Cannot uphold just because it is useful- slowly chip away from article III

11A

Chisholm provoked a strongly adverse reaction that subsequent opinions of the Supreme Court have characterized as "a shock of surprise." In due course, both Houses of Congress proposed, and in 1798 the states ratified, the Eleventh Amendment. It provides: "The Judicial power of the United States shall not be construed to extend to any suit in law or equity, commenced or prosecuted against one of the United States by Citizens of another State, or by Citizens or Subjects of any Foreign State."

The Well-Pleaded Complaint Rule and Cases of Exclusive Federal Jurisdiction.

some more specialized jurisdictional provisions both within and outside of the Judicial Code (Title 28) make federal court jurisdiction exclusive. Typically, these provisions use the same statutory phrase ("arising under") as does § 1331 (a) These grants of exclusive jurisdiction have generally been interpreted as embodying the well-pleaded complaint rule. As a result, when important questions of (for example) federal antitrust, patent, or copyright law arise as defenses to state law claims, the case lies outside of original federal question jurisdiction. (b) In one important area of exclusive jurisdiction, cases arising under federal intellectual property laws, Congress altered the regime to restrict the role of state courts. The Leahy-Smith America Invents Act: 28 U.S.C. § 1338 (a) The district courts shall have original jurisdiction of any civil action arising under any Act of Congress relating to patents, plant variety protection, copyrights and trademarks. No State court shall have jurisdiction over any claim for relief arising under any Act of Congress relating to patents, plant variety protection, or copyrights. What happens if they counterclaim? Then that is a claim for relief and they can remove- so when the plaintiff asserts a patent claim then it goes to federal court. If something else- state court has no jurisdiction but that claim can be removed to a federal court where they will then remand the other parts of the case

Lujan v. Defenders of Wildlife- Kennedy Concurrence

Congress has power to define injuries and articulate chains of causation that will give rise to C/C where none existed. But congress must at least ID the injury it seeks to vindicate and relate the injury to the class of persons entitled to sue + P must show that the actions injure him in a concrete and personal way.

The Concept of a "Legislative Court

Congress has vested adjudicative authority not only in administrative agencies, but also in "legislative courts"—federal tribunals denominated as "courts" but not as Article III courts, and staffed by judges lacking life tenure and salary protection.

Miller v. French

Congress was concerned about how the courts handled cases about the treatment of prisoners under the PLRA—overbroad injunctions. So it implemented a statute that terminated previously granted injunctions unless they were narrowly tailored to remedy constitutional violation. PLRA also established a rule saying that application to terminate injunction would act as an automatic stay. Congress, by statute, ordered suspension of a court injunction. Congress can require federal courts to revise their injunctions to be in compliance with changes in the law Congress can change the law and thereby alter the effect of existing injunctions.

Terry Williams O'Connor

Contrary to: flatly contradicts or reaches different result on materially indistinguishable facts Unreasonable application of: if the state court identifies the correct governing legal rule from this Court's cases but unreasonably applies it to the facts of the particular state prisoner's case. Clearly established federal law as determined by SCOTUS: holdings, not dicta as of date relevant state-court decision

Harlow Qualified immunity

Court agrees however, that they are entitled to qualified immunity and the court modifies the test to remove the subjective component- >Conduct did not violate clearly established statutory or constitutional rights of which a reasonable person would have known This is a Bivens claim- comparing approaches to immunity based on type of claim (sec 1983) - untenable to draw a distinction - needs to work the same way across Bivens and 1983- same with Ashcroft v. Iqbal

Ex Parte Young The court is exercising their Equitable relief powers. That has two complications

Criminal proceedings and timing: Generally a court has no equity jurisdiction to enjoin a criminal proceeding: It is proper to add that the right to enjoin an individual, even though a state official, from commencing suits under circumstances already stated, does not include the power to restrain a court from acting in any case brought before it, either of a civil or criminal nature, nor does it include power to prevent any investigation or action by a grand jury However, because it was already in the federal court then it is fine- The federal court enjoins him from acting before he brings his prosecutions- it was entered before he started which allows it to stand. It would be absolutely different If the state court proceedings were already underway Adequate remedy at law? Court says that the railroad cannot get anyone to do this and the prosecutor might not bring it and it would take forever to get to SCOTUS. This acts as another bar- must show you can avoid the 11A and show you are entitled to an injunction

DeFunis v. Odegaard

DeFunis was denied admission to the University of Washington Law School despite test scores that were higher than some of the minorities admitted. DeFunis then successfully asked a trial court to require the school to admit him. On appeal, the Washington Supreme Court reversed, upholding the school's decision to deny DeFunis admission. The U.S. Supreme Court considered the case as DeFunis was entering his final year of school. Court found this case to be moot because the school said they would let him stay regardless. The starting point for analysis is the familiar proposition that "federal courts are without power to decide questions that cannot affect the rights of litigants in the case before them." -> The inability of the federal judiciary "to review moot cases derives from the requirement of Art. III of the Constitution under which the exercise of judicial power depends upon the existence of a case or controversy."

Federal Election Commission v. Wisconsin Right to Life, Inc.

Despite the passing of the 2004 election, the Court held (in an opinion by Chief Justice Roberts) that the case was not moot because (1) it would have been unreasonable to expect the respondents' challenge to make its way through the courts within a single election cycle and (2) similar issues were likely to arise between the parties in the future.

Wellness Int'l Network

Does letting Bankruptcy court determine Stern claims by consent threaten the institutional integrity of the Judicial Branch Yes- can waive personal right. Do not need to expressly consent- you can consent through your conduct.

42 U.S.C. § 1983:

Every person who, under color of any statute, ordinance, regulation, custom, or usage, of any State or Territory or the District of Columbia, subjects, or causes to be subjected, any citizen of the United States or other person within the jurisdiction thereof to the deprivation of any rights, privileges, or immunities secured by the Constitution and laws, shall be liable to the party injured in an action at law, suit in equity, or other proper proceeding for redress."

Grable Breakdown

Expected court to say no - would flow from Merrell - the federal interest is imbedded but not creating a cause of action + there is no implied right of action under section 6335, but the court does not do that and instead explains/minimizes its previous decisions. - Smith: The Smith statement has been subject to some trimming to fit earlier and later cases recognizing the vitality of the basic doctrine, but shying away from the expansive view that mere need to apply federal law in a state-law -> this Court had confined federal-question jurisdiction over state-law claims to those that "really and substantially involv[e] a dispute or controversy respecting the validity, construction or effect of [federal] law." - Merrell: Because arising-under jurisdiction to hear a state-law claim always raises the possibility of upsetting the state-federal line drawn (or at least assumed) by Congress, the presence of a disputed federal issue and the ostensible importance of a federal forum are never necessarily dispositive; there must always be an assessment of any disruptive portent in exercising federal jurisdiction. Merrell Dow cannot be read whole as overturning decades of precedent, as it would have done by effectively adopting the Holmes dissent in Smith Merrell Dow should be read in its entirety as treating the absence of a federal private right of action as evidence relevant to, but not dispositive of, the "sensitive judgments about congressional intent" that § 1331 requires. Justice Thomas Concurrence: I'd be willing to overturn it all.

Stovall v. Denno

that "[s]ound policies of decision-making, rooted in the command of Article III of the Constitution that we resolve issues solely in cases or controversies, militate against" pure prospectivity.

Civil Rule 60(b) motion can count as a successive petition subject a few exceptions.

Gonzalez v. Crosby: whose federal habeas petition had been dismissed as time barred, filed a new petition (a Rule 60[b] petition) in light of the Artuz ruling. The district court denied Gonzalez's new motion. The 11th Circuit affirmed the denial, holding that Gonzalez's latest motion amounted to a second or successive habeas petition which could not be filed without precertification by the court of appeals. Buck v. Davis (2017)

Heyburn's Case: Plaut v. Spendthrift Farm

Heyburn's Case stands for the principle that Congress cannot vest review of the decisions of Article III courts in officials of the executive branch.

Implied Rights of Action on Behalf of the United States

It has been long settled law that the United States needs no specific statutory authorization to bring common law actions of a kind that private citizens could also bring—for example, for trespass or breach of contract.

Heyburn's Case: North Carolina

Judiciary no doubt owes some degree of deference to the legislature, as it is the one who establishes courts and decides the cases that may be heard. However, "no decision of any court of the United States can, under any circumstances, in our opinion, be liable to a reversion, or even suspension, by the Legislature itself, in whom no judicial power of any kind appears to be vested, but the important one relative to impeachments." Rule: Part of the judicial power is the idea that the decisions of the judiciary are not subject to review and revision by the legislature. Any acts or duties which are subject to revision are outside of that power and the courts lack jurisdiction.

Herb v. Pitcairn:

LA action in an Illinois city court that, under state law, lacked jurisdiction of causes of action, such as this one, arising outside the city. Before action on plaintiff's motion for a change of venue, the FELA's two-year statute of limitations had run. The state court dismissed on the ground that there was no timely filed action to transfer, and the Supreme Court affirmed. Acknowledging the state's authority to allocate jurisdiction among its courts, the Court found no violation of the "qualification that the cause of action must not be discriminated against because it is a federal one".

failed to develop

Michael Williams v. Taylor: Justice Kennedy's opinion for a unanimous Court rejected the state's argument that non-development of the facts in state court was in itself enough to bar a federal evidentiary hearing. Section 2254(e)(2) applies only if the prisoner "failed to develop" the facts, and the Court said that the word "fail" typically "connotes some omission, fault, or negligence on the part of the person who has failed to do something." Thus, "a failure to develop the factual basis of a claim is not established unless there is lack of diligence, or some greater fault, attributable to the prisoner or the prisoner's counsel." If we have an instance where no reasonable lawyer could have known, that could take us out of the realm here. Here is was that the juror didn't reveal some issues. Not a failure to develop.

Net result of Skelly + FTB

Net result: FQ jurisdiction usually exists if one of the parties could bring a coercive (non-DJ) action that falls within FQ, but there's an exception at least in the ERISA context for suit by state regulators for a DJ that state regulations are not preempted by federal law.

Immigration and Naturalization Serv. v. St. Cyr

On March 8, 1996, Enrico St. Cyr, a lawful permanent resident, pled guilty in a Connecticut court to a charge of selling a controlled substance. That conviction made him deportable. Before the effective dates of the Antiterrorism and Effective Death Penalty Act of 1996 (AEDPA) and the Illegal Immigration Reform and Immigrant Responsibility Act of 1996 (IIRIRA) section 212(c) of the Immigration and Nationality Act of 1952 was interpreted to give the Attorney General broad discretion to waive deportation of resident aliens. The AEDPA and IIRIRA restricted the class of aliens depending on section 212(c) for relief. St. Cyr's removal proceedings commenced after AEDPA's and IIRIRA's effective dates. Subsequently, the Attorney General claimed that the AEDPA and IIRIRA withdrew his authority to grant St. Cyr a waiver. The Supreme Court held that the 1996 amendments had not foreclosed jurisdiction under the pre-existing general grant of habeas corpus jurisdiction in 28 U.S.C. § 2241. In reaching that conclusion, the Court relied heavily on a series of interpretive presumption.

Bases for liability for municipals

Rule promulgated adopted by municipality legislative body- yes; clearest example Policy statement or decision that is officially made by a policy making official- yes Pembaur v. City of Cincinnati (1986), the Court ruled that a single decision of a high official like the county prosecutor, who had authority under state law to decide whether police officers should enter a premises and whose decision "may fairly be said to represent official policy", was an adequate basis for imposing governmental liability under § 1983. Custom that is widespread, well-settled practice that constitutes a standard operating procedure of municipality Inadequate training/ supervision screening: Only if municipality is deliberately indifferent to the fact that a violaiton of that federal right is a highly predictable consequences of the inadequate training etc

Competitors' Standing:

Rule was that a proprietor of a business could not object to gov't support of competing activities, because the common law does not recognize an interest in freedom from competition FCC v. Sanders Bros. Radio (1940): FCC awarded broadcast license to competitor, and §402(b) of the Communications Act allowed appeal by any person aggrieved or whose interest were adversely affected by a Commission decision granting or refusing an application

Ripeness:

Seeks to separate out matters that are not ready for review b/c the injury is speculative and may never occur from those causes that are appropriate for federal court action Two considerations: 1) Hardship to the parties of withholding court consideration; and 2) Fitness of the issues for judicial decision

Longstanding examples of legislative courts fall into three main but not necessarily exclusive categories.

Territorial Courts. Military Courts Courts to Adjudicate Public Rights Disputes

Variable Preemption and Federal Common Lawmaking:

The Supreme Court has similarly indicated that preemption analysis operates differently in different areas. Presumption against preemption applies differently in different areas. Strongest where Congress acting in an area traditionally regulated by the states. Weakest where the federal interest is so dominant that the federal system will be assumed to preclude enforcement of state laws on the same subject.

Alden v. Maine

The obligation in Reich arises from the constitution itself; Reich does not speak to the power of Congress to subject States to suits in their own courts.

Burger and Rehnquist courts narrow the scope of federal habeas

There was a lot of back and forth between legislature and courts. - Non-retroactivity - Procedural default- When a claim raised in a federal habeas petition was not presented to the state courts, or was not presented in accordance with state procedural rules (e.g., was not raised on a timely basis), ordinarily the state courts will not have reached the merits of the claim. When that is so, subject to only the narrowest exceptions, the federal habeas court will not consider the defaulted claim. - Restrictions on 4th amendment claims- Stone v. Powell- restricted this

Bell v. Hood

There, the Supreme Court held that a federal district court had jurisdiction under § 1331 over a claim for damages against federal officers for allegedly unconstitutional arrests and searches, but reserved the question whether there was a good claim on the merits under federal (as opposed to state) law. If someone is asserting a Bivens claim they are asserting something within Fed Q jurisdiction. [I]t is established practice for this Court to sustain the jurisdiction of federal courts to issue injunctions to protect rights safeguarded by the Constitution and to restrain individual state officers from doing what the 14th Amendment forbids the state to do. Moreover, where federally protected rights have been invaded, it has been the rule from the beginning that courts will be alert to adjust their remedies so as to grant the necessary relief."

Congressional Apportionment of Jurisdiction Among Federal Courts and Resulting Limitations on the Authority of Enforcement Courts:

Under its broad authority to apportion jurisdiction among federal courts, Congress can create specialized courts and vest them with exclusive jurisdiction over particular categories of cases. But hard issues can arise when Congress gives a federal court jurisdiction over a case while simultaneously providing that one or more issues in that case may be decided only by a different federal court. Question here: the scope of congressional power to confer jurisdiction on a court while limiting its authority to consider particular issues that are relevant to the controversy.

United States v. Ferreira

Under treaty between U.S. and Spain, Congress directed the judge of the territorial court, and later of the district court, in Florida to receive and adjudge claims for losses suffered by certain Spanish citizens due to operations of the American army in Florida. Court reviewed a statute requiring judges to examine claims for losses and report them to the Secretary of the Treasury, who would authorize final payment if satisfied they accorded with a treaty. Court dismissed an appeal by the United States from an award of a lower judge concluding that the judge was not acting judicially as the executive branch could revise his work. Instead, was acting as a commissioner—a non-Article III administrative official charged with adjusting claims against the US.

Monell v. Department of Social Services (1978)

a class of female employees sued municipal agencies for back pay and injunctive relief, challenging defendants' policy of requiring pregnant employees to take unpaid leaves of absence. ***Cities and counties may be sued directly under § 1983 for damages or for declaratory and injunctive relief "where the action that is alleged to be unconstitutional implements or executes a policy statement, ordinance, regulation or decision officially adopted and promulgated by that body's officers. Moreover local governments may be sued for constitutional deprivations visited pursuant to governmental 'custom' even though such a custom has not received formal approval through the body's official decisionmaking channels". Different reading of 1983's claims- clears out the way to approach it differently -> Sherman amendment would have created municipal liability for certain acts by private citizens, so the rejection of the Sherman amendment does not show intent to foreclose municipal liability for acts by government officials. ***But the Court clearly stated, albeit in dictum, that "a municipality cannot be held liable solely because it employs a tortfeasor—or, in other words, a municipality cannot be held liable under § 1983 on a respondeat superior theory".

Pennhurst State School & Hosp. v. Halderman (1984):

a resident of a Pennsylvania state institution for the developmentally disabled (Pennhurst) filed a federal class action seeking injunctive relief against the institution and various state and county officials, alleging that conditions at Pennhurst violated federal statutory and constitutional requirements, as well as state law. On its initial review in 1981, the Supreme Court held that there was no basis for relief under one provision of federal law, but remanded for determination whether plaintiffs might be entitled to relief on other grounds. On remand, the Third Circuit en banc affirmed its prior judgment, ruling that state law required the award of certain injunctive relief ordering the placement of residents in the least restrictive setting. Young does not apply to claims under state rather than federal law- the point of Young is we need it to uphold the supremacy of Federal law, but that rationale is not available here.

Testa v. Katt

att sold Testa an automobile in Rhode Island for $210 more than the vehicle's established ceiling price. Testa sued Katt in a Rhode Island state court pursuant to the Emergency Price Control Act (Act), a federal law. Section 205(e) of the Act authorized suits for up to treble damages against those who sold goods in excess of their prescribed ceiling price. The Supreme Court of Rhode Island reversed on the ground that its state courts could not enforce § 205(e) because it was a "penal" statute that was "foreign" to Rhode Island." hence Rhode Island courts, though their jurisdiction is adequate to enforce similar Rhode Island "penal" statutes, need not enforce § 205(e). Penal, perhaps, said the U.S. Supreme Court, but foreign, no. Federal laws cannot be treated "as though they were laws emanating from a foreign sovereign....[T]he Constitution and the laws passed pursuant to it are the supreme laws of the land ... the policy of the federal Act is the prevailing policy in every state."

Griffith v. Kentucky

the Court wrote: "[F]ailure to apply a newly declared constitutional rule to criminal cases pending on direct review violates basic norms of constitutional adjudication. * * * [A]fter we have decided a new rule in the case selected, the integrity of judicial review requires that we apply that rule to all similar cases pending on direct review."

South Carolina v. Katzenbach

upheld a provision of Voting Rights Act of 1965, requiring that actions to exempt states from the coverage of the act and actions to permit certain "suspended" state voting regulations to go into effect be brought in the D. D.C.

Nevada v. Hall

which held that a California state court need not honor Nevada's claim of sovereign immunity in a suit against it by a private party, on the grounds that it dealt with the immunity of a state in the courts of another state, not in its own courts, and that "an implied constitutional limit on the power of the States cannot be construed * * * to support an analogous reluctance to find implied constitutional limits on the power of the Federal Government". However, this was overturned in Franchise Tax Bd of Cal. V. Hyatt (2019): now states maintain sovereign immunity over cases in sister courts

Congressional Power over the Supreme Court's Appellate Jurisdiction

Article III's provision that Congress may create "exceptions" to the Supreme Court's appellate jurisdiction was added to the Constitutional Convention's working draft by the Committee of Detail. Hart: The measure is simply that the exceptions must not be such as will destroy the essential role of the Supreme Court in the constitutional plan Professor Ratner argues that, to be constitutionally valid, "exceptions" to the Court's appellate jurisdiction must not "negate" the Court's "essential constitutional functions of maintaining the uniformity and supremacy of federal law."

Voluntary Cessation

As noted in DeFunis, a long line of cases holds that an action for an injunction, or other judgment with continuing force, does not become moot merely because the conduct immediately complained of has terminated, if there is a sufficient possibility of a recurrence that would be barred by a proper decree. Q: is the recurrence sufficiently likely

2254 (e)(1)

In a proceeding instituted by an application for a writ of habeas corpus by a person in custody pursuant to the judgment of a State court, a determination of a factual issue made by a State court shall be presumed to be correct. The applicant shall have the burden of rebutting the presumption of correctness by clear and convincing evidence.

Douglas v. City of Jeannette

Ordered dismissal, for want of equity, of a class action by Jehovah's Witnesses (whose religious practice was to distribute books door-to-door) seeking to restrain, as a violation of the First Amendment, their prosecution under a city ordinance forbidding solicitation of orders for merchandise without a license. The Chief Justice noted first that Congress had adopted a deliberate policy of "leaving generally to the state courts the trial of criminal cases arising under state laws, subject to review by this Court of any federal questions involved." He then deployed equitable concepts to implement that policy, reasoning that a criminal prosecution, "even though alleged to be in violation of constitutional guarantees, is not a ground for equity relief since the lawfulness or constitutionality of the statute or ordinance on which the prosecution is based may be determined as readily in the criminal case as in the suit for an injunction"

Seminole Tribe other solutions in the absence of allowing suit here

United States can still sue the states and in fact it often has on behalf of tribes. When that occurs then tribes can litigate as a party of interest as well to some extent. 1983 claims are still available as well. Ex Parte Young remains a possibility: FN 17: this does not mean that Congress cannot authorize federal jdx under Ex Parte Young over a cause of action with a limited remedial scheme. However, Congress did not intend that result in the IGRA.

Cannon Dissent

- Really emphasizes the concept of separation of powers- court is really legislating when they do this and Congress should determine when private parties are to be given causes of action under legislation it adopts. - Powell willing to give more leeway there- there is no legislature there to create a right now or recently. - Powell doesn't like Cort or Borak and is able to distinguish the other three cases in which the court has recently implied an action because there was no alternative enforcement mechanism there. Finally, he notes that this is also expanding federal question jurisdiction.

Why not like it?

- Sometimes its an inanimate object that cannot adjudicate it themselves-> environmental cases - What about the fact that we are a nation of laws and in some instances no one else will be able to litigate the issue. Richardson- P sues as a federal taxpayer- court says generalized grievance, but court acknowledges that it can be argued that if respondent permitted to litigate this issue, no one can do so- but this is committed to the political process.

Enforcing this mandate

Normally the Supreme Court, when reversing a state court judgment, remands the case for proceedings "not inconsistent" with the Court's opinion. The state court is thus free to resolve any undecided questions or even to alter its determination of underlying state law. The reversal may not, therefore, be decisive of the final judgment. If a state court deviates from the Supreme Court's mandate, the proper remedy is to seek a new review of the judgment, as in Martin v. Hunter's Lessee. So long as such review is possible, issuance of a writ of mandamus as a means of obtaining compliance has been considered inappropriate.

Stern and the adjunct model:

Not just limited to fact finding - broad jurisdiction here. Bankruptcy courts under the 1984 Act resolve "[a]ll matters of fact and law in whatever domains of the law to which" the parties' counterclaims might lead. Given that authority, a bankruptcy court can no more be deemed a mere "adjunct" of the district court than a district court can be deemed such an "adjunct" of the court of appeals.

Webb v. Webb

the Court held that a litigant who had complained in a state court custody suit about a failure to give "full faith and credit" to a prior judgment, but who had not mentioned the Full Faith and Credit Clause, had presented only a state law issue under the Uniform Child Custody Jurisdiction Act, and thus could not raise the federal constitutional issue in the Supreme Court.

When is a federal issue raised? The Governing Standard. T

"There are various ways in which the validity of a state statute may be drawn in question on the ground that it is repugnant to the Constitution of the United States. No particular form of words or phrases is essential, but only that the claim of invalidity and the ground therefor be brought to the attention of the state court with fair precision and in due time." New York ex rel. Bryant v. Zimmerman The Court has long required a litigant to show "that some provision of the Federal, as distinguished from the state, Constitution was relied upon," Claims that a state statute violates the Constitution or denies due process, without more, have been treated as referring to state and not federal provisions.

Comparing direct and habeas review for defaults

(a) A state procedural default that would not bar direct review by the Supreme Court (because the procedural ruling is not an "adequate" state ground) also does not bar federal habeas corpus review -> An inadequate procedural ground bars neither SCOTUS review nor federal habeas review These are fairly limited but something like needs to be on recycled paper. (b) Some defaults that would bar Supreme Court review do not bar habeas review—for example, where "cause and prejudice" can be shown. Cause and prejudice test does not apply to direct review. Unlike the direct review, there are ways around procedural default for purposes of obtaining federal habeas relief - cause and prejudice or actual innocence But, the standards applied in habeas for excusing state court defaults are so difficult for petitioners to satisfy that in practice the standards applied on direct and collateral review differ very little (c) If the highest state court to address the issue disregards the default and addresses the merits then the procedural default bars neither direct SCOTUS review nor federal habeas review-> merits decision; no default.

The Various Ways in Which State Law Applies

(a) The federal government (Congress as well as the federal courts) lacks lawmaking authority. (b) Congress has lawmaking authority, but in the absence of legislative action, state law governs. (Wasn't that so on the facts of Erie itself?) (c) Federal legislation calls for the application of state law as part of a federal scheme. (The Federal Tort Claims Act is an example.) (d) Although federal common law governs a given question, state law furnishes an appropriate and convenient measure of the content of this federal law.

Historically, Congress shown a desire to go under the maximum allowed for judicial power:

- Federal Question Jurisdiction: Only in 1875 did Congress provide an enduring grant of general federal question jurisdiction, now found in 28 U.S.C. § 1331. From 1875 to 1980, the general federal question statute included an amount-in-controversy requirement. Even today, § 1331 confers jurisdiction only when the federal question appears on the face of the plaintiff's well-pleaded complaint. - Diversity Jurisdiction: Beginning in 1789, lower federal courts have had jurisdiction to hear many diversity cases. But the general statutory grant has always required "complete diversity" when there are multiple parties on one or more sides of a case, a requirement not imposed by Article III. - Supreme Court Jurisdiction. From 1789-1914, the Supreme Court could review only those state court decisions that denied a claim of federal right; state court decisions upholding federal claims were thus excluded from the Court's appellate jurisdiction.

Edelman v. Jordan (1974)

, Jordan had brought an individual and class action against various state officers, including Edelman, seeking declaratory and injunctive relief on the basis that the defendants were administering the federal-state programs of Aid to the Aged, Blind, or Disabled (AABD) in a manner inconsistent with various federal regulations and with the Fourteenth Amendment". The district court granted a permanent injunction requiring compliance with the federal time limits for processing and paying AABD applicants" and also ordered the defendants to release AABD benefits "wrongfully withheld" from certain AABD applicants during a specified period. While the rationale of Ex parte Young permitted the part of the judgment that constituted a prospective injunction, the provision for the payment of funds "wrongfully withheld" in the past was retrospective relief barred by the Eleventh Amendment. The order that the state officials pay the past benefits that had been wrongfully withheld did not fall within Ex parte young-> when the action is in essence one for the recovery of money from the state, the state is the real, substantial party in interest and is entitled to invoke its sovereign immunity from suit even though individual officials are nominal defendants the state's immunity derived from the fact that the judgment mandated "a form of compensation"—a "retroactive award of monetary relief" that would "to a virtual certainty be paid from state funds, and not from the pockets of the individual state officials who were the defendants in the action

Johnson v. Williams

, the Court extended Harrington. In Williams, the defendant had argued on appeal that the trial court's discharge of a particular juror violated both a California statute and the Sixth Amendment's right to jury trial. The state appellate court's extended discussion of the matter never expressly stated that the court was deciding the Sixth Amendment issue. On habeas review, the Court of Appeals for the Ninth Circuit held that § 2254(d) did not apply, deeming it "obvious" that the state appellate court had "overlooked or disregarded" the Sixth Amendment claim. The Supreme Court reversed. After noting that "Richter itself concerned a state-court order that did not address any of the defendant's claims," the Court said that there was "no reason why the Richter presumption should not also apply when a state-court opinion addresses some but not all of a defendant's claims." Presumption that it's an adjudication on the merits may be rebutted if state standard is less protective than or very different from federal standard By the state to show the claim was not decided on the merits at all and thus might have been forfeited By the petitioner to get de novo federal review

Prominent on the surface of any case held to involve a political question is found:

- A textually demonstrable constitutional commitment of the issue to a coordinate political department; - A lack of judicially discoverable and manageable standards for resolving it; - The impossibility of deciding without an initial policy determination of a kind clearly for nonjudicial discretion; - The impossibility of a court's undertaking independent resolution without expressing lack of the respect due coordinate branches of government; or an unusual need for unquestioning adherence to a political decision already made; - The potentiality of embarrassment from multifarious pronouncements by various departments on one question

Congress's Jurisdiction Options

- Concurrent: Congress can prescribe that federal jurisdiction is concurrent with state jurisdiction, leaving a choice of venue to the parties; or - Federal Exclusivity: make federal jurisdiction exclusive, prohibiting state courts from exercising jurisdiction; or - State Exclusivity: prescribe that a certain type of suit can only be filed in state court (would have to consider how much state courts would be free to refuse to hear such cases); or - AI Court: Channel initial adjudication to a federal tribunal other than AIII court (e.g., an administrative agency) with judicial review in AIII courts - Limit SCOTUS Appellate Jurisdiction: In any of the foregoing cases, Congress might or might not purport to limit the capacity of the Supreme Court ultimately to review the matter.

McCardle Breakdown

- Court looks first to jurisdiction (compare Marbury v. Madison):The first question necessarily is that of jurisdiction; for, if the act of March, 1868, takes away the jurisdiction defined by the act of February, 1867, it is useless, if not improper, to enter into any discussion of other questions. - It is quite true, as was argued by the counsel for the petitioner, that the appellate jurisdiction of this court is not derived from acts of Congress. It is, strictly speaking, conferred by the Constitution. But it is conferred "with such exceptions and under such regulations as Congress shall make." - We are not at liberty to inquire into the motives of the legislature. We can only examine into its power under the Constitution; and the power to make exceptions to the appellate jurisdiction of this court is given by express words.

State court and Mootness

- DeFunis: after concluding that the action was moot, the Supreme Court vacated the judgment and remanded "for such proceedings [in the state supreme court] as may be deemed appropriate." ASARCO Inc. v. Kadish (1989), discussed p. 158, supra, the Court noted its decision in DeFunis to vacate and remand for further proceedings in state court, but said that its more recent practice has been to dismiss cases that become moot on review from the state courts, leaving undisturbed the state court judgment If a state court holds moot a case involving a federal question, is Supreme Court review precluded? Liner v. Jafco, Inc. (1964), a state court enjoined picketing in a labor dispute, despite a contention that its jurisdiction was federally preempted. Pending decision on appeal, construction at the site was completed, and the state appellate court held that the case had become moot (though it also expressed an opinion on the merits). The Supreme Court unanimously held, per Justice Brennan, that "in this case the question of mootness is itself a question of federal law upon which we must pronounce final judgment."

Why require Standing

- Ensuring the stakes and limits of any resulting ruling are well understood; Particularized injury- specific to you- more zealous advocate - Docket control- save it for people with more cases. - Resolve issues on facts. - Focus on people who are more concerned- eliminates nefarious intermeddler- limiting the judicial process to litigants who will be energetic adversaries - Avoid undue court interference with the executive branch

Sheldon Breakdown

- If the Constitution had ordained and established the inferior courts, and distributed to them their respective powers, they could not be restricted or divested by Congress, but as it has made no such distribution, Congress, having the power to establish the courts, must define their respective jurisdiction - AIII simply provides for diversity jurisdiction; it doesn't contain the assignment limitation of § 11. But Congress is free to circumscribe the broad limits of AIII, at least as far as the lower federal courts are concerned.

Problems with Justice Story's Theory

- In tension with Madisonian Compromise and language of AIII, which reflected understanding that decision whether to create lower federal courts should be a matter of Congressional discretion - J. Story's approach is undermined by the Judiciary Act of 1789: It did not provide for general FQJ in the lower federal courts. It did not extend SCOTUS appellate jurisdiction to review of state cases where the federal claim was upheld in state court. It had an amount-in-controversy requirement for FQJ (this remained until 1980). - Pragmatically, makes little sense: Example: diversity jurisdiction decided in state court under state law below the amount in controversy wouldn't get SCOTUS review, and if this position is adopted, would have to vest the full scope of diversity jurisdiction in a federal court à striking expansion of diversity jurisdiction and hard to square with Madisonian Compromise SCOTUS jurisdiction to review state court decisions used to be mandatory, but discretionary since 1914. Is cert. jurisdiction a sufficient "vesting" of federal judicial power in federal judiciary? - Although J. Story's dichotomy between "cases" and "controversies" was not as starkly presented as those put forth by modern academics, some scholars have noted that emphasizing the all to speak to mandatory jurisdiction may be flawed.

reconciling Akins and Lujan

- Injury sufficient for purposes of Art. III may exist simply by virtue of the invasion of a legal right created by Congress, - Congress has power to transform concrete harms that were previously inadequate at law into cognizable injuries - Inability to procure information to which Congress has created a right itself

Lujan v. Defenders of Wildlife - Breakdown

- Injury: 1) Prior visits- don't show future imminence because there was no concrete plans to return-When thinking about building the record you need to have some evidence to back it up. 2) Court is not buying other novel arguments - look to evidence of one part of the world- court in Lujan - Did Congress succeed in conferring standing? Congress can't convert a generalized grievance into an individual right - this creating a claim is illegal. Court believes that in this type of law suit it is policing separation of powers- gives Congress inappropriate role of letting anyone enforce the claim. - Redressability- only a plurality not the focus of the opinion Effectiveness of relief v. SOI - plurality would not be effective Proportion of funding coming from agencies: plurality: don't know that cutting this portion of the funding would stop the projects. - Procedural Injury: This is not a case where respondents are seeking to enforce a -procedural requirement the disregard of which could impair a separate concrete interest. Procedural rights are special. The person who has been accorded a procedural right to protect his concrete interest can assert that right without meeting the normal standing requirements of redressability and immediacy.

When is it justified to use federal common law to displace state law?

- Is US or US officer or agency as a party a necessary condition to the application of the federal common law to displace state law? No (Boyle) - Is US or US officer or agency as a party a sufficient condition to the application of the federal common law to displace state law? Not always-> see Kimbell. But its absence can cut against displacing state law- See Miree, Parnell; Wallis - Matter of strong local/state interest? - Interest in vertical uniformity? Reliance on state-law rule?

Characteristics of modern federal common law

- It is distinctively federal so unlike the general law applied under Swift it presents federal issue for jurisdictional purposes - Its part of federal law for purposes of the Supremacy Clause- applied in states as well - Congress can displace it or override it - It must be supported by a particular justification unlike the scheme of general law.

Stern on the public rights doctrine: Works through the various public rights precedent Murray Lessee; Crowell; Thomas and Schor

- Murray Lessee- challenge to government sale of custom collectors land - Crowell- suits between the government and others - Northern Pipeline- gov only - Thomas: important that federal statute did not depend on or replace a right to compensation under state law. Claim derives from fed regulatory scheme or agency expertise essential to a limited regulatory objective= Integrally related to a particular federal gov action. Here it is not a federal statutory scheme that is as important - Schor: orders required enforcement by district court, parties consented there but not here, and total overlap - Granfinanciera we rejected a bankruptcy trustee's argument that a fraudulent conveyance action filed on behalf of a bankruptcy estate against a noncreditor in a bankruptcy proceeding fell within the "public rights" exception. We explained that, "[i]f a statutory right is not closely intertwined with a federal regulatory program Congress has power to enact, and if that right neither belongs to nor exists against the Federal Government, then it must be adjudicated by an Article III court."

Private rights fact questions

- Ordinary facts: the apparently solid thing about Crowell is the holding that administrative findings of non-constitutional and jurisdictional facts may be made conclusive upon the courts, if not infected with any error of law, as a basis for judicial enforcement of a money liability of one private person to another - - Recurrent themes include: 1) a desire to take advantage of specialized expertise, 2) to adapt law and administration swiftly to changing priorities, 3) to unite in a single body the power to adjudicate and to promulgate regulations, and 4) to avoid having to increase the number of Article III judges, thereby diminishing their prestige. - Jurisdictional facts: Today any requirement of independent judicial judgment on questions of jurisdictional fact is at best infrequently applied, and independent judicial fact-finding—rather than redetermination of facts on the administrative record, ordinarily with considerable deference to the agency's determination—virtually never occurs.

The DTA and the MCA

- Rasul v. Bush (2004) - the Court held, as a matter of statutory interpretation, that the general grant of habeas corpus jurisdiction, embraced petitions filed on behalf of aliens detained by the United States as enemy combatants at Guantánamo Bay, Cuba. Congress reacted by enacting the Detainee Treatment Act of 2005 - Hamdan: DTA jurisdiction stripping provision does not appyl to cases pending on date of enactment. - Congress enacts MCA: express strip of habeas jurisdiction - Boumediene (2008): Petitions for habeas corpus were filed on behalf of detainees at Guantánamo Bay who had been determined by CSRTs to be enemy combatants. The government argued that their petitions were barred by the MCA's jurisdiction-stripping provision; in turn, the detainees contended that the jurisdiction-stripping provision violated the Suspension Clause. MCA is unconstitutional suspension of the writ. The only Supreme Court decision clearly holding that a congressional enactment restricting jurisdiction—in that case, of both federal and state courts—is unconstitutional.

Tyler v. Cain

. After Tyler's federal habeas petition was denied, he filed a second petition, contending that the jury instruction on proof beyond a reasonable doubt was substantively identical to one later invalidated in Cage v. Louisiana, 498 U.S. 39 (1990). Under § 2244(b)(2)(A), in order to raise this claim in a second federal habeas petition, Tyler had to demonstrate that the ruling in Cage had been "made retroactive to cases on collateral review by the Supreme Court." The court held that recognition of first exception is the holding. It was not enough that the Court has established principles of retroactivity (as it did in Teague) and leaves the application of those principles to lower courts. SCOTUS itself has to make retroactive

Robertson v. Seattle Audubon Soc'y,

03 U.S. 429 (1992). Two lawsuits alleged that that the U.S. Forest Service's plan to permit some timber harvesting in areas that were home to the Northern Spotted Owl violated the Endangered Species Act. While those cases were pending, Congress, as part of an appropriations bill, enacted a provision, known as Section 318, that contained a management plan for timber harvesting only in the areas that were home to the Northern Spotted Owl. Section 318, which expired at the end of the fiscal year, declared that management of the forests in accordance with the plan set forth in that section constituted "adequate consideration" for the purposes of the legal requirements governing the two pending lawsuits, which Section 318 identified by docket number. The Supreme Court, however, unanimously disagreed, contending that Section 318 did not instruct a court in how to apply pre-existing legal standards to a pending case, but rather amended the pre-existing standards.

Two Exceptions to the general rule that new constitutional rules do not retroactively apply.

1) A new rule does apply retroactively if it provides that the conduct for which the defendant was prosecuted is constitutionally protected. 2) A habeas court may apply a new rule (a) that implicates the fundamental fairness of the trial and (b) without which the likelihood of an accurate conviction is seriously diminished. [not really applied]

Exceptions to Younger Abstention

1) Bad Faith Prosecution or Harassment. The Supreme Court has never authorized intervention under this exception. If Younger itself is sound, why should there be such an exception? Is a state court unable to determine whether a prosecutor is acting in bad faith? Perhaps we could call Dombrowski this type of exception 2) Patent and Flagrant Unconstitutionality. Younger also suggested that federal courts might be justified in restraining prosecutions under statutes that are "flagrantly and patently violative of express constitutional prohibitions in every clause, sentence, and paragraph, and in whatever manner and against whomever an effort might be made to apply it." -> Whatever the rationale, not much is left of this "exception" Other Extraordinary Circumstances. Only example here was in front of an agency that was clearly biased Gibson v. Berryhill (1973),

Justice Story's opinion for the Court in Martin v. Hunter's Lessee:

1) Congress must vest the whole judicial power in some federal court either in original or appellate form 2) For cases described in Art. III that are beyond state court jurisdiction and that don't fall within SCOTUS original jurisdiction, Congress must create lower federal courts. 3) Two sets of cases- Story limited this congressional obligation to the first three categories of cases described in AIII, as to which the Constitution uses the adjective "all" - Focus on mandatory language of AIII: "the judicial power of the United States shall be vested" (not may be vested). "All Cases": (1) cases arising under constitution, laws, and treaties of U.S.; (2) cases affecting ambassadors, other public ministers and consults; and (3) cases of admiralty and maritime jurisdiction Why? - Possibly because these are cases of national importance.

Congress then has Four Options with regard to bestowing jurisdiction:

1) Exclusive state original jurisdiction Not in AIII headings of jurisdiction or w/i areas where Congress conferred federal jurisdiction Example: state-law claims between non-diverse parties; diversity suits below $ in controversy 2) Concurrent federal and state jurisdiction Promotes convenience when easier for litigants to appear in state court Relatively free choice of forum 3) Concurrent state and federal jurisdiction, but with right of state court Ds to remove to federal court; Most common configuration Right to remove depends on contents of the well-pleaded complaint With some exceptions, removal is not allowed based on federal counterclaims or defenses 4) Exclusive federal jurisdiction Desirability of uniform interpretations of federal law Presumptive expertise of federal judges in federal law Likelihood that federal courts will be sympathetic to federal purposes than state courts

Calpper Breakdown

1) Wallet injury: look at what we have to spend- respondents cannot manufacture standing merely by inflicting harm on themselves based on their fears of hypothetical future harm that is not certainly impending 2) Anyone with common sense would know this is likely- too speculative- rests on attenuated chain of possibilities, including decisions by others. Not certainly impending. Court acknowledged that Court's cases have sometimes permitted standing based on substantial risk of harm rather than a literally certain harm. Here, however, Ps' injuries were too speculative to reach even the "substantial risk" threshold. Where injury has not yet occurred, must be certainly impending, but substantial risk of injury would also suffice. Cannot meet either standard here. Court found it speculative that: The gov't would target parties with whom the plaintiffs communicated; That the gov't would rely on the statute at issue (1881(a)) rather than some other source of authority, FISC would authorize the surveillance; The surveillance would successfully intercept intended communication, and The interceptions would include Ps' communications

Capable of repetition yet evading review

1) challenged action is in its duration too short to be fully litigated before cessation or expiration; and 2) there's a reasonable expectation that the SAME complaining party will be subject to the same action again The requisite degree of likelihood is unclear. The Court has spoken of a "reasonable expectation" or "demonstrated probability" that the controversy would recur and insisted that a "mere physical or theoretical possibility" was insufficient.

Are there limits to Testa

1. A limit on this is the anticommandeering principle. Congress cannot commandeer state governments 2. State courts cannot discriminate against a federal cause of action but they are not required to exercise jurisdiction that they otherwise would not have. 3. Several Supreme Court decisions have upheld, as a "valid excuse," non-discriminatory refusals by a state court to entertain a federal cause of action. The leading cases have involved rules relating in some way to the convenience of the state as a forum for resolving the case in question, and the refusal to adjudicate has typically left the courts of another, putatively more convenient state, as well as the federal courts, open to hear the dispute.

Problems with Advisory Opinions

1. Dilemma: if another branch disregards the Court's advisory opinion and then the question actually comes before the Court, the Court can either issue the same result (appearing petty) or acquiesce in the other branch's decision (appearing weak). 2. Bias: court may later be presented with the same question in an actual lawsuit. 3. Inefficiency: deciding issues they might not have had to or might have to again 4. Shield elected officials, allowing them to shift blame onto the judiciary. 5. Separation of Powers: the federal courts require "cases" or "controversies." 6. Due Process: if a party was not part of the prior judgment, their DP rights may be violated. 7. If the Court couldn't determine the constitutionality of legislation before it comes to them in an applied setting, the Court—and not the Executive or Congress—would be the only interpreter of the Constitution; the other branches would have no constitutional role. 8. Protects the adversarial system

Mootness in criminal cases

1. For many years, the general rule in the federal courts was that after criminal defendants had served their sentences, their cases were moot because "there was no longer a subject matter on which the judgment could operate." But that rule gradually eroded because of recognition, first, that issues characteristically involving only short sentences might forever escape review and, second, that criminal convictions have collateral consequences (recidivism statutes, testimonial impeachment, civil disabilities, etc.) 2. The death of a criminal defendant ordinarily moots the defendant's case on direct or collateral review. 3. In federal criminal cases, the Supreme Court held for a time that death abated "not only the appeal but also all proceedings had in the prosecution from its inception", thus requiring dismissal of the indictment. But in Dove v. United States, the Court overruled this holding without discussion and dismissed a petition for a writ of certiorari on learning of petitioner's death.

Gateway claim of innocence

1. Innocent of the crime: more likely than not that no reasonable juror would find guilt beyond a reasonable doubt 2. Ineligible for death penalty: clear and convincing evidence that no reasonable juror would find D eligible for death penalty. -> this is a harder point to make - need to make it more difficult because of the number of defendant.

situations when state procedural rulings will not preclude SCOTUS review of the federal-law issues raised

1. State Rule Unconstitutional: if they deny due process Due Process Violations. (Brinkerhoff-Faris Trust & Savings Co. v. Hill - change from unforeseeable appellate decision)( Herndon v. Georgia -limits Brinkerhoff)( Reece v. Georgia - unreasonably strict time limit- did not receive counsel until after the indictment stage) (Michel v. Louisiana - limits Reece) 2. Unduly Burdensome: If a state procedure is unduly burdensome, thus unreasonably interfering with the assertion of the federal right, an asserted state procedural bar will be disregarded. (Staub v. City of Baxley) 3. Novel State Procedural Rule 4. Discretionary State Procedural Rule: not really applied anymore

Hudson Breakdown

1. The lower courts are of limited jurisdiction and this is not a power granted to them. Only the Supreme Court possesses jurisdiction derived immediately from the constitution, and of which the legislative power cannot deprive it. All other Courts created by the general Government possess no jurisdiction but what is given them by the power that creates them, and can be vested with none but what the power ceded to the general Government will authorize them to confer Certain implied powers must necessarily result to our Courts of justice from the nature of their institution. But jurisdiction of crimes against the state is not among those powers. 2. Court does okay contempt powers though - To fine for contempt—imprison for contumacy—enforce the observance of order, & c. are powers which cannot be dispensed with in a Court, because they are necessary to the exercise of all others: and so far our Courts no doubt possess powers not immediately derived from statute 3. Public opinion: in no other case has this type of jurisdiction been asserted, accompanied by the "acquiescence of legal men" to that lack of jurisdiction.

Klein The brief and delphic opinion includes at least four strands:

1. The opinion stresses that Congress' attempt to regulate jurisdiction is not a talisman that renders any such legislative effort constitutional 2. The opinion continues by broadly questioning the power of Congress to "prescribe rules of decision to the Judicial Department of the government in cases pending before it". It is doubtful, however, that this language can be taken at face value, for congressional power to make valid statutes retroactively applicable to pending cases has often been recognized. This is true even if it removes jurisdiction. 3. More broadly, one could read Klein as supporting a conception of judicial integrity: Congress may not, in the guise of enacting jurisdictional legislation, direct a federal court to decide a case in a fashion that conflicts with the Constitution. If Congress is using SCOTUS appellate jurisdiction to withhold power of another branch then it is unconstitutional. Has to be defined power or other Constitutional restriction 4. The Court suggested that jurisdiction-stripping legislation enacted "as a means to an end" that is itself constitutionally impermissible "is not an exercise of the acknowledged power of Congress to make exceptions and prescriptive regulations to the appellate power". (No mention was made of the Court's statement in McCardle, two years earlier, that it could not inquire into the motives of the legislature.)

Kimbell Factors

1. Whether there is a need for nationally uniform body of law Controversies directly affecting the operations of federal programs, although governed by federal law, do not inevitably require resort to uniform federal rules. Conversely, when there is little need for a nationally uniform body of law, state law may be incorporated as the federal rule of decision. 2. When application of state law would not frustrate specific objectives of the federal programs If it would, we must fashion special rules solicitous of those federal interests 3. How much the application of a federal rule would disrupt commercial relationships predicated on state law: Here, the court incorporates state law: state commercial codes "furnish convenient solutions in no way inconsistent with adequate protection of the federal interest[s]," so we decline to override intricate state laws of general applicability on which private creditors base their daily commercial transactions.

Murdock 7 points

1.That it is essential to the jurisdiction of this court over the judgment of a State court, that it shall appear that one of the questions484mentioned in the act must have been raised, and presented to the State court. 2.That it must have been decided by the State court, or that its decision was necessary to the judgment or decree, rendered in the case. 3.That the decision must have been against the right claimed or asserted by plaintiff in error under the Constitution, treaties, laws, or authority of the United States. 4.These things appearing, this court has jurisdiction and must examine the judgment so far as to enable it to decide whether this claim of right was correctly adjudicated by the State court. 5.If it finds that it was rightly decided, the judgment must be affirmed. 6.If it was erroneously decided against plaintiff in error, then this court must further inquire, whether there is any other matter or issue adjudged by the State court, which is sufficiently broad to maintain the judgment of that court, notwithstanding the error in deciding the issue raised by the Federal question. If this is found to be the case, the judgment must be affirmed without inquiring into the soundness of the decision on such other matter or issue. 7.But if it be found that the issue raised by the question of Federal law is of such controlling character that its correct decision is necessary to any final judgment in the case, or that there has been no decision by the State court of any other matter or issue which is sufficient to maintain the judgment of that court without regard to the Federal question, then this court will reverse the judgment of the State court, and will either render such judgment here as the State court should have rendered, or remand the case to that court, as the circumstances of the case may require.

Availability of federal evidentiary hearing

28 U.S.C. §2254(e)(1): In a proceeding instituted by an application for a writ of habeas corpus by a person in custody pursuant to the judgment of a State court, a determination of a factual issue made by a State court shall be presumed to be correct. The applicant shall have the burden of rebutting the presumption of correctness by clear and convincing evidence. 28 U.S.C. §2254(e)(2): If the applicant has failed to develop the factual basis of a claim in State court proceedings, the court shall not hold an evidentiary hearing on the claim unless the applicant shows that— A)The claim relies on— A new rule of constitutional law, made retroactive to cases on collateral review by the Supreme Court, that was previously unavailable; or A factual predicate that could not have been previously discovered through the exercise of due diligence; and B)The facts underlying the claim would be sufficient to establish by clear and convincing evidence that but for constitutional error, no reasonable factfinder would have found the applicant guilty of the underlying offense. -- How does the §2254(e)(2) standard compare with Sykes? For Sykes it is cause and prejudice OR actual innocence. However here you need to do both. Clear and convincing evidence that but for constitutional error- even more demanding than the innocence test here. ADEPA - made it even more demanding to trigger the statute.

Cullen v. Pinholster

: a prisoner claimed a denial of the effective assistance of counsel at his capital sentencing hearing, a claim rejected on the merits in state courts. In his federal habeas action, the district court held an evidentiary hearing and granted relief. In affirming, the Ninth Circuit en banc ruled that a habeas court could consider evidence adduced in a federal evidentiary hearing permitted by § 2254(c)(2) when determining, under § 2254(d)(1), whether the state court's rejection of a constitutional claim was contrary to, or an unreasonable application of, clearly established federal law. Question for the court was considered whether, when determining if § 2254(d)(1) bars habeas relief, the habeas court may consider evidence introduced in an evidentiary hearing permitted by § 2254(e)(2). § 2254(d)(1)'s use of the past tense—in referring to a state court determination that "resulted" in a decision that was contrary to, or "involved" an unreasonable application of, clearly established law—required an examination of the record when the state court decision was made.

The Anti-Injunction Act § 2283

A court of the United States may not grant an injunction to stay proceedings in a State court except as expressly authorized by Act of Congress, or where necessary in aid of its jurisdiction, or to protect or effectuate its judgments. There are three statutory exception the idea that the federal court cannot enjoin a state court proceeding: 1) it is expressly authorized by Congress 2) It is in aid of their jurisdiction and 3) Relitigation exception

Damage claims as moot?

A damage based claim often cannot be found moot - if not paid then it won't become moot. The injury is in the past and if there is a lawsuit ongoing that means the remedy damages has not yet been provided to P What if they reject the full amount- Campbell- Ewald v. Gomez- an unaccepted settlement offer is a nullity so case is not moot. But the court said the result would be different if a D deposits the full amount of the P's individual claim in an account payable to the plaintiff and the court enters judgment for the plaintiff in that amount.

2254 (d)(1)

An application for a writ of habeas corpus on behalf of a person in custody pursuant to the judgment of a State court shall not be granted with respect to any claim that was adjudicated on the merits in State court proceedings unless the adjudication of the claim (1) resulted in a decision that was contrary to, or involved an unreasonable application of, clearly established Federal law, as determined by the Supreme Court of the United States; or Teague -with respect to claim not adjudicated on the merits Teague applies

Shoshone Mining Co. v. Rutter (1900)

A federal law established conditions for issuance of federal patents for mining claims. The law provided that if, after notice of an application, an adverse claim were filed, "it shall be the duty of the adverse claimant, within thirty days after filing his claim, to commence proceedings in a court of competent jurisdiction, to determine the question of the right of possession"; in turn, the patent should issue in accordance with the judgment in those proceedings. Congress provided that this right of possession could be determined by "local customs or rules of miners in the several mining districts, so far as the same are applicable and not inconsistent with the laws of the United States", or "by the statute of limitations for mining claims of the state or territory where the same may be situated." [It is] well settled that a suit to enforce a right which takes its origin in the laws of the United States is not necessarily one arising under the Constitution or laws of the United States, within the meaning of the jurisdiction clauses Not doing much sort of an oddity-> As a general matter, there is federal jurisdiction when a federal statute creates a federal right of action, even if the statute incorporates state law standards of liability in substantial part

Directing Courts to Reopen a Case

A federal statute directing federal courts to reopen final judgments in private lawsuits violated AIII and separation of powers. Although Congress may not give retrospective relief to a party that the courts rule is entitled to none (e.g., damages), it may alter parties' rights to prospective relief (e.g., an injunction) b/c injunctions, ongoing remedies, are never final + such judgments are already subject to judicial revision.

Initiation and nature of the habeas proceeding

A habeas petition (sometimes called an "application") names as the respondent a state officer having custody of the petitioner—ordinarily the prison warden or the director of the state correctional system. A habeas petition is not an appeal from, but rather a collateral attack upon, the state criminal conviction. Successive petitions: A prisoner may file more than one habeas petition only in exceedingly narrow circumstances Appeals. The custodian may appeal a district court's grant of relief. A prisoner, before appealing a denial of relief, must obtain, from either the district court or the court of appeals a "certificate of appealability," which issues upon "a substantial showing of the denial of a constitutional right" and which must indicate the specific issue(s) satisfying that standard. 28 U.S.C. § 2253(c).

Harlow v. Fitzgerald

A. Ernest Fitzgerald (plaintiff) was a management analyst in the Department of the Air Force. During the Johnson administration, Fitzgerald testified to a congressional committee regarding technical problems and cost overruns on the C-5A cargo plane. Fitzgerald's testimony was embarrassing to the Defense Department, and a memo outlined a way to discharge Fitzgerald using a reduction in force despite Fitzgerald's civil service protection. Approximately one year later, the Nixon administration carried out the reduction in force and eliminated Fitzgerald's position. Fitzgerald sued Nixon, and two senior White House aides, Bryce Harlow and Alexander Butterfield (defendants). Discovery progressed for eight years. Nixon, Fitzgerald, and Harlow moved for summary judgment based on immunity. The trial court denied the motion for summary judgment, and Nixon, Fitzgerald, and Harlow appealed. Government officials performing discretionary functions, generally are shielded from liability for civil damages insofar as their conduct does not violate clearly established statutory or constitutional rights of which a reasonable person would have known.

Cause

An elaborate series of decisions has given "cause" a very restricted meaning, embracing only these categories: (a) reliance on a novel constitutional claim, (b) deficient performance by counsel that is serious enough to constitute ineffective assistance of counsel under the standards established by the Sixth Amendment, and (c) the state's creation of an "external impediment" to presentation of the claim

Board of Trustees of the University of Alabama v. Garrett

ADA prevents discrimination against disabled; must make reasonable accommodations P diagnosed w/ breast cancer and had to take substantial leave for treatment; she was told that she would have to give up her Director position and take a job paying much less. The ultimate guarantee of the 11A is that nonconsenting states may not be sued by private individuals in federal courts. The Supreme Court has recognized, though, that Congress may abrogate States' 11A immunity when it both (1) unequivocally intends to do so and (2) acts pursuant to a valid grant of constitutional authority. Congress' abrogation of state immunity from damages actions for violation of Title I of the Americans with Disabilities Act—the Title prohibiting discrimination in employment—could not be sustained because there was insufficient evidence of state discrimination in violation of the Fourteenth Amendment; the record of discrimination by local governments was irrelevant - Record needed to violation of equal protection by state not local entities and it was not sufficient here Why not counting the bad behavior of the municipal? Because we are looking for whether we can abrogate state sovereign immunity so we look only to misconduct by states and not to entities that don't have sovereign immunity in the first place. - Fails Boerne - Not a protected class (Cleburne) + no long history of discrimination by the states - Legislative record of state discrimination was not strong enough & discrimination by local gov'ts didn't count

Kimel v. Florida Board of Regents

ADEA prohibits discrimination on the basis of age (40+) with an exception for bona fide occupational qualifications (BFOQ) reasonably needed for normal operation of a particular business. FSU faculty and librarians sue state for not allocating funds for market adjustment of salaries, which has a disparate impact on base pay for older employees. This suit concerns ADEA damages for remedies against states and the invocation 11A. § 5 could not furnish a basis for overcoming a state's immunity from private suit under the Age Discrimination in Employment Act because "the substantive requirements the ADEA imposes on state and local governments are disproportionate to any unconstitutional conduct that conceivably could be targeted by the Act" Here they are needing to remedy an equal protection violation, but age is not a suspect class as there is no hatred or lineage of discrimination-> apply Rational Basis to see the scope of the judiciaries view on this class -> states may discriminate on the basis of age without offending the 14th if the age classification in question is rationally related to a legitimate state interest--> The act on the other hand through its broad restriction on the use of age as a discriminating factor prohibits substantially more state employment decisions and practices.-> A lot of ADEA violations would not violate EP.

Congress and Article III Courts

AIII does not set up courts itself, but instead lets Congress "from time to time ordain and establish" lower federal courts-> Madisonian Compromise. Four sources of congressional authority are particularly important.: 1. Under the resolution reached by the "Madisonian Compromise", Congress is authorized but not obliged to "ordain and establish" federal tribunals "inferior" to the Supreme Court—a power that has generally been understood to permit establishing lower federal courts whose jurisdiction is more limited than the Constitution would allow. 2. The Exceptions Clause. Article III, § 2, cl. 2 specifies that the appellate jurisdiction of the Supreme Court shall be subject to "such Exceptions, and under such Regulations as the Congress shall make". 3. Limitation of State Court Jurisdiction. Under the Necessary and Proper Clause, Congress may limit state court jurisdiction, by providing for exclusive federal jurisdiction in some cases and for removal in others. 4. Legislative Courts. Under its legislative authority in Article I (and in Article IV, Sec. 3, relating to the governance of territories), Congress has been understood to have power to assign at least the initial adjudication of some claims to "legislative courts"—federal tribunals whose judges lack the tenure and salary protections afforded to the Article III judiciary.

Heyburn's Case: NY

Act here improper as it allows the Secretary of War and Congress to basically review the Court's decisions on the matter. Instead read the act to ask them to serve as commissioners, which they accepted.

Madison v. Marbury

Adams had lost the U.S. presidential election of 1800 to Jefferson, and in March 4, 1801, Adams and Congress passed the Judiciary Act of 1801, which created new courts, added judges, and gave the president more control over appointment of judges. Just two days before his term as president ended, Adams appointed several dozen Federalist Party supporters to new circuit judge and justice of the peace positions. Jefferson believed the commissions were void because they had not been delivered in time, and instructed his new Secretary of State, James Madison, not to deliver them. The supreme court can and must engage in Constitutional interpretation in order to reach certain decisions. To say otherwise would strip the importance of the constitution. Congress also cannot increase the courts original jurisdiction-> All of Art. III, §2, Cl. 2 would be surplusage if things could be moved from appellate to original juris Chief Justice Marshall's opinion for the Court also grappled with another question that we take as a given today: judicial authority to judge the legality of actions by the officer of a coordinate branch and to direct that officer to comply with federal law.

Havens Realty Corp. v. Coleman (1982)

African American "tester" posed as a renter/buyer of housing to collect evidence of racial steering practices, hand standing to seek equitable and monetary relief against private parties under §804 of the Fair Housing Act Conferred legal right not to be denied truthful housing information re: housing availability on the basis of race. Injury was conceived of more broadly than being denied housing yourself, but denial of truthful information as the harm. Congress recognized as legally cognizable. Court found that Black tester who got wrong information had standing but White tester did not have standing under this theory. Difference between the two groups: Plaintiffs were not actually living there. They were testers - Congress has thus conferred on all persons a legal right to truthful information about available housing.

Modern Analysis Implied Statutory right

After Cannon, focus on second factor of the Cort v. Ash test. Look at affirmative congressional intent to create a cause of action and if there is none the inquiry stops. Reject significance of legal context only matters to the extent that it clarifies the text no independent valence other than this clarifying role

United States v. Jones

After Congress amended the statute to remove language about not paying objectionable claims, Court found it could entertain appeals—case was justiciable. SCOTUS has jurisdiction to entertain appeals from Court of Claims (AI court). This review of a judicial decision by a legislative court was an exercise of appellate jurisdiction within the meaning of AIII. Not an article III court- can make appellate decision though. Can they? State courts - this is a legislative court- created under congress not under Article III. The court says we have appellate jurisdiction now- does not give legislative power to revise the judgment. Article I court is being reviewed by an Article III court

Danforth v. Minnesota

After Danforth's conviction became final, the Supreme Court decided Crawford v. Washington, 541 U.S. 36 (2004), which reshaped a defendant's rights under the Confrontation Clause. Crawford announced a "new" rule not retroactively applicable in federal habeas corpus proceedings. When Danforth filed a state postconviction petition relying on Crawford, the state supreme court ruled that it could not give retroactive effect to a federal constitutional ruling that, under Teague, does not apply retroactively in federal habeas corpus proceedings. State court are free to apply a new rule even when the federal habeas courts under Teague would not. Court says it is about comity- who are we to say a state cannot do this The Supreme Court reversed. Justice Stevens' majority opinion reasoned that the Court's retroactivity jurisprudence "is primarily concerned, not with the question whether a constitutional violation occurred, but with the availability or nonavailability of remedies."

Friends of the Earth v. Laidlaw Environmental Services (TOC), Inc.

After Laidlaw Environmental Services, Inc. bought a wastewater treatment plant, it was granted a National Pollutant Discharge Elimination System (NPDES) permit. The permit authorized Laidlaw to discharge treated water and limited pollutants. Laidlaw's discharge of mercury into the North Tyger River repeatedly exceeded the limits set by the permit. Ultimately, Friends of the Earth and others (FOE) filed a citizen suit under the Clean Water Act against Laidlaw, alleging noncompliance with the NPDES permit, seeking injunctive relief and an award of civil penalties. Laidlaw moved for summary judgement on the ground that FOE lacked standing to bring the lawsuit. The District Court denied the motion. Ultimately, the District Court found that Laidlaw violated the mercury discharge limitation. In issuing its judgment, the District Court concluded that a civil penalty of $405,800 would be adequate to forestall future violations, given that Laidlaw would have to reimburse the plaintiffs for a significant amount of legal fees and had itself incurred significant legal expenses. The Court of Appeals held the case moot; it reasoned that all elements of Article III standing must persist throughout litigation and that the only remedy available once the defendant's violations had stopped—civil penalties payable to the government—would not redress any injury to the plaintiff. Something too speculative for standing but fine for moot- some of what standing doctrine is doing is setting facts. Mootness- we have facts.

Procedural Defaults in State Postconviction Proceedings Challenging the Effectiveness of Counsel. Martinez v. Ryan

After Martinez's conviction became final on direct review, his lawyer commenced a state postconviction proceeding but made no claim that Martinez's trial counsel had been ineffective, instead stating that she could not identify any colorable claim to raise on Martinez's behalf. Martinez did not respond to the trial court's invitation to raise any claims he felt his lawyer had overlooked, and the court then denied postconviction relief. When a new lawyer launched a second state postconviction proceeding, alleging that Martinez's trial counsel had been ineffective, it was dismissed on the basis of a state rule barring a claim that could have been raised in a prior state postconviction proceeding. When a state requires a prisoner to raise an ineffective-assistance-of-trial-counsel claim in a collateral proceeding rather than on direct review, and either No counsel was appointed, or Counsel was ineffective under Strickland, This establishes cause. Does the Martinez rule apply to ineffective-assistance-of-appellate-counsel claims? No - Davila v. Davis (2017) - applies only to trial counsel

Eleventh Amendment Immunity and Bankruptcy

After Seminole, there was some disagreement in the lower federal courts about whether Congress might be able to abrogate the states' immunity under any of its Article I powers, including that involving patents and copyrights, though that was dispelled in Florida Prepaid. To the surprise of many observers, however, the Court has found that somewhat different rules may apply to congressional efforts to subject the states to suit in the domain of federal bankruptcy law. Central Virginia Community College v. Katz: a proceeding initiated by the federally appointed trustee of a bankrupt's estate to set aside, and recover for the estate, certain allegedly preferential transfers made by the debtor to several state agencies. The state agencies moved to dismiss the proceedings on grounds of state sovereign immunity; the motions were denied by the bankruptcy court, the district court, and the court of appeals, and the Supreme Court affirmed. Bankruptcy clause gives Congress the power to authorize courts to void preferential transfers and recover transferred property. This authority operates free and clear from the states' claims of sovereign immunity.

Terry Williams v. Taylor

After Terry Williams was convicted of robbery and capital murder; his punishment was fixed at death. In state habeas corpus proceedings a judge determined that his conviction was valid. However, the judge also found that Williams' counsel's failure to discover and present significant mitigating evidence violated his right to effective counsel and recommended that he be re-sentenced. Rejecting this, the Virginia Supreme Court held that Williams had not suffered sufficient prejudice to warrant relief. In habeas corpus proceedings under the Antiterrorism and Effective Death Penalty Act of 1996 (AEDPA), a federal trial judge also found that the death sentence was constitutionally weak on ineffective-assistance grounds. The court, under the AEDPA, concluded that the Virginia Supreme Court's decision "was contrary to, or involved an unreasonable application of, clearly established Federal law, as determined by the Supreme Court of the United States." In reversing, the Court of Appeals determined that it could not conclude that the Virginia Supreme Court's decision on prejudice was an unreasonable application of standards established by the Supreme Court. Under § 2254(d)(1), the writ may issue only if * * * [(1)] the state court arrives at a conclusion opposite to that reached by this Court on a question of law or if the state court decides a case differently than this Court has on a set of materially indistinguishable facts[,] or (2) * * * the state court identifies the correct governing legal principle from this Court's decisions but unreasonably applies that principle to the facts of the prisoner's case.

Shinn v. Kayer

After being convicted of murder, the petitioner "refused to fully cooperate with a mitigation specialist" retained by his counsel. The state trial court - finding that there was only a single mitigating factor (petitioner's importance to his son) and that this mitigating factor failed to outweigh two aggravating factors - sentenced the petitioner to death. In his subsequent state-court postconviction proceeding, the petitioner claimed ineffective assistance of counsel in the sentencing process. The state trial court denied relief, holding that neither the deficient performance prong nor the prejudice prong of the Strickland test was met. The Arizona Supreme Court denied review. The petitioner's request for federal habeas relief was denied by the district court but was granted by a divided panel of the Ninth Circuit, with the panel majority reasoning that petitioner's sentencing counsel was deficient for tardiness in commencing work on the mitigation evidence, and that this deficiency prejudiced the petitioner because effective assistance would have presented (at sentencing and on direct appeal) evidence of mental impairment that could have led the state courts to avoid imposing the death penalty. Because, it argued, ineffective-assistance claims could permit end-runs around waiver and forfeiture principles. It also noted that Strickland sets a general standard, and that the more general the standard, the greater the state courts' latitude in applying it.

Plaut v. Spendthrift Farm, Inc

After the Court interpreted a statute to have created a short statute of limitations, Congress amended it to provide a longer one and to reinstate certain actions that SCOTUS had dismissed as time-barred. In Plaut, Ps lose based on the Lampf rule. Judgment is entered and appeal time runs out. - SCOTUS says that Congress is stomping on the final judgment. It is not okay for Congress to say courts you have to reopen their cases and start again. Congress can change the substantive law, but it cannot simply direct a court to hear a case that has already been decided. - If it was still on appeal- courts will apply the law at the time of the decision. We have to pick a point where the court becomes sticky in order for courts to function as courts.

Patsy v. Board of Regents of the State of Florida

Alleging that her employer, a state university, had discriminated against her on the basis of race and gender, Patsy filed a civil rights action in federal district court. The district court dismissed, based on Patsy's failure to exhaust administrative remedies provided by the university itself. The en banc court of appeals reversed, ruling that a § 1983 plaintiff was required to exhaust administrative remedies when (but only when): (i) an orderly system of review is provided by statute or agency rule; (ii) the agency can grant relief more or less commensurate with the claim; (iii) relief is available without undue delay; (iv) the procedures are fair, not burdensome, and are not used to harass those with legitimate claims; and (v) interim relief is available in appropriate cases. Exhaustion of state administrative remedies is not required in actions under 42 U.S.C. § 1983.

Prentis Breakdown

Although "exhaustion" doctrine requires plaintiffs to avail themselves of state administrative remedies, but not to sue in state court, before proceeding to federal court, some of the generative cases, including Prentis, are potentially confusing because they arose from circumstances in which state courts acted in administrative rather than judicial capacities. If in legislative capacity you need to exhaust it, if it is judicial you need to not What about the preclusion problem? If it is in the judicial role then the Federal court must give preclusive effect to the state court determination under 1783. Really requires you to guess right in the first instance. When state courts act in legislative/administrative capacity litigants can challenge state court ruling in federal district court without facing preclusion When state courts act in a judicial capacity the litigant cannot challenge the state court ruling in federal district court and would face preclusion in a collateral proceeding. Litigants can only seek out SCOTUS review.

Abrogation under the 14th Amendment following Seminole

Although Congress retains authority to abrogate the states' immunity when legislating pursuant to § 5, decisions subsequent to Seminole have emphasized the limits on Congress' § 5 power. In general, For § 5 legislation to be valid, it must exhibit "congruence and proportionality between the [constitutional violations] to be prevented or remedied and the means"—including abrogation of the states' immunity—"adopted to that end." City of Boerne v. Flores (1997): The Court held the Religious Freedom Restoration Act unconstitutional insofar as it attempted to overrule a Supreme Court decision interpreting the First Amendment as it applied to the States through the Fourteenth Amendment. The Court reasoned that although Congress' power under § 5 extends to the creation of remedies, it does not include the power to alter substantive rights. It laid down the principles that also govern Congress' abrogation power: For § 5 legislation to be valid, it must exhibit "congruence and proportionality between the [constitutional violations] to be prevented or remedied and the means"—including abrogation of the states' immunity—"adopted to that end." Institution that can say what the constitution means - we can say what section 1 means and cannot change what the constitution means- can't change Smith. You can only enforce- S5 What the court is saying then is that legislation under the 14A can be prophylactic, but only within certain bounds. Applied here, this was prohibiting too many things that would not have violated the FEC as the court had interpreted Congruent and Proportional: identify to what Congress is responding to and then identify what they have done in response to see whether or not it is proportional

National Security Function

Although Harlow held that absolute immunity might be appropriate for presidential aides with discretionary authority in national security matters or foreign affairs, the Court rejected a claim of absolute immunity in Mitchell v. Forsyth, 472 U.S. 511 (1985), a Bivens action against former Attorney General John N. Mitchell for having authorized a warrantless wiretap for the purpose of protecting national security. The Court reasoned that the secrecy of national security matters reduced both the likelihood of unfounded and burdensome lawsuits and the effectiveness of other possible mechanisms of restraining misconduct.

Contemporary Common Lawmaking in Criminal Cases

Although non-statutory prosecutions ended with Coolidge, the federal courts continue to make law in criminal cases through certain ways: 1) Contempt power: Federal courts have continued to enforce law by use of the contempt power—a role the Court approved in the last paragraph of the Hudson opinion—and to use a variety of common law techniques, forms, and writs in the enforcement of congressionally defined crimes 2) Supervisory power`The Supreme Court has recognized that federal courts may exercise a "supervisory power to formulate and apply proper standards for enforcement of the criminal law in the federal courts" - Marshall v. United States (1959): the Court, invoking this "supervisory power", set aside a jury verdict when the jurors had been exposed to potentially prejudicial publicity. - But see United States v. Williams (1992), the Court ruled that the supervisory power did not extend to prescribing standards of prosecutorial conduct before a federal grand jury. - 3) Crafting defenses; In the face of Congress' failure, in general, to prescribe the scope or even the existence of defenses (e.g., self-defense or duress) to federal crimes, federal courts have freely crafted such defenses. 4) Interpreting statutory definitions of a crime.-Fill ambiguous gaps

Territorial Courts

American Ins. Co. v. Canter: Chief Justice Marshall upheld employment of non-Article III courts to adjudicate an admiralty dispute in the then-territory of Florida. Territorial courts, the Chief Justice said, were created in virtue of every government's general right of sovereignty, or in virtue of Article IV, Sec. 3's conferral on Congress of authority to "make all needful Rules and Regulations respecting the Territory belonging to the United States." As courts of general jurisdiction, territorial courts resemble state courts, which need not satisfy Article III's strictures. Palmore v. United States: upheld the constitutionality of a criminal prosecution before the local, non-Article III courts that Congress created in the District of Columbia, pursuant to Article I's conferral of legislative authority over the District

The Significance of a "Sue and Be Sued" Clause.

American Nat'l Red Cross v. S.G., (1992), a tort suit alleging that one of the plaintiffs had contracted AIDS from a blood transfusion. In upholding the Red Cross' removal of the action from state court, the Court ruled that the congressional charter authorizing the Red Cross "to sue and be sued in courts of law and equity, State or Federal, within the jurisdiction of the United States" conferred federal question jurisdiction. The Court viewed Deveaux, Osborn, and other precedents as supporting "the rule that a congressional charter's 'sue and be sued' provision may be read to confer federal court jurisdiction if, but only if, it specifically mentions the federal courts." Majority says this is a grant- it says federal courts rather than just general sue or be sued. Lightfoot v. Cendant Mortgage Corp: Fannie Mae's corporate charter authorized it to sue and to be sued and to complain and to defend in any court of competent jurisdiction State or Federal. Because it says any court or competent jurisdiction- no grant Together: sue and be sued grants if it mentions federal court unless it also injects an agnostic view - "any court of competent jurisdiction".

Common Law Theories of Statutory Interpretation

An important strain of legal thought has long maintained that American judicial power should be understood to include common law powers in relation to statutes. Professor Eskridge argues that the original understanding of "the judicial Power of the United States" would have included the power to engage in equitable interpretation, a form of inherent authority that, as relevant here, empowered English judges to extend a statute to cover omitted cases that fell within the statute's reason or purpose.

The Cause of Action Test:

American Well Works Co. v. Layne & Bowler Co.: merican Well Works Company had a patent for a specific pump that it owned, manufactured and sold, which was known as the best in the market. American sued its competitor, Layne & Bowler Company, in Arkansas state court, alleging Layne falsely and maliciously slandered American's patent rights to the pump and falsely claimed the pump and its parts were infringements of Layne's patented pumps A suit arises under the law that creates the cause of action The "cause of action" test that Justice Holmes announced is more useful for inclusion than for the exclusion for which it was intended. With only the most uncertain and limited exceptions § 1331 confers federal question jurisdiction when the plaintiff's complaint pleads a non-frivolous federal cause of action. But Holmes' test fails as a rule of exclusion. § 1331 has been upheld in some cases not involving a federal cause of action, on the basis that a state law cause of action incorporates a question of federal law in a fashion that merits the exercise of federal question jurisdiction

City of Los Angeles v. Lyons

An African-American male, brought a civil rights action against the city and certain of its police officers in federal district court, claiming that he had been unconstitutionally subjected to a "chokehold" after being stopped for a traffic violation. He alleged that pursuant to official authorization, chokeholds were routinely applied in situations where they were not warranted and that many people had suffered injury as a consequence. Lyons sought both damages and declaratory and injunctive relief. The district court granted a preliminary injunction against the use of chokeholds "under circumstances which do not threaten death or serious bodily injury"—an injunction that was to continue in effect until an improved training and reporting program had been approved by the court—and the court of appeals affirmed. 1) need standing for each type of remedy + 2) "no more than speculation to assert either that Lyons himself will again be involved in one of those unfortunate instances, or that he will be arrested in the future and provoke the use of a chokehold by resisting arrest, attempting to escape, or threatening deadly force or serious bodily injury." Not ripe because it is speculative as to whether he would be injured again and no standing because the request for a forward-looking injunction failed the redressability requirement because no injunction could redress the past harm. But was it not inevitable? There was a policy supporting it.

Section 2254(b) (1

An application for a writ of habeas corpus on behalf of a person in custody pursuant to the judgment of a State court shall not be granted unless it appears that--- (A) the applicant has exhausted the remedies available in the courts of the State; or- (B)(i) there is an absence of available State corrective process; or•(ii) circumstances exist that render such process ineffective to protect the rights of the applicant."

2254 (d)(2)

An application for a writ of habeas corpus on behalf of a person in custody pursuant to the judgment of a State court shall not be granted with respect to any claim that was adjudicated on the merits in State court proceedings unless the adjudication of the claim (2) resulted in a decision that was based on an unreasonable determination of the facts in light of the evidence presented in the State court proceeding

Seems to be that they are usually upholding the lower courts determination that there is qualified immunity- other times SCOTUS faults the lower court for defining the relevant right at to high a level of generality.

Anderson v. Creighton: involving the unquestioned Fourth Amendment rule that a warrantless search of a home is impermissible absent probable cause and exigent circumstances. Reversing a ruling by the court of appeals, the Supreme Court held (6-3) that the availability of qualified immunity should turn not on the general right, but on its application to particular facts: "The contours of the right must be sufficiently clear that a reasonable official would understand that what he is doing violates that right" A court must conduct a two-step process: (1) determining whether a constitutional violation had occurred because of the officer's unreasonable behavior and, if so, (2) determining whether it was "reasonable" for the officer to have been unaware of the legal significance of his conduct -> Would it have been clear to a reasonable officer under the circumstances that his or her conduct was unlawful under the circumstances Saucier- gives a lot of latitude to the officers White v. Pauley: Court noted the importance of the immunity to society as a whole and emphasized that its prior cases on deadly force to not by themselves create a clearly established law outside of obvious care. Kiesla v. Hughes (2018): involved a police shooting that invoked issues of excessive force. The court stated "This court has repeatedly told courts and the Ninth Circuit in particular - not to define clearly established law at a high level of generality.

Congress and Remedy

Any claim that the Constitution guarantees any particular remedy, or indeed even that it guarantees some remedy, for a constitutional violation can be in some tension with well-established doctrines like the political question doctrine and sovereign immunity. Marbury: it is a general and indisputable rule, that where there is a legal right, there is also a legal remedy by suit or action at law, whenever that right is invaded.' " But Marbury, of course, claimed a violation of a non-constitutional right to a judicial commission. The constitutional text refers to only two remedies: (1) a right to just compensation for takings and (2) the privilege of the writ of habeas corpus, a means of contesting the legality of executive detention. Congress's power to choose among remedies. Government can say that you can have some remedy even if it is not your favorite.

Exceptions" to Hans

Appeals of Actions Initiated by the States in State Court. Appeals to the Supreme Court of state criminal convictions are not suits against the state within the meaning of the Eleventh Amendment. Neither are appeals of civil actions instituted by the states. Suits by Other States and the United States. The Supreme Court has declined to bar either suits against a state by another state or suits against a state by the US. In Monaco v. Mississippi, the Court explained that the former holding "was essential to the peace of the Union" and "a necessary feature of the formation of a more perfect Union", while the latter was "inherent in the constitutional plan" Waiver and Abrogation. Beyond the waiver doctrine, the Court has held that in certain limited circumstances, Congress may abrogate, or strip a state of, Eleventh Amendment immunity. Suits Against Local Governments. Lincoln County v. Luning, 133 U.S. 529 (1890), decided the same day as Hans, held that the Eleventh Amendment does not bar an individual's suit in federal court against a county for nonpayment of a debt. In allowing suits against counties and municipalities, the Court was unanimous, relying in part on its "general acquiescence" in such suits over the prior thirty years. The Court has adhered to this position as to local government bodies ever since. William Fletcher explains the different treatment on the ground that in the nineteenth century, a municipal corporation was viewed as more closely analogous to a private corporation than to a state government. Officer Suits. Hans had no occasion to address the availability of "officer suits" as a device for ensuring state compliance with the Constitution and laws of the United States.

Retroactivity

Application to activity before the case was decided. Should that tradition of full retroactivity also apply to all pending or future cases of collateral attack on a final conviction? The retroactivity question grew in significance during the 1960s as a result of the sharp increase in the number of habeas petitions filed, the expansive scope of habeas review recognized by the Warren Court, and that Court's broad and novel criminal procedure decisions. Linkletter v. Walker (1965): the Court for the first time asserted the power to render a constitutional decision that was not fully retroactive, ruling that its decision in Mapp v. Ohio, 367 U.S. 643 (1961), which applied the Fourth Amendment's exclusionary remedy to the states, would not be retroactively applied to state court convictions that had become final before Mapp was decided Johnson v. New Jersey (1966): the Court held that the Miranda rules did not apply to trials that commenced before the date of the Miranda decision. Eventually, the Court held that the retroactivity of new constitutional rulings depended on three factors: 1) the purpose of the new rule, 2) the extent of reliance on the old rule, and 3) the effect on the administration of justice of retroactive application of the new rule.

Arizona v. United States

Arizona attempted to (a) make it a crime for an alien to fail to comply with federal registration requirements; (b) make it a crime for an unauthorized alien to work or apply for employment in the state; and (c) authorize state law enforcement authorities to arrest a person on probable cause that he or she has committed an offense that makes that individual removable. The court found this pre-empted. At the outset of its analysis, the Court emphasized the federal government's "broad, undoubted power" over immigration and the potential foreign relations implications of the treatment of foreign nationals within U.S. borders. The Court also noted the pervasiveness of federal regulation of immigration-> entered a field that federal law had fully occupied or stood as an obstacle to the purposes of federal law

Marbury Q: Constitutional Q

Article III - SCOTUS original jurisdiction- court says some things are appellate, some are original and we cannot divide that up. This is original jurisdiction being sought but because he is not on the list, the court says he must come through appellate jurisdiction. Of course they can expand the appellate jurisdiction- can assign the lower federal courts power so that they can have original jurisdiction- expand the appellate jurisdiction.

The "Diversity" Interpretation of the Eleventh Amendment.

Atascadero State Hosp. v. Scanlon (1985): Justice Brennan Dissent attempted to make sense of the language of the Eleventh Amendment as reflecting a historical purpose that would have permitted federal jurisdiction in cases such as Hans in which plaintiffs assert federal causes of action. Justice Brennan's "diversity theory" rested on two central distinctions. - First, he sharply differentiated "sovereign immunity"—a traditional concept barring uncontested suit against the sovereign in any court—from the jurisdictional bar to suit in federal court erected by the Eleventh Amendment. The Eleventh Amendment, he argued, had nothing to do with sovereign immunity (in suits in state court, for example); it was designed exclusively to regulate the scope of federal judicial power. - Second, Justice Brennan distinguished between two grounds of federal jurisdiction under Article III: jurisdiction based on subject matter (such as suits arising under federal law) and that dependent on party status (such as citizen-state diversity). The Eleventh Amendment, he contended, barred federal jurisdiction in suits based on party status, but not those based on subject matter.

Atlantic Coast Line RR v. Brotherhood of Locomotive Engineers

Atlantic Coast Line Railroad Company (ACL) (plaintiff) owned Moncrief yard, a railroad switching yard near Jacksonville, Florida. In 1967, members of the Brotherhood of Locomotive Engineers union (BLE) (defendant) began picketing use of Moncrief yard over a labor dispute with their third-party employer. After being denied an injunction in federal court, ACL was granted an injunction in state court. In 1969, the United States Supreme Court held that unions had a right to picket under the Railway Labor Act, 45 U.S.C. § 151et seq., and that the right was not subject to state court injunctions. Brotherhood of Railroad Trainmen v. Jacksonville Terminal Co., 394 U.S. 369 (1969). BLE then filed a motion in state court to dissolve ACL's injunction, but the state court refused, holding that the decision in Brotherhood v. Jacksonville Terminal was not controlling. Rather than appeal in state court, BLE filed a motion in federal court seeking an injunction against the enforcement of the state court injunction in favor of ACL. The respondent union does contend that the injunction was proper either as a means to protect or effectuate the District Court's 1967 order, or in aid of that court's jurisdiction. We do not think that either alleged basis can be supported. The first federal law suit only involved federal claims - you are not entitled to an injunction, but it did not address the state claims-> It is about relitigation and the state law claims were not litigated. Rather we are convinced that the union in effect tried to get the Federal District Court to decide that the state court judge was wrong in distinguishing the Jacksonville Terminal decision. Such an attempt to seek appellate review of a state decision in the Federal District Court cannot be justified as necessary "to protect or effectuate" the 1967 order

Northern Pipeline Construction Co. v. Marathon Pipe Line Co:

Bankruptcy court judges from the Bankruptcy Act of 1978: appointed by the president but only to 14 years terms subject to removal for specified by circuit judicial council. Not Article III judges pretty clearly/ Bankruptcy court- broad subject matter jurisdiction- could hear different claims that could effect the property of the estate. Extended to state law disputes as well. Debtor brought contract (state law) claim against non-creditor. - Public rights disputes include only disputes to which government is a party Here the dispute is between 2 private parties concerning state-law created private rights. Completely apart from anything Congress would have created as a right of action. Adjunct Theory: Can't be upheld under adjunct theory: AI court is only adjunct where adjunct's functions are limited so that the AIII court retains the essential attributes of judicial power. Here their power was just too big and they were only subject to clear error review.

Battaglia v. General Motors Corp

Battaglia and other employees (plaintiffs) of General Motors Corporation (General Motors) (defendant) brought four separate suits against their employer for monetary damages under the Fair Labor Standards Act of 1938 (FLSA), 29 U.S.C. §§ 201-219, for failure to pay overtime wages. The suits were brought as a result of three 1946 United States Supreme Court decisions involving miners that interpreted "work week" to include underground travel time and other preliminary and incidental activities. Those cases spurred almost two thousand lawsuits from July 1946 through January 1947, seeking more than five billion dollars for unpaid overtime. Congress reacted by passing the "Portal-to-Portal" Act of 1947, 29 U.S.C. §§ 251-62, on May 14, 1947, which, under sections 2(a) and (b), eliminated employer liability under the FLSA for failure to pay employees for the type of preliminary and incidental activities that were the subject of the Supreme Court decisions. Section 2(d) of the Act also removed jurisdiction of federal and state courts over these wage claims, regardless of whether the claims were brought before or after May 14, 1947. The Portal-to-Portal Act became effective while the Battaglia claims were pending in district court. It was the duty of the court to ascertain whether it had jurisdiction before proceeding to hear and decide the case on the merits. We think, however, that the exercise of Congress of its control over jurisdiction is subject to compliance with at least the requirements of the Fifth Amendment. Most basic point- not willing to just look to jurisdictional limit with first bein sure that Congress is not a depriving of Due Process. Here the contract is their property.

Alexander v. Sandoval

Because it is a recipient of federal financial assistance, the Alabama Department of Public Safety is subject to Title VI of the Civil Rights Act of 1964. Section 601 of Title VI prohibits discrimination based on race, color, or national origin. Under section 602, the Department of Justice issued a regulation forbidding funding recipients to utilize criteria or administrative methods having the effect of subjecting individuals to discrimination based on the prohibited grounds. Martha Sandoval brought a class action suit to enjoin the Department from administering state driver's license examinations only in English. Sandoval argued that the English-only policy violated the DOJ regulation because it had the effect of subjecting non-English speakers to discrimination based on their national origin. Ordering the Department to accommodate non-English speakers, the District Court enjoined the policy. The Court of Appeals affirmed. James Alexander, the Director of the Department, unsuccessfully argued before both courts that Title VI did not provide a cause of action to enforce the regulation. Language in a regulation may invoke a private right of action that Congress through statutory text created, but it may not create a right that Congress has not. The court has recently insisted that the question whether to imply a remedy should be regarded essentially as a conventional question of statutory interpretation, viewed in terms of "legislative intent".

The New Textualism

Beginning in the late twentieth century, an approach that came to be known as the "new textualism"challenged the Legal Process school's purposive assumptions as inconsistent with the constitutional structure and the realities of the legislative process. - First, the leading judicial proponent of textualism (Justice Scalia) emphasized that Article I, § 7 of the Constitution prescribes the elaborate and cumbersome requirements of bicameralism and presentment and that judges must therefore pay close attention to the details of the enacted text, which is the only expression of policy that has made its way through that process. The process is simply too complex, opaque, and path dependent to allow judges to reconstruct what Congress would have intended to do about a matter that the text itself does not conclusively resolve. Furthermore, textualists stressed that the framing of statutory policy entails not merely the articulation of legislative purposes, but also the specification of the means for carrying out those purposes

Unforeseeable Appellate Court Rulings.

Brinkerhoff-Faris Trust & Savings Co. v. Hill (1930), an equal protection challenge to a state tax was denied by the state appellate court because the taxpayer had failed first to seek administrative relief (which was no longer available)—even though earlier state decisions had held that the state administrative body lacked power to award relief. The taxpayer's petition to the state appellate court, objecting to this shift of course, was denied. The Supreme Court viewed the state court's action as a denial of due process, and thus, brushing aside claims of procedural default, reversed and remanded for the state court to consider the equal protection issue on the merits. But see Herndon v. Georgia (1935). Herndon, an organizer for the Communist Party, was convicted of attempting to incite insurrection, after the trial court instructed the jury that the defendant must have expected or advocated immediate serious violence against the state. The Supreme Court of Georgia, in rejecting Herndon's contention that the evidence was insufficient to convict him under the statute as interpreted by the trial court, construed the statute as not in fact requiring proof of the immediacy of violence. Herndon then challenged that construction of the statute as a violation of his First Amendment rights—first on a motion for rehearing in the state supreme court (which was denied), and then on appeal to the United States Supreme Court. The Supreme Court refused to hear the First Amendment issue, ruling that Herndon had defaulted by not having challenged the statute until filing his motion for rehearing. The Court reasoned that while Herndon's motion for a new trial was pending in the trial court, the Supreme Court of Georgia had decided another case that, in the Court's view, had construed the statute as not requiring proof of the immediacy of violence; thus, the Georgia Supreme Court was justified in holding that Herndon should have anticipated the construction of which he complained and should have challenged it in his initial appeal in state court.

Textile Workers Union v. Lincoln Mills Concurrence and Dissent

Burton and Harlan- no FCL but protective jurisdiction: "that some federal rights may necessarily be involved in a § 301 case, and hence that the constitutionality of § 301 can be upheld as a congressional grant to Federal District Courts of what has been called 'protective jurisdiction.' Frankfurter - no FCL so cannot send them to federal court just because the unions are involved in the litigation. He suggests that Congress either has to make the substantive law or the suit will be in state court. He objected to the breadth of congressional power under Wechsler's theory, noting that it would permit conferral of federal court jurisdiction over every state law contract or tort affecting commerce. Frankfurter also suggested that insofar as protective jurisdiction rested on mistrust of state tribunals, Article III confined the grant of jurisdiction on this basis to cases falling within its party-based clauses, notably the diversity clause. Plus Chief Justice Marshall's failure to rely on a theory of protective jurisdiction "at a time when conditions might have presented more substantial justification strongly suggests its lack of constitutional merit."

Bush v. Lucas

Bush, an aerospace engineer employed by the federal government, sued his superior for damages, alleging, inter alia, that he had been demoted in retaliation for exercising his First Amendment rights. The Civil Service Commission's Appeals Review Board had previously restored him to his former position and awarded him back pay. Although assuming that the civil service remedy was "less than complete", the Court held that in this matter of "federal personnel policy" the "elaborate remedial system" constructed by Congress should not be "augmented by the creation of a new judicial remedy". This remedy was found to be "constitutionally adequate", even though it was not an "equally effective substitute" for the judicial remedy sought. Court assumes that a federal right has been violated and Congress has provided a less than complete remedy for the wrong but special factors counselling hesitation: federal personnel policy- think of the Standard oil case Contrast this with Carlson: here the court did not allow damages despite the fact that the alternative was not as strong. Furthermore, it is not clear which of the Bivens exceptions, 1) special circumstances and 2) alternative remedies is at play.

Hollingsworth v. Perry

CA voters enacted Prop 8, which confined marriage to heterosexual couples. Same-sex couples who wanted to get married filed suit in district court challenging Prop 8 and seeking to enjoin the governor and other state officials for enforcing it. State court held the initiative unconstitutional, and state officials refused to defend the law further. Ballot initiative proponents successfully moved to intervene. As a matter of statutory and state constitutional law, proponents of a ballot initiative had authority to defend the initiative's constitutionality when state officials do not do so. State law cannot transform a generalized grievance into a justiciable controversy by designating official proponents of a state ballot initiative to defend its constitutionality when state officials decline to do so. That a state thinks that private parties should have standing to seek relief for a generalized grievance cannot override Court's settled law to the contrary

Cannon v. University of Chicago

Cannon sued the University of Chicago in federal court after she was denied admission to the university's medical school. Cannon alleged gender discrimination under § 901(a) of Title IX of the Education Amendments of 1972, 20 U.S.C. § 1681 (Title IX), and sought declaratory, injunctive, and monetary relief. The district court dismissed the action on the basis that Title IX does not expressly authorize a private right of action. The court of appeals affirmed, and the United States Supreme Court granted certiorari. Court applies Cort to find an implied remedy in Title IX.

standing v. mootness (Laidlaw)

Careful reflection on the long-recognized exceptions to mootness," such as that allowing adjudication of issues capable of repetition yet evading review, revealed that "there are circumstances in which the prospect that a defendant will engage in (or resume) harmful conduct may be too speculative to support standing, but not too speculative to overcome mootness. Standing doctrine functions to ensure, among other things, that the scarce resources of the federal courts are devoted to those disputes in which the parties have a concrete stake. In contrast, by the time mootness is an issue, the case has been brought and litigated, often (as here) for years. To abandon the case at an advanced stage may prove more wasteful than frugal. Other reasons for treating mootness doctrine differently from standing: (i) an actual course of conduct, even if past, continues to frame litigation in a factual context and thereby focus judicial decision-making; (ii) the unlawful causation of a past injury deprives a defendant of any moral entitlement to freedom from judicial intervention; (iii) since a defendant who has caused wrongful conduct would otherwise remain free to repeat it, a judicial decision forbidding such conduct is not an advisory opinion in any objectionable sense; and (iv) there is an important public interest in protecting the legal system against manipulation by parties, especially those prone to involvement in repeat litigation, who might contrive to moot cases that otherwise would be likely to produce unfavorable precedents.

Limiting Merrill

Central Bank of Denver v. First Interstate Bank of Denver, 511 U.S. 164 (1994), the Court (5-4) refused to imply a right of action for aiding and abetting a violation of § 10(b) of the Securities Exchange Act. In some tension with Merrill Lynch, the Court rejected the contention that Congress had acquiesced in the courts of appeals' recognition of aiding and abetting liability under § 10(b) of the Securities Exchange Act, even though Congress had left those interpretations untouched while otherwise amending the Act on several occasions. "Congress' failure to overturn a statutory precedent" cannot be equated with " 'affirmative congressional approval of the [courts'] statutory interpretation' " Most importantly, as seen below in Sandoval, the court held that courts should interpret a statute of any vintage according to current principles governing the implication of private rights of action, rather than reverting to those that prevailed when the statute was enacted.

The most prominent cases implicating "special factors" involved claims against military officers -> alternative system

Chappell v. Wallace (1983), Navy enlisted men sued their superior officers, alleging racial discrimination in violation of the Constitution and federal civil rights legislation. The Court unanimously held that the constitutional claim could not be maintained, stating that "the unique disciplinary structure of the Military Establishment and Congress' activity in the field constitute 'special factors' which dictate that it would be inappropriate to provide enlisted military personnel a Bivens-type remedy against their superior officers United States v. Stanley (1987), a former servicemember sued military officers and civilians for injuries resulting from the administration to him of the drug LSD, without his consent, as part of an army experiment. The Court held that the special factors found in Chappell "extend beyond the situation in which an officer-subordinate relationship exists, and require abstention in the inferring of Bivens actions for injuries that 'arise out of or are in the course of activity incident to [military] service

Private rights Law Questions

Chevron v. Natural Resources Defense Council elaborates a two-step process to be followed by courts reviewing agency interpretations: 1. First, always, is the question whether Congress has directly spoken to the precise question at issue. If the intent of Congress is clear, that is the end of the matter. 2. If, however, the court determines Congress has not directly addressed the precise question at issue, the court does not simply impose its own construction on the statute, as would be necessary in the absence of an administrative interpretation. Rather, if the statute is silent or ambiguous with respect to the specific issue, the question for the court is whether the agency's answer is based on a permissible construction of the statute." Doesn't this highly deferential approach cut against the role of the federal judiciary as outlined in Marbury? Well they still have a role in applying the Marbury decision.

Inadequate training/ supervision screening

City of Canton v. Harris (1989): a complaint that the due process rights of a person under arrest had been violated because he had been given inadequate medical attention by the police, the Court held that municipal liability for inadequate training is permitted by § 1983, but "only where the failure to train amounts to deliberate indifference to the rights of persons with whom the police (the officials involved in the particular case) come into contact". Board of County Commissioners v. Brown (1997): In Brown, the plaintiff brought a § 1983 damages action against the county, alleging that a deputy had arrested her with excessive force and that the county was liable for her injuries because the sheriff—a policymaking official—had hired the deputy without adequate review of his background. In the absence of a claim that action by a policymaking official directly violated federal law or directed or authorized the deprivation of federal rights, the plaintiff must establish deliberate indifference on the part of the policymaking representative of the municipality, not merely to the risk of any constitutional injury, but to the risk of the particular injury suffered by the plaintiff.

What makes someone a policy making official?

City of St. Louis v. Praprotnik (1988): the Court confronted the question of which officials' decisions can render a municipality liable under § 1983. Praprotnik, a municipal employee, brought suit contending that the Director of Urban Design (UD) (to whom plaintiff reported) in the St. Louis Community Development Agency (CDA), and CDA's Director, had violated the First Amendment by discharging plaintiff in retaliation for earlier appeals to the city's Civil Service Commission. State law determines who is a policymaking official Here, the court found that under state and local regulations, only the mayor and aldermen of St. Louis, and the Civil Service Commission, had policymaking authority over personnel decisions; and concluded that no policymaker had adopted an unconstitutional municipal policy authorizing retaliatory discharges.

Standard of reviews

Civil Nature of Proceedings and Applicable Rules. Habeas corpus actions are civil proceedings. Since 1977, they have been subject to the "Rules Governing Section 2254 Cases in the United States District Courts" The Federal Rules of Civil Procedure, to the extent that they are not inconsistent with any statutory provisions or these rules, may be applied to a proceeding under these rules Usually applies a preponderance of the evidence standard except 2254 (e)(1) which incorporates a clear and convincing evidence standard. Law - 2254(d)(1) Fact - 2254(d)(2), (e) (1)

Testa v. Katt (con.)

Claflin v. Houseman: Its teaching is that the Constitution and the laws passed pursuant to it are the supreme laws of the land, binding alike upon states, courts, and the people, "anything in the Constitution or Laws of any State to the contrary notwithstanding." It cannot be assumed, the supremacy clause considered, that the responsibilities of a state to enforce the laws of a sister state are identical with its responsibilities to enforce federal laws. Such an assumption represents an erroneous evaluation of the statutes of Congress and the prior decisions of this Court in their historic setting. So here, the fact that Rhode Island has an established policy against enforcement by its courts of statutes of other states and the United States which it deems penal cannot be accepted as a "valid excuse." Creation of state courts of general jurisdiction is consent to participate in system of dual sovereignty.

Hans v. Louisiana Breakdown

Claim was brought under federal question jurisdiction, specifically under the Contracts clause: Contracts clause: Article I, Section 10, Clause 1: No State shall enter into any Treaty, Alliance, or Confederation; grant Letters of Marque and Reprisal; coin Money; emit Bills of Credit; make any Thing but gold and silver Coin a Tender in Payment of Debts; pass any Bill of Attainder, ex post facto Law, or Law impairing the Obligation of Contracts, or grant any Title of Nobility. The Court does not doubt that the state is legally bound by the Contract Clause or other provisions of the Constitution, but it holds that the Eleventh Amendment bars the enforcement of the Constitution in a suit against the state. Justice Bradley- gives recap of the history of Chisolm - defenders of the Constitution protecting against the potential of suits against the government Hamilton: Revolutionary war debts- will people sue the states and Hamilton says no "It is inherent in the nature of sovereignty not to be amenable to the suit of an individual without its consent. Court essentially says it does not want to make the same mistake as Chisolm: It seems to us that these views of those great advocates and defenders of the Constitution were most sensible and just; and they apply equally to the present case as to that then under discussion. The letter is appealed to now, as it was then, as a ground for sustaining a suit brought by an individual against a State. Focus is not so much the exact words - but the spirit/purpose of the Amendment. The Hans court says that the 11th amendment vindicates the Iredell decision and that means that we should take that as the opinion of the court Clark: argues that, contrary to the Supreme Court's assumption in Hans, the narrow language of the Eleventh Amendment made perfect sense in the historical context in which it was written. Undoubtedly a State may be sued by its own consent

Teague What is New Law

Conceding the difficulty of determining whether a case announces a new rule, she suggested as the measure whether "it breaks new ground or imposes a new obligation on the States or the Federal Government. To put it differently, a case announces a new rule if the result was not dictated by precedent existing at the time the defendant's conviction became final." Subsequent decisions gave a wide definition to the concept of new law, thereby sharply contracting the scope of federal postconviction review.

Pennsylvania v. Union Gas (1989)

Congress can abrogate state immunity from federal court suit in the exercise of its power under the Commerce Clause—and that it had done so in the environmental statutes there at issue. Justice Brennan, who wrote for the plurality, argued that the states had ceded their sovereignty to Congress insofar as the Constitution vests Congress with power to regulate the states- > Is legislative power complete when you can outline conduct but cannot enforce it against the state when they offend? However, as pointed out in Seminole tribe, the Court did not have a majority opinion and instead Justice White concurred while not going as broad as the plurality.

Congressional Power to Regulate Standing:

Congress can create statutory rights or entitlements, the alleged deprivation of which can confer standing, even if there would be no cognizable injury absent the statutory right. But Congressional conferral of standing is still subject to AIII requirements. Previously it was well known that Congress may create standing where none previously existed.

Gordon v. United States

Congress has often used non-Article III tribunals—which don't have a justiciability doctrine—to determine whether to pay claims. When Congress does use Article III courts, however, they have had to determine whether their judgment will be subject to congressional revision. Till 1855, US had sovereign immunity. Then created non-Art. III Court of Claims, which could hear non-tort claims for money. Eventually could render judgment, but still required appropriation from Congress. found that it could not entertain appeals from such decisions as they were subject to revision.

United States v. Coolidge

Coolidge and a codefendant were indicted in the circuit court for having forcibly rescued a vessel that had been captured as a prize. The indictment charged the offense as one committed upon the high seas. In his opinion ("riding circuit") upholding the indictment, Justice Story expressed the view that "all offences against the sovereignty, the public rights, the public justice, the public peace, the public trade and the public police of the United States, are crimes and offences against the United States." Accordingly, he concluded that traditional "public offenses" at common law, such as "treasons, and conspiracies to commit treason, embezzlement of the public records, bribery and resistance of the judicial process, riots and misdemeanors on the high seas, frauds and obstructions of the public laws of trade, and robbery and embezzlement of the mail of the United States, would be offences against the United States." In the Supreme Court, the Attorney General declined to argue the case, treating Hudson as having settled the matter. Several Justices, however, indicated a willingness at least to reconsider Hudson even though they did not. Nevertheless its holding generally rings true: there is no such thing as federal common law crimes - judges are not defining what it means to offend criminal law.

Lexmark (2014)

Court called continued validity of prudential standing into question. Whether P fell into "Zone of interests" protected by the Lanham Act. In a prior decision, Assoc. of Data Processing Serv. Orgs. (1970), the Court reoriented the standing inquiry in administrative cases away form the question of whether the plaintiff had suffered an invasion of a legal interest. After satisfying Art. III minimum, P had to assert that claim was within the "zone of interests that are protected or regulated by the statute or constitutional provision in question. Court described this test as a gloss on judicial review provisions of the APA. Court here tried to resolve the confusion of the zone of interests test by recharacterizing it as one of statutory interpretation, whether the claim filed by the plaintiff falls within the scope of the right of action established by the Act. - To come w/i the zone of interests in a suit for false advertising under the Lanham Act, a P must allege an injury to a commercial interest in reputation or sales. - A consumer who is hoodwinked into purchasing a disappointing product may well have an injury-in-fact cognizable under AIII, but he cannot invoke the protection of the Lanham Act

Clearfield Breakdown

Court seems to be embrace two component questions: 1. Do the federal courts have authority to make a federal common law principle? 2. If so, what should be the content of that principle?- possibly state law? Court rejects that here - we want something nationally uniform. In our choice of the applicable federal rule we have occasionally selected state law. But reasons which may make state law at times the appropriate federal rule are singularly inappropriate here.-> just saying that federal common law should govern doesn't give us the content of the principle. Court is free to choose the state law. If you choose a federal principle, then you may have horizontal uniformity but at the expense Applies their precedent: The National Exchange Bank case went no further than to hold that prompt notice of the discovery of the forgery was not a condition precedent to suit. It did not reach the question whether lack of prompt notice might be a defense. We think it may. If it is shown that the drawee on learning of the forgery did not give prompt notice of it and that damage resulted, recovery by the drawee is barred.

Massachusetts v. EPA (2007)

Court upheld the standing of state to challenge EPA's refusal to issue regulations governing greenhouse gas emissions Congress authorized this type of challenge to EPA actions, and conferred on parties the procedural right to challenge the EPA's denial of petition to promulgate emissions standards. When a litigant is vested with a procedural right, that litigant has standing if there is some possibility that the requested relief will prompt the injury-causing party to reconsider the decision that allegedly harmed the litigant. MA alleged special injury in its capacity as a landowner involving threatened loss of state-owned coastal property as a result of global warming traceable to greenhouse gas emissions. Special solicitude: congress has conferred a procedural right- MA is protecting its quasi- sovereign interests. Not clear how much mileage can we get from this if you are not the government.

Advisory Opinions: Hypothetical Jurisdiction

Court used to proceed to the merits under the presumption of jurisdiction and then come back to jurisdiction to cover all basis. See, e.g., Marbury v. Madison (going from merits back to jurisdiction). However this is an unlawful advisory opinion. Steel Co. v. Citizens for a Better Environment: several courts of appeals had found "it proper to proceed immediately to the merits question, despite jurisdictional objections, at least where (1) the merits question is more readily resolved, and (2) the prevailing party on the merits would be the same as the prevailing party were jurisdiction denied." Justice Scalia reasoned, however, that "[h]ypothetical jurisdiction produces nothing more than a hypothetical judgment—which comes to the same thing as an advisory opinion". Now all courts today follow with 1) do I have jurisdiction and 2) merits

Criminal cases and agency

Criminal cases are private-rights cases, despite involvement of gov. as a party, but there's no doubt that administrative agency may not directly impose criminal punishments. But can Article III court hearing a criminal case rely on an agency's determination? If an AIII court, in imposing criminal punishment, relies in whole or in part on an agency's determination of fact or law, that is a grey issue b/c: - Yakus: Congress could require a criminal enforcement court to give conclusive effect to the decisions of a prior judicial proceeding. - Crowell: in a civil action, federal court could defer to agency determinations of the facts.

Deference to state-court finding

Current § 2254(d) precludes habeas relief (won't grant habeas relief at all) unless the federal court finds that a state court decision was either (i) contrary to or an unreasonable application of clearly established law (subsection (d)(1), the provision at issue in Terry Williams), or (ii) "based on an unreasonable determination of the facts in light of the evidence presented in the State court proceeding" (subsection (d)(2))

What about extradition?

Currently, 18 U.S.C. § 3184 requires the government to file a complaint with a federal judge or magistrate judge, or a state judge of general jurisdiction, to effect an extradition. After a hearing and finding of probable cause of an extraditable crime, the judge certifies the finding to the Secretary of State, who then makes the ultimate decision whether to deliver the person to the country of extradition. Circuit Courts have upheld the act against Hayburn's Case objections, reasoning that judges in extradition proceedings act as extradition officers in their individual rather than judicial capacities. Lo Duca (2d Cir. 1996): court is not acting as a court, but as extradition officers. Court's judgment still could be viewed as AIII judgment b/c, arguably, if the court does not find sufficient evidence, then there will not be extradition. Judgment matters, but still must be an additional step

Michigan v. Long (1983

David Long was convicted for possession of marijuana found by Michigan police in the passenger compartment and trunk of his car. The police searched the passenger compartment because they suspected Long's vehicle contained weapons potentially dangerous to the officers. After a state appellate court affirmed the conviction, the Michigan Supreme Court reversed. The Michigan Supreme Court held that the search violated the Fourth Amendment and the Michigan Constitution. Michigan Supreme Court reverse citing some state cases and some federal cases not specifying - two separate constitutions say you cannot do it. Seems like mostly mentioning federal constitution but in passing mentions Michigan constitution. When a state court decision fairly appears to rest primarily on federal law, or to be interwoven with the federal law, and when the adequacy and independence of any possible state law ground is not clear from the face of the opinion, we will accept as the most reasonable explanation that the state court decided the case the way it did because it believed that federal law required it to do so. If a state court chooses merely to rely on federal precedents as it would on the precedents of all other jurisdictions, then it need only make clear by a plain statement in its judgment or opinion that the federal cases are being used only for the purpose of guidance, and do not themselves compel the result that the court has reached. If the state court decision indicates clearly and expressly that it is alternatively based on bona fide separate, adequate, and independent grounds, we, of course, will not undertake to review the decision.

Bivens Breakdown

Defendants argued that this is really a matter of state tort law: We think that respondents' thesis rests upon an unduly restrictive view of the Fourth Amendment's protection against unreasonable searches and seizures by federal agents, a view that has consistently been rejected by this Court. "where federally protected rights have been invaded, it has been the rule from the beginning that courts will be alert to adjust their remedies so as to grant the necessary relief." Of course the Fourth Amendment does not in so many words provide for its enforcement by an award of money damages for the consequences of its violation. But "it is also well settled that where legal rights have been invaded, and a federal statute provides for a general right to sue for such invasion, federal courts may use any available remedy to make good the wrong done. More Liberal view than current Statutory reading: we have here no explicit congressional declaration that persons injured by a federal officer's violation of the Fourth Amendment may not recover money damages from the agents. But there may be special factors counseling hesitation in the absence of affirmative action by Congress.

Defining Public Rights

Despite its historical lineage, the public rights category has never received a canonical formulation. 1. The first category is "claims against the United States" for "money, land or other things. 2. The second category is disputes arising from coercive governmental conduct outside of the criminal law. 3. The third category is immigration cases, "apparently because of the longstanding assumption that the executive and legislative branches possess plenary power over immigration issues" A decision like St. Cyr surely qualifies the broadest statements that public rights matters may always be decided with no judicial involvement. Nonetheless, there is little doubt that Congress generally has greater latitude to limit judicial involvement in the resolution of matters that involve public as compared to private rights.

Vaden v. Discover Bank

Discover Bank (through an affiliate) sued Vaden under state law in state court to recover pastdue charges on a credit card. Vaden counterclaimed that the charges and fees in question violated state law, though both parties later agreed, and the Supreme Court assumed, that the counterclaims were entirely based on federal law, because a provision of the Federal Deposit Insurance Act (FDIA) "completely preempted" any applicable state law and supplied the only possible basis for relief. At this point in the litigation, Discover filed a federal court action against Vaden under § 4 of the Federal Arbitration Act. The Court unanimously agreed to "look through" the dispute over arbitrability to see whether the court would have had subject matter jurisdiction over the underlying controversy between the parties and found that the "whole" controversy between the parties was one arising under state law.

28 U.S.C. § 1257: (a)

Final judgments or decrees rendered by the highest court of a State in which a decision could be had, may be reviewed by the Supreme Court by writ of certiorari where the validity of a treaty or statute of the United States is drawn in question or where the validity of a statute of any State is drawn in question on the ground of its being repugnant to the Constitution, treaties, or laws of the United States, or where any title, right, privilege, or immunity is specially set up or claimed under the Constitution or the treaties or statutes of, or any commission held or authority exercised under, the United States.

So after Harlow, qualified immunity requires a two step process: 1) did the conduct violate a constitutional right? 2) if so, was the right clear established. But what makes a law clearly established?

Do not need SCOTUS holding in order for it to be clearly established - United States v. Lanier (1997), a criminal case brought under 18 U.S.C. § 242, the Court stated that it was possible for a right to be "clearly established" even in the absence of a Supreme Court decision so holding, but that disparate decisions in the lower courts might preclude such a determination. A court of appeals precedent in the relevant circuit? This would appear to be enough: What about a consensus of persuasive authorities- ex state high courts and outside circuits when the relevant circuit has not said anything? The court has implied that this will work However, the Court has cited disagreement among lower court judges as its principal ground for upholding a qualified immunity defense Safford Unified School District #1 v. Redding (2009): the Court first held that the defendant school officials had violated the Fourth Amendment when they strip-searched a thirteen-year-old middle school student without adequate reason to suspect her of hiding dangerous contraband in her underwear. But the defendants possessed qualified immunity, the Court said, largely because lower courts had "reached divergent conclusions regarding how" the controlling precedent allowing school searches under a reasonableness standard.

United States v. Klein

During the Civil War, agents of the government, acting pursuant to wartime legislation, seized and sold a large quantity of cotton that belonged to a Southerner named Wilson. The legislation provided, however, that those whose property had been seized could recover the proceeds from its sale in the Court of Claims, if they could prove that they had "never given any aid or comfort to the present rebellion." 1863 presidential proclamation offers pardon who those who take an oath of allegiance. Wilson took the oath. After Wilson's death, Klein, the administrator of his estate, prevailed in an action in the Court of Claims, which awarded the considerable sum of $125,300. The Supreme Court had previously held that one who, like Wilson, had received a presidential pardon must be treated as loyal. While an appeal by the United States from that judgment was pending, Congress passed an act providing that no pardon should be admissible as proof of loyalty and, further, that acceptance of a pardon without a protest that the claimant took no part in the rebellion was conclusive evidence of the claimant's disloyalty. The statute directed the Court of Claims and the Supreme Court to dismiss "for want of jurisdiction" any claim based on a pardon. Court has a view about why Congress is doing this. Not just regarding this as an exercise of exception authority. Is it important why Congress is doing it? The language of the provisio shows plainly that it does not intend to withhold appellate jurisdiction except as a means to an end. Its great and controlling purpose is to deny to pardons granted by the president the effect which this court had adjudged them to have

Summers v. Earth Land Inst. (2009)

Environmental orgs. Sought to prevent US forest service from enforcing regulations that exempt certain projects from the notice, comment, and appeal process used by the Forest Service for more significant land management decisions. Ps could not base standing on the statistical probability that some of their members would suffer harms that intended to visit a site again.

Pape Breakdown

Essentially, the defendants argue that there is a remedy available under state law so there is no need for 1983 and also, because this action was in violation of state law, it cannot be said to be under the color of state law. However, the court rejects these arguments. Why? - Relying on state law was insufficient- Sometimes the laws on the book were a problem and were against Congress's intent while others were fine but not enforced-> It is no answer that the State has a law which if enforced would give relief. The federal remedy is supplementary to the state remedy, and the latter need not be first sought and refused before the federal one is invoked. - It is abundantly clear that one reason the legislation was passed was to afford a federal right in federal courts because, by reason of prejudice, passion, neglect, intolerance or otherwise, state laws might not be enforced and the claims of citizens to the enjoyment of rights, privileges, and immunities guaranteed by the Fourteenth Amendment might be denied by the state agencies. One who reads [the debates] in their entirety sees that the present section had three main aims. - First, it might, of course, override certain kinds of state laws. - Second, it provided a remedy where state law was inadequate. * * * - The third aim was to provide a federal remedy where the state remedy, though adequate in theory, was not available in practice. Furthermore, the court finds that they had interpreted a similar statute in the past in the same way

By a clear incompatability between state- court jurisdiction and federal interest

Factors indicating clear incompatability include 1. The desirability of uniform interpretation- different state means different application 2. The expertise of federal judges in federal law and 3. The assumed greater hospitality of federal courts to peculiarly federal claims One category seems to be areas where there may be different political receptions. For example civil rights cases might be met with less rigorous enforcement in certain state courts.

Historically you couldn't always challenge the factual basis of the charge (Falbo, Estep). But Mendoza-Lopez seems to provide at least some process right for a collateral attack.

Falbo v. US: failed to report for work of national importance. Didn't classify right. Court says he should have made that argument to the administrative framework- should have exhausted administrative hearing. Estep v. US: not supposed to be drafted. Appealed through the system and loses. Takes it to the appeal board and loses. Goes all the way up the executive branch- exhausted administrative remedy. Prosecuted for refusing. Can he defend saying he is an administer of religion? District court says no. He is convicted. SCOTUS says there are some challenges you can make and some you cannot. Can say on his defense that the board was outside of its jurisdiction. Needs to be extreme. Needs to have no basis in fact. But you cannot say as a factual matter they got it wrong it won't work. High bar to challenge administrative determination in criminal cases.

Qui tam Cases

False Claims Act authorizes private citizens, called relators, to bring qui tam actions on behalf of the US seeking civil penalties and damages payable to the Treasury against any person who procured payment on false claim against the US. The realtor then gets a portion of the proceeds. Vermont Agency of Natural Resources v. US ex rel. Stevens (2000): Relator has Art. III standing but not because relator has an interest in receiving the percentage payable for successful prosecution. Where an interest is merely the byproduct of the suit itself, does not satisfy the injury requirement. Relator, as assignee of the gov't claim, has standing to assert the injury in fact suffered by the assignor Court did not express a view on whether qui tam violates Take Care clause of Art. II § 3.

Banco Nacional De Cuba v. Sabbatino

Farr, Whitlock & Co. contracted to buy sugar from a Cuban corporation. The corporation loaded the sugar on to the S.S. Hornfels, but in response to President Eisenhower reducing the Cuban sugar quota, Cuba issued a decree taking possession of the sugar. The Cuban government would only allow the sugar to leave Cuba if Farr, Whitlock entered into a new contract with Banco Nacional de Cuba, an instrumentality of the Cuban government. NY corp. secured a Cuban export license for the sugar, and promised to pay proceeds to the petitioner, but then paid proceeds to Sabbatino, a receiver for the Cuban corp. that had sold the sugar. After the sugar left Cuba, Farr, Whitlock refused to pay Banco Nacional. Banco Nacional sued in the U.S. District Court for the Southern District of New York to recover payment. The court granted summary judgment for Far, Whitlock, holding that Cuba's taking of the sugar violated international law. Dist. Ct. found that the Cuban decree had not conveyed good title to the sugar to petitioner, and court of appeals affirmed. Court of Appeals below relied on "exception" to the act of state doctrine under Bernstein, where State Dept. letters regarding the seizure of property by Nazis against Jews in Europe, stating that courts could exercise jdx. to determine validity of acts of Nazi officials. An issue concerned with a basic choice regarding the competence and function of the Judiciary and the National Executive in ordering our relationships with other members of the international community must be treated exclusively as an aspect of federal law.

General v. Federal Common Law:

General law was very broad under Swift. Now, federal common law is more specialized to fill gaps or omissions in statutes State courts have an obligation to apply federal common law (did not for general law) Assuming that federal common law is part of "supreme federal law" (Supremacy Clause).

Miree v. DeKalb Cty

Federal Aviation Administration (FAA) to "restrict the use of land adjacent to . . . the Airport to activities and purposes compatible with normal airport operations including landing and takeoff of aircraft." At issue was whether the county breached its contractual obligation by operating a garbage dump adjacent to the airport, which allegedly attracted the swarm of birds that caused a plane crash. Federal common law would undoubtedly have controlled in any suit by the Federal Government to enforce the provision against the county or to collect damages for its violation. The diversity suit, however, was brought not by the Government, but by assorted private parties injured in some way by the accident. We observed that * * * "the United States has a substantial interest in regulating aircraft travel and promoting air travel safety." Nevertheless, we held that state law should govern the claim because "only the rights of private litigants are at issue here,"

Federal Question Jurisdiction

Federal Question jurisdiction has its basis in the Constitution Article III s2: The judicial power shall extend to all cases, in law and equity, arising under this Constitution, the laws of the United States, and treaties made, or which shall be made, under their authority;--to all cases affecting ambassadors, other public ministers and consuls;--to all cases of admiralty and maritime jurisdiction;--to controversies to which the United States shall be a party;--to controversies between two or more states;--between a state and citizens of another state;--between citizens of different states;--between citizens of the same state claiming lands under grants of different states, and between a state, or the citizens thereof, and foreign states, citizens or subjects. This is quite generous However, the federal question jurisdiction does not reach the extent of what the constitution allows. Therefore it is a two step analysis: Figure out what the statute does then ask if that is constitutional.

Advisory Opinions

Federal courts require (1) an actual dispute (adverse litigants); and (2) a substantial possibility that the decision will have a real-world impact.

Constitutional Avoidance

Felker v. Turpin, 518 U.S. 651 (1996), the Court avoided potential constitutional questions about congressional restriction of the Supreme Court's appellate jurisdiction by holding that a statute withdrawing the Court's certiorari jurisdiction in certain habeas corpus cases did not restrict the Court's authority to review the case before it by entertaining a petition for an original writ of habeas corpus. In this respect, Felker echoed the penultimate paragraph of the McCardle decision. But rather than merely mentioning in passing, as in McCardle, that alternative means of review existed, the Felker Court, and in particular the concurring Justices, placed far greater reliance on it. Implied repeals of SCOTUS appellate jurisdiction are disfavored. AEDPA does not withdraw SCOTUS authority to entertain original habeas petitions The Court, per Rehnquist, C.J., held unanimously that the Act's preclusion of certiorari review of the court of appeals' gatekeeping decisions did not offend Article III, § 2. But the Court reached that conclusion only after having interpreted § 2244(b) as not withdrawing the Court's authority to entertain original habeas petitions under 28 U.S.C. § 2241.

Habeas Corpus

First developed in English common law courts, the writ is a means of testing the lawfulness of detention. Historically its central role was to permit a court to oversee the legality of detention imposed extra-judicially by executive officials. If detention is deemed to be unlawful, a court exercising habeas corpus jurisdiction orders that the prisoner be discharged from confinement. Spans before the founding, based in English Law. Blackstone: Magna Carta only, in general terms, declared, that no man shall be imprisoned contrary to law: the habeas corpus act points him out effectual means, as well to release himself, though committed even by the king in council, as to punish all those who shall thus unconstitutionally misuse him. The constitution mentions the writ: U.S. Const. Art. I, § 9, cl. 2: The Privilege of the Writ of Habeas Corpus shall not be suspended, unless when in Cases of Rebellion or Invasion the public Safety may require it.

Stage to a Habeas Corpus proceeding Prelims

First requires there to be a traditional state criminal case- pretrial-> trial-> post verdict proceedings -> sentencing -> direct appeal-> state post conviction review. A prisoner seeking federal habeas relief then must satisfy two pre-conditions. - (1) Custody. Because habeas corpus is a remedy for unlawful custody, the prisoner must be in custody when the petition is filed Custody" includes not only physical detention but also being subject to parole or probation conditions. But a convict who has served the entire sentence (including parole or probation terms) before filing a habeas petition, or whose only penalty was a fine, is not in custody. - (2) Exhaustion of State Remedies. Before seeking habeas relief, a prisoner must exhaust state remedies, including direct appellate review in the state courts (but not Supreme Court review of the state court conviction) Note the interaction of the custody and exhaustion requirements: in cases involving short sentences, a prisoner may no longer be in custody by the time state remedies have been exhausted. Plus the defendant must get their writ in before the statute of limitations outlined above. get a lawyer Get a stay of execution

"In Aid of Its Jurisdiction"

First, most courts have viewed the language as confirming the "res" exception- the court that first asserts itself can proceed - usually cannot enjoin unless you are the first to get the thing and it is the grounds for jurisdiction then you can enjoin Can only use the in aid of jurisdiction exception when the federal court has jurisdiction - What about if there is exclusive federal court? Most lower courts have viewed Richman Bros refusal to authorize an injunction to protect NLRB's exclusive jurisdiction as applying equally to protection of federal courts' exclusive jurisdiction. Nevertheless, The existence of exclusive jurisdiction may be one relevant factor, however, in determining whether a statute constitutes an "express" exception within the meaning of Mitchum.

Cannon Factor 1

First, the threshold question under Cort is whether the statute was enacted for the benefit of a special class of which the plaintiff is a member. That question is answered by looking to the language of the statute itself. Language here: "No person in the United States shall, on the basis of sex, be excluded from participation in, be denied the benefits of, or be subjected to discrimination under any education program or activity receiving Federal financial assistance" The language in these statutes—which expressly identifies the class Congress intended to benefit—contrasts sharply with statutory language customarily found in criminal statutes, such as that construed in Cort, and other laws enacted for the protection of the general public.

Brown Frankfurter guiding principles

First. Just as in all other litigation, a prima facie case must be made out by the petitioner. The application should be dismissed when it fails to state a federal question, or fails to set forth facts which, if accepted at face value, would entitle the applicant to relief. Second. Failure to exhaust an available State remedy is an obvious ground for denying the application Third. If the record of the State proceedings is not filed, the judge is required to decide whether it is more desirable to call for the record or to hold a hearing. Unless a vital flaw be found in the process of ascertaining such facts in the State court, the District Judge may accept their determination in the State proceeding. So-called mixed questions or the application of constitutional principles to the facts as found leave the duty of adjudication with the federal judge.

Nixon v. Fitzgerald

Fitzgerald (the plaintiff in the Harlow case) sought damages from President Nixon, who allegedly shared responsibility for the loss of his federal job. Justice Powell's opinion for the Court ruled that the President enjoyed an absolute immunity from damages liability for all acts within the "outer perimeter" of his official responsibilities. The court thought this was a "functionally mandated incident of the President's unique office, rooted in the constitutional tradition of the separation of powers" he Court stressed that the President's prominence made the Chief Executive an easy target for damages actions, and that if Presidents had only a qualified immunity, the resulting diversion of their energies in defending themselves would jeopardize the effective functioning of government.

Boyle v. United Technologies Cor

Following the death of US Marine helicopter pilot David A. Boyle, Delbert Boyle sued the helicopter's manufacturer ("Sikorsky") for defectively designing its copilot emergency escape hatch. On appeal from a state-law based jury verdict favoring Boyle, the Court of Appeals found that Sikorsky could not be held liable under Virginia tort law for any design flaws since it met the requirements of the "military contractor defense." Boyle appealed; the Supreme Court granted certiorari. Extends beyond cases where the government is a direct party.

FTB Breakdown

For the second, it raised the new question of does Skelly Oil limit original federal court jurisdiction under § 1331 when a question of federal law appears on the face of a well-pleaded complaint for a state law declaratory judgment. What they want is colored in federal law- it is a DJ complaint about what federal law does and does not require, but under a state action. However, he suggested that they should apply its spirit which leads us to extend it to state declaratory judgment actions as well. What does that mean? It means the second-> federal courts have "regularly" assumed jurisdiction over declaratory judgment actions in which the declaratory defendant could have brought a coercive federal action against the declaratory plaintiff. ERISA gives trustees of plans like CLVT a right to sue for injunctive relief and even apart from that statutory authorization, CLVT might have been able to sue to enjoin application of allegedly preempted state law (see below) Nonetheless, the Court refused to exercise jurisdiction, creating a new exception to the rules for arising under jurisdiction set forth in Skelly Oil. Court looked at what ERISA does and doesn't do. Furthermore, the court finds federalism concerns- better suited for the states.

Legacy of Non-Literalism.

Foreign Government: 11A does not mention anything about a suit brought by a foreign government, only citizens of a foreign state. Nevertheless, in Monaco v. Mississippi (1934), that the effect of the Amendment is to bar suit against a state by a foreign country. Admiralty: Does not mention suits in admiralty and only says suits in law or equity, but In Ex parte New York (1921), the Court applied the Amendment to bar a suit in admiralty, despite the textual limitation to suits "in law or equity" Administrative Agencies: text only says judicial power of the United states, and agencies are arguably not within that power. However, in Fed. Maritime Comm'n v. S.C. State Ports Auth. The court extended the 11th Amendment to administrative agency proceedings State Courts: again text only says judicial power of the United states, but the Court barred an unconsented private action against a state brought on a federal claim in a state court, in Alden v. Maine.

Why should we stick just to the text

Formal argument: the text of the pre-emption clause is the only thing that Congress enacted. If it is constitutional this is the only thing that encompasses the Supremacy Clause. But maybe the law that underlies the purpose of the statute is also the federal law. Worried about purposivist interpretation - might be in tension with some purpose. If Congress can enact a statute we just stick with what they had to say on it. If Congress takes the trouble to enact an express preemption clause does the Court threaten to disrupt the balance that Congress Struck if it invokes implied preemption as well? Gobeille v. Liberty Mutual: Thomas: approvingly noting that in some cases involving express pre-emption provisions the statutory text has been the beginning and often the end of the analysis.

Cannon Factor 4+

Fourth, whether implying a federal remedy is inappropriate because the subject matter involves an area basically of concern to the States. Note that this is really a matter of advocacy and characterization- here they could have said this was about education, but the court emphasizes that this is a federal spending scheme. Finally, though, the court notes that it "far better" for Congress to so specify when it intends private litigants to have a cause of action

Brown v. Allen breakdown Frankfurter

Frankfurter was known to be protective of state courts and the role of the states but here says it's a separation of powers issue- up to congress. Very much cabins his decisions: Congress told us to do it and very few cases are meritorious so it won't matter. The prior State determination of a claim under the United States Constitution cannot foreclose consideration of such a claim, else the State court would have the final say which the Congress, by the Act of 1867, provided it should not have. If we are to give effect to the statute and at the same time avoid improper intrusion into the State criminal process by federal judges we must direct them to probe the federal question while drawing on available records of prior proceedings to guide them in doing so.

Military Courts

From 1789 to the present, Congress has authorized military tribunals, whose judges are commissioned officers not protected by life tenure, to conduct courts martial to enforce military discipline and punish service members. Their constitutionality was upheld in Dynes v. Hoover The justifications for establishing such a system outside of Article III have included: (i) the physical separation of civilian society from the military, the latter of which requires a tribunal staffed with military officers to permit a quick determination of guilt; (ii) a concern that the military system differs from civilian society and that civilian officials unfamiliar with or even antagonistic to military life might misapply the legal standards of the military justice system; and (iii) the view that the goals of civilian and military justice differ, as the latter serves to preserve good order and discipline in the military as much as to punish, deter, incapacitate and rehabilitate offenders.

Hans v. Louisiana

Hans (plaintiff) was a citizen of the state of Louisiana (defendant). He brought suit against the state in the Circuit Court of the United States for the Eastern District of Louisiana for interest accrued on bonds issued by the state. The suit alleged that an amendment to the state constitution that barred the state from paying the interest violated the United States Constitution. The circuit court dismissed the action for lack of jurisdiction, and Hans appealed to the United States Supreme Court. The court clearly rejects a textual reading of the 11A -> The State cannot be sued under the 11th Amendment by citizens of their own statement.

Preemption of Parallel State Law

Generally, federal law permits parallel or supplemental state law to co-exist. Sometimes federal law exists where state law also wants to. California v. ARC America Corp. (1989) the Court, without dissent, held that the federal antitrust laws—which the Court has interpreted as generally precluding suit by "indirect purchasers" of products whose price has been illegally fixed—do not preempt state antitrust laws that do permit suit by indirect purchasers. But sometimes federal regulation displaces parallel state law on the basis that Congress intended federal regulation to be exclusive. Sears, Roebuck & Co. v. Stiffel Co. + Compco Corp. v. Day-Brite Lighting, Inc.: the Court held that state unfair competition law could not proscribe the copying of products not entitled to federal patent protection: "[T]he patent system is one in which uniform federal standards are carefully used to promote invention while at the same time preserving free competition.

FCL Criminal Law

Generally: up to Congress to articulate criminal law. Court will not recognize or punish a criminal offense that Congress has not created-> Congress must 1) criminalize the conduct 2) set the punishment and 3) grant jurisdiction to the lower federal court

FEC v. Akins (1998)

Group of voters had standing to challenge a determination by the FEC that AIPAC was not a "political committee" within the meaning of the FEC Act of 1971, and thus not subject to the same disclosure requirements. The group of voters filed a complaint with the FEC arguing that AIPAC was a political committee. Respondents had standing to challenge the FEC decision. Congress has specifically provided in FECA that "[a]ny person who believes a violation of this Act ... has occurred, may file a complaint with the Commission. Congress provided for federal suit for any party aggrieved by an order of the FEC Informational injury here, directly related to voting, is sufficiently concrete and specific such that the fact that it was widely shared does not deprive Congress of the constitutional power to authorize its vindication in the courts. Acknowledging that abstractness often corresponds with an injury's being widely shared, their association is not invariable and where a harm is concrete, though widely shared, it is an injury in fact. Comparison to Richardson- rested on a reading of the accounts clause where the taxpayers have no logical nexus to state of government spending.

State Waivers of Sovereign Immunity.

Hans did not purport to question the rule that a state, like any sovereign entity, may waive its immunity and consent to suit. Is this an anomaly (however well-established) in light of the ordinary rule that the parties lack power to confer subject matter jurisdiction on the federal courts? IS this jurisdictional or not? The Court has taken a range of positions on this issue: - Edelman v. Jordan (1974): the matter is jurisdictional, - Patsy v. Board of Regents (1982), that an Eleventh Amendment question is not jurisdictional "in the sense that it must be raised and decided by this Court on its own motion". - In Wisconsin Department of Corrections v. Schacht (1998) the Court said that the issue has yet to be resolved. What about limiting the scope of express waivers? A state may waive sovereign immunity as to suits for tax refunds in its own courts, while retaining its Eleventh Amendment immunity from such lawsuits in federal court. (Smith v. Reeves)

Retroactivity Harlan

Harlan strongly criticized the Warren Court's approach to retroactivity. In his view, cases still subject to adjudication or on appeal differed importantly from cases on habeas review. On direct review, he argued, courts must apply all decisions retroactively; to do otherwise would suggest that their function was not one of adjudication but of legislation. But because habeas corpus is an extraordinary remedy, no such judicial obligation exists, and the state's interest in finality called for a narrower judicial inquiry. Courts are declaring law and if it is law it has to be applied to everyone similarly situated. Doing otherwise is legislation. In Griffith v. Kentucky (1987), a divided Supreme Court endorsed Justice Harlan's view that new rules should be fully retroactive on direct review. Justice Blackmun wrote for the Court that the "failure to apply a newly declared constitutional rule to criminal cases pending on direct review violates basic norms of constitutional adjudication.

Seminole Tribe Has Congress Attemped to Abrogate

Has Congress attempted to abrogate state sovereign immunity? The court has applied the Clear statement requirement- when the court is looking at this question it is not just reading it like any old statute but demanding more of the statute requiring a clear statement abrogating.-> whether Congress has "unequivocally expressed its intent to abrogate the immunity" The court finds the standard met here: the statute specifically mentions that states will be subject to suits Why require a clear statement before abrogating? 1) Congressional Accountability: Prevent congressional overreach by relying on courts to abrogate by using vague. 2) Federalism: Court has view of state-national government balance and don't think abrogation fits well- need to be clear because we don't think you do it too much 3) State advocacy in the legislative process: To the extent we believe states lobby for their rights during the bill making process, the clear statement acts as a tip off so the state is aware

Sheldon v. Sill

Hastings, a citizen of Michigan, assigned a bond and mortgage to Sill (plaintiff), a citizen of New York. Sill then sued Sheldon (defendant), a citizen of Michigan, to recover on the bond and mortgage. Sill brought the action in the United States Circuit Court for the District of Michigan based on diversity jurisdiction. Sheldon argued that diversity jurisdiction did not apply, because diversity existed only as a result of an assignment of rights to Sill. The eleventh section of the Judiciary Act, which defines the jurisdiction of the Circuit Courts, restrains them from taking "cognizance of any suit to recover the contents of any promissory note or other chose in action, in favor of an assignee, unless a suit might have been prosecuted in such court to recover the contents, if no assignment had been made, except in cases of foreign bills of exchange." The circuit court upheld jurisdiction, and an appeal was taken directly to the United States Supreme Court to determine whether jurisdiction in the circuit court was proper. Congress does not need to legislate to the constitutional limits. Cannot be in conflict with the Constitution unless it confers a power not enumerated.-> cannot go too far and give too much jurisdiction.

Ineffective assistance of counsel

Have to show cause under Strickland v. Washington - standard of performance. We also need to delineate when there is a right to counsel as to make it ineffective- Sixth Amendment right to counsel during:-Pretrial-Trial-Sentencing-First appeal as of right But generally no such Sixth Amendment right as to: Subsequent discretionary appeals or State postconviction review Sometimes you really won't have an opportunity to raise the ineffective counsel claim until collateral review- trial attorney isn't going to go to the court and say I was terrible and unlikely on appeals as well. Murray v. Carrier (1986), appellate counsel inadvertently failed to include in the brief on appeal one claim that was listed in the notice of appeal. In an opinion for five Justices, Justice O'Connor held that this state court procedural default barred federal habeas review of the claim. The Court held that a defendant who is "represented by counsel whose performance is not constitutionally ineffective under the standard established in Strickland v. Washington, [466 U.S. 668 (1984),]" bears "the risk of attorney error that results in a procedural default."

Capital Cases and Herrera

We may assume, for the sake of argument that in a capital case a truly persuasive demonstration of 'actual innocence' "would warrant habeas relief "if there were no state avenue open to process such a claim. But the threshold would need to be extraordinarily high. SCOTUS has not defined what this threshold would look like. But lower courts have worked through it.

Counterclaims and the Well-Pleaded Complaint Rule

Holmes Group, Inc. v. Vornado Air Circulation Systems, Inc. (2002): the Court held without dissent that a federal counterclaim, even when compulsory, does not establish "arising under" jurisdiction. Justice Scalia's opinion for the Court reasoned that a contrary rule would (i) permit the defendant to defeat the plaintiff's forum choice by raising a federal counterclaim, (ii) "radically expand the class of removable cases" and thereby fail to respect "the rightful independence of state governments", and (iii) undermine administrative simplicity by making jurisdictional determinations depend on the content not only of the complaint but also of responsive pleadings.

prosecutorial immunity administrative tasks limit

However, even if it is an administrative task, if it is tied to what happens in the court then it will also trigger this Van de Kamp v. Goldstein (2009), which held that absolute prosecutorial immunity barred claims based on the prosecution's non-disclosure of impeachment material, which was allegedly due to the defendants' failure to (1) train and supervise prosecutors and (2) establish an information system containing potential impeachment material concerning informants. Although the alleged constitutional violations involved "administrative activities", absolute prosecutorial immunity applied, Justice Breyer reasoned for a unanimous Court, because "[t]he management tasks at issue * * * concern how and when to make impeachment information available at trial" and "are thereby directly connected with the prosecutor's basic trial advocacy responsibilities".

United States v. Hudson & Goodwin

Hudson and Goodwin made a written accusation in a Connecticut publication that the President and Congress of the United States had secretly arranged a payment of two million dollars to Napoleon Bonaparte in return for permission to enter into a treaty with Spain. The United States brought an indictment for libel, a common law crime, against Hudson and Goodwin in the Circuit Court for the District of Connecticut. Hudson and Goodwin argued that the federal courts had no jurisdiction over common law crimes. The legislative authority of the Union must first make an act a crime, affix a punishment to it, and declare the Court that shall have jurisdiction of the offence.

SCOTUS invoking executive

If the state court is really recalcitrant then you need to get the federal executive involve- In one instance, state executive officials have been punished for contempt of a Supreme Court order. United States v. Shipp the Court had ordered a stay of execution of an African-American man who had been convicted of rape in Tennessee, pending an appeal, which the Court had allowed, from a federal circuit court's denial of the prisoner's habeas corpus petition. A group of men (including a state sheriff with custody of the prisoner), with knowledge of the Supreme Court's order, lynched the prisoner. After the Attorney General of the United States filed an information charging contempt of the Supreme Court, the Court appointed a commissioner to take testimony, rendered judgments of conviction, and sentenced the defendants to prison.

Louisville & Nashville R.R. Co. v. Mottley

In 1871, the Mottleys (plaintiffs) were injured in a railway accident. The railroad, Louisville & Nashville Railroad (defendant), settled the Mottleys' claims with a lifetime pass for free transportation on its line. In 1906, Congress passed an act forbidding passes granting free transportation. In 1907, the railroad refused to renew the Mottleys' passes. The Mottleys brought suit in federal district court, seeking specific performance of their settlement agreement with the railroad. The Mottleys alleged that the act did not apply to their free pass and that, if the law is construed as prohibiting such passes, it deprives them of their property in violation of the Fifth Amendment. The railroad demurred to the Mottley's complaint. The trial court overruled the demurrer and entered judgment in favor of the Mottleys. ***When analyzing arising under jurisdiction for purposes of section 1331 look only to the plaintiff's well-pleaded complaint and not to a defense or counterclaim. It is not enough that the plaintiff alleges some anticipated defense to his cause of action and asserts that the defense is invalidated by some provision of the Constitution * * *. Although such allegations show that very likely, in the course of the litigation, a question under the Constitution would arise, they do not show that the suit, that is, the plaintiff's original cause of action, arises under the Constitution. "a suggestion of one party, that the other will or may set up a claim under the Constitution or laws of the United States, does not make the suit one arising under that Constitution or those laws.

Fairchild v. Hughes

In 1920, Charles S. Fairchild (plaintiff), a member of the political action group The Constitutional League brought suit against the Secretary of State and Attorney General of the United States (defendants) in the Supreme Court of the District of Columbia to challenge the constitutionality of the Nineteenth Amendment to the United States Constitution. The suit sought an injunction to prevent the Secretary of State from ratifying the amendment and to prevent the Attorney General from enforcing it. The suit alleged that the proposed Suffrage Amendment was unconstitutional because it would force states to allow women to vote even if the state constitution limited voting rights to men. Plaintiff has only the right, possessed by every citizen, to require that the government be administered according to law . . . Obviously this general right does not entitle a private citizen to institute in the federal courts a suit to secure by indirection a determination whether a statute, if passed, or a constitutional amendment about to be adopted will be valid.

Montgomery v. Louisiana

In 1963, Henry Montgomery was found guilty and received the death penalty for the murder of Charles Hunt, which Montgomery committed less than two weeks after he turned 17. He appealed to the Louisiana Supreme Court, and his conviction was overturned because of community prejudice. At his new trial, Montgomery was again convicted, but he was sentenced to life without parole. In 2012, the U.S. Supreme Court decided Miller v. Alabama, in which the Court held that mandatory sentencing schemes requiring children convicted of homicide to be sentenced to life imprisonment without parole violate the Eighth Amendment. In light of that decision, Montgomery filed a motion in state district court to correct what he argued was now an illegal sentence. The trial court denied Montgomery's motion, and the Louisiana Supreme Court denied Montgomery's application by holding that the decision in Miller does not apply retroactively. B/c of the Supremacy Clause, when a new substantive rule of constitutional law controls the outcome of a case, the Constitution requires state collateral review courts to give retroactive effect to that rule-> State court cannot refuse to apply on state collateral review a new rule of federal constitutional law that would be applied retroactively in federal habeas Thomas- state courts might be concerned, but they can avoid that problem by refusing to hear claims all together. But what about Testa?

Bivens v. Six Unknown Named Agents of Federal Bureau of Narcotics

In 1965, six agents of the Federal Bureau of Narcotics forced their way into Webster Bivens' home without a warrant and searched the premises. The agents handcuffed Bivens in front of his wife and children and arrested him on narcotics charges. Later, the agents interrogated Bivens and subjected him to a visual strip search. Bivens sued the agents for $15,000 in damages each for humiliation and mental suffering. The district court dismissed the complaint for failure to state a cause of action. The U.S. Court of Appeals for the Second Circuit affirmed. violation of [the Fourth Amendment] by a federal agent acting under color of his authority gives rise to a cause of action for damages consequent upon his unconstitutional conduct. Bivens has stated a claim for damages under the Fourth Amendment

*Grable & Sons Metal Products, Inc. v. Darue Engineering and Manufacturing (2005)

In 1994, the Internal Revenue Service (IRS) seized Michigan real property belonging to Grable & Sons Metal Products, Inc. to satisfy federal back taxes Grable owed to the IRS. After giving Grable the statutorily guaranteed notice of the upcoming sale via certified mail, the IRS sold the property to Darue Engineering and Manufacturing. Five years after the sale, Grable filed a motion to quiet title in state court, alleging that he did not receive proper notice of the sale because he was not personally served in the manner required by 26 U.S.C. §6335, but rather received notice via certified mail. Darue removed the action to federal court, arguing that the case presented a federal question, because it required interpretation of a federal tax law. The district court found that the federal jurisdiction was appropriate: Sets out the test above- Federal question jurisdiction depends on 1) Necessarily raised (look at the elements of the claim); 2) Actually disputed; 3) Substantial (Smith); 4) Capable of resolution in federal court without disrupting the federal state balance (Merrell Dow)

Prisoner Litigation:

In 1996, Congress responded to concern about mounting numbers of prisoner civil rights actions, many of which were frivolous, by enacting the Prisoner Litigation Reform Act (PLRA), 110 Stat. 1321-66 to 1321-77. The PLRA imposes restrictions on prisoner suits that do not apply to nonprisoner § 1983 actions. Among the innovations are: 1) A requirement that plaintiffs, for whom filing fees had previously been waived, pay the customary filing fee (if necessary, in installments from the small prison financial account that they would otherwise use to pay for sundries). 2) A requirement that before filing suits challenging prison conditions, plaintiffs must first exhaust administrative remedies. 3) A provision addressed to "frequent filers" that prohibits a prisoner who has brought three or more previous actions that were dismissed as frivolous or malicious or for failure to state a claim from filing additional actions unless in imminent danger of serious physical injury. 4) A prohibition on the award of damages for mental and emotional distress unaccompanied by physical injury. 5) Limits on the permissible scope of consent decrees and injunctive orders, designed to keep judicial intrusion into prison management to the minimum required by the Constitution. 6) Special limits on attorney's fees.

Cort v. Ash, 422 U.S. 66 (1975)

In Cort, the Court refused to infer—from a provision of the Federal Election Campaign Act making it a crime for a corporation to make certain campaign contributions—a shareholder's right to bring a derivative action against corporate directors alleged to have violated the criminal prohibition. Established that the fact that a federal statute has been violated and some person harmed does not automatically give rise to a private cause of action in favor of that person. The decision was especially noteworthy for its effort, through its four-part test: 1) First, the threshold question under Cort is whether the statute was enacted for the benefit of a special class of which the plaintiff is a member. 2) Second, the Cort analysis requires consideration of legislative history. 3) Third, under Cort, a private remedy should not be implied if it would frustrate the underlying purpose of the legislative scheme. 4) Fourth, the final inquiry suggested by Cort is whether implying a federal remedy is inappropriate because the subject matter involves an area basically of concern to the States.- This test is not longer good law after Sandoval, but some courts apply it as a means of applying Sandoval- it is circuit dependent. Though stricter than Borak, Cort stated (in language on which Cannon relied) that when "federal law has granted a class of persons certain rights, it is not necessary to show an intention to create a private cause of action, although an explicit purpose to deny such cause of action would be controlling."

Cardinale v. Louisiana

In a trial by the State of Louisiana (plaintiff) against Cardinale (defendant) for murder, Cardinale's voluntary confession was introduced in its entirety, which was required by a Louisiana statute. La. Rev. Stat. § 15:450. Cardinale was convicted and sentenced to death. During his trial and subsequent proceedings in the Louisiana courts, Cardinale never challenged the admission of his confession in its entirety. Eventually, Cardinale petitioned the United States Supreme Court for certiorari, contending that the Louisiana statute was unconstitutional and that certain portions of his confession should have been redacted on the grounds of irrelevance and prejudice. It was very early established that the Court will not decide federal constitutional issues raised here for the first time on review of state court decisions. In view of the petitioner's admitted failure to raise the issue he presents here in any way below, the failure of the state court to pass on this issue, the desirability of giving the State the first opportunity to apply its statute on an adequate record, and the fact that a federal habeas remedy may remain if no state procedure for raising the issue is available to petitioner, the writ is dismissed for want of jurisdiction.

The Meaning of "Under Color of" Law

In equating action "under color of" state law with the state action requirement, Monroe establishes that § 1983 remedies are available even in cases in which a state official violates state law. -> so long as the act is done while cloaked with gov authority If claims he was not acting under the color look at factors: - On or off duty - Clothing/ badge - Use of weapon/ car provided by gov - Purpose of action - Statements asserting gov authority? Possible ways to show action under color include for a private person Conspiracy with government official to do the act State delegated authority State delegated public function Government in the actors management control **Yet the fundamental question to ask it: Was the government involved enough? Did the provide a mantle of authority that enhanced D's power?

Expressly Authorized

In order to be expressly authorized the statute does not need to explicit in mentioning 2283 nor expressly authorize injunctions of state-court decisions. Instead what we look for is that the legislation 1) create a specific and uniquely federal right or remedy 2) enforceable in a federal court of equity 3) that could be frustrated

Heyburn's Case

In the Invalid Pensions Act of 1792, Congress instructed circuit courts to review claims for pensions by Revolutionary War veterans. Circuit Court determinations would be submitted to the Secretary of War for review, who would then recommend to Congress what individuals would be put on the pension list. In the Circuit Court of the district of Pennsylvania, Justices James Wilson and John Blair and district judge Richard Peters refused to consider William Hayburn's pension claim. In a letter to President Washington, the jurists explained that the judiciary was a distinct and independent branch of government and that Congress could not assign duties to the courts that were not judicial in nature. Wilson, Blair and Peters also pointed to the fact that decisions under the Act would be reviewed by the legislative and executive branches, which they deemed "radically inconsistent" with judicial independence under the Constitution.

What concerns arise if Congress assigns adjudicative tasks to a non-Article III federal decisionmaker.

Individual interests: Due process in having a neutral decision maker who is separated from salary concerns if they decide against Congress's wishes. - Constitution does not require the lower courts often they will be heard by State Court judges who do not have life tenure and salary protection when the state judges are in the same boat. But State court judges can be insulated from the political process - appointed by gov + they are insulated from other political branches - Congress cannot just remove them. Systemic interest: Undermines the legitimacy of Article III judges. Their job to say what the law is?

Congressional Limits

Internal restraints: a statute defining the jurisdiction of the lower federal courts "cannot be in conflict with the Constitution, unless it confers powers not enumerated therein." That statement presumably refers only to a conflict with Article III External restrictions: imposed by other constitutional provisions- e.g. EPC

Banco Breakdown

It is not compelled by the constitution. Nevertheless, the act of state doctrine does, however, have "constitutional" underpinnings. It arises out of the basic relationships between branches of government in a system of separation of powers. -> not pointing to a particular statute or anything. The problems involved here are uniquely federal in nature: foreign affairs is unique to federal gov. Application of state law would frustrate uniformity needed in the U.S.'s relations with other countries. The act of state doctrine in its traditional formulation precludes the courts of this country from inquiring into the validity of the public acts a recognized foreign sovereign power committed within its own territory. However, the court does not full embrace the principle: It should be apparent that the greater the degree of codification or consensus concerning a particular area of international law, the more appropriate it is for the judiciary to render decisions regarding it, since the courts can then focus on the application of an agreed principle to circumstances of fact rather than on the sensitive task of establishing a principle not inconsistent with the national interest or with international justice. Therefore, rather than laying down or reaffirming an inflexible and all-encompassing rule in this case, we decide only that the Judicial Branch will not examine the validity of a taking of property within its own territory by a foreign sovereign government, extant and recognized by this country at the time of suit, in the absence of a treaty or other unambiguous agreement regarding controlling legal principles, even if the complaint alleges that the taking violates customary international law.

Younger v. Harris

John Harris, Jr. was a member of a labor party that promoted a type of socialism, called syndicalism, to replace capitalism in the American economy. Harris was indicted in state court and charged with violating California Penal Code § 11400 and § 11401, known as the California Criminal Syndicalism Act. Harris then filed a complaint in federal district court seeking an injunction to prohibit the District Attorney of Los Angeles County, California, Younger, from prosecuting Harris. Harris alleged the Act violated his rights of free speech and the press as guaranteed by the First and Fourteenth Amendments of the U.S. Constitution. The district court held that it had jurisdiction over the claim and the power to enjoin Younger from prosecuting Harris. Generally, a federal court cannot enjoin a state criminal proceeding once it has been commenced.

Some have suggested relief should be targeted to those who are innocent

Judge Friendly contended that, subject to limited exceptions, a petitioner who received a fair hearing in state court should not be able collaterally to attack a criminal conviction without making "a colorable showing that an error, whether 'constitutional' or not, may be producing the continued punishment of an innocent"-> Justice Powell agreed with this as well noting it in a concurrence in Schneckloth v. Bustamonte Stone v. Powell (1976) the Court held that a state prisoner generally cannot obtain federal habeas corpus relief on the ground that evidence obtained in an unconstitutional search or seizure was introduced at his trial. Relying on the premise that the exclusionary rule is not a personal constitutional right of the defendant but rather a judicially created remedy designed to safeguard Fourth Amendment rights by deterring police misconduct, Justice Powell argued that enforcing the rule on habeas imposed serious costs while providing little incremental deterrence of Fourth Amendment violations.

Judicial Immunity.

Judges have absolute immunity. The considerations offered by the Court as justifications for absolute judicial immunity have been summarized as follows: (1) the need for a judge to 'be free to act upon his own conviction, without apprehension of personal consequences to himself'; (2) the controversiality and importance of the competing interests adjudicated by judges and the likelihood that the loser, feeling aggrieved, would wish to retaliate; (3) the record-keeping to which self-protective judges would be driven in the absence of immunity; (4) the availability of alternative remedies, such as appeal and impeachment, for judicial wrongdoing; and (5) the ease with which bad faith can be alleged and made the basis for 'vexatious litigation.

Jurisdiction in Bankruptcy Proceedings

Jurisdiction in Bankruptcy Proceedings. The Supreme Court has repeatedly upheld the statutory grant of federal court jurisdiction over bankruptcy proceedings. In such proceedings, much of the law is federal. But many bankruptcy proceedings include disputes between the representative of the bankrupt's estate (formerly called the assignee in bankruptcy, now called the trustee) and a third party, and such disputes may turn entirely on state tort, contract, or property law. Note three distinct rationales on the basis of which broad bankruptcy jurisdiction might be upheld as constitutionally valid. 1) Supplemental Jurisdiction. 2) Federal Ingredient. 3) Protective Jurisdiction.

Staub Dissent

Justice Frankfurter is surely correct that, in general, state rules of practice presumptively determine the time when, and the mode by which, federal claims must be asserted in the state courts. Thus, ordinarily when a state court litigant has committed a procedural default—that is, has failed to raise a federal question in accordance with state procedural rules—the state court will refuse to decide the federal question, and any effort to obtain Supreme Court review will be rejected on the basis that there is an adequate and independent state procedural ground precluding the exercise of jurisdiction.

The Range of Remedies. Should the appropriateness of recognizing an implied right of action depend on just what remedy is sought?

Justice Powell, in Cannon, distinguished Allen v. State Board of Elections (1969)—in which the Court had permitted private litigants to seek a declaratory judgment that certain legislation had to be submitted for preclearance under § 5 of the Voting Rights Act of 1965—on the ground that "the remedy implied was very limited, thereby reducing the chances that States would be exposed to frivolous or harassing suits." Transamerica Mortgage Advisors, Inc. (TAMA) v. Lewis (1979) involved § 206 of the Investment Advisers Act of 1940, which makes it unlawful for an investment adviser to "employ any device, scheme, or artifice to defraud", as well as § 215, which renders "void" any contract made in violation of the Act. Treating the remedial issue as one of legislative intent, the Court construed § 215 as implying a private remedy for rescission of the void contract and restitution of money paid, but refused (over four dissents) to recognize a right of action for damages under § 206 Franklin v. Gwinnett County Pub. Schools (1992), all nine Justices agreed that when a private right of action exists, "[t]he general rule is that absent clear direction to the contrary by Congress, the federal courts have the power to award any appropriate relief". Thus, the Court held that in a private action under the same statute involved in Cannon—Title IX of the Education Amendments of 1972—a damages remedy was available. Barnes v. Gorman (2002). Narrowed Frankllin In that case, an action alleging that the municipal defendant had discriminated on the basis of disability, the Court held, without dissent, that punitive damages were unavailable under the Americans with Disabilities Act and the Rehabilitation Act, which incorporate by reference the remedies available under Title VI. Noting that Title VI's remedial scheme originates in Congress's Spending Power, the Court's opinion (per Scalia, J.) stressed that spending programs are similar in nature to a contract; that " 'if Congress intends to impose a condition on the grant of federal moneys, it must do so unambiguously'

Younger Breakdown

Justiciability: intervenors did not have standing because there was no prosecution toward him- your claim is not ripe - there is no injury yet Younger gets standing because he is being prosecuted but that also gives him other issues "Our Federalism,": This underlying reason for restraining courts of equity from interfering with criminal prosecutions is reinforced by an even more vital consideration, the notion of "comity," that is, a proper respect for state functions, a recognition of the fact that the entire country is made up of a Union of separate state governments, and a continuance of the belief that the National Government will fare best if the States and their institutions are left free to perform their separate functions in their separate ways. Ode to Ex parte Young and following cases: they have established the doctrine that, when absolutely necessary for protection of constitutional rights, courts of the United States have power to enjoin state officers from instituting criminal actions. But this may not be done, except under extraordinary circumstances, where the danger of irreparable loss is both great and immediate. Distinguishes Dombrowski:

Crowell v. Benson

Knudsen was a longshoreman who was injured while working on a boat for Benson. Knudsen brought a workers' compensation claim against Benson under the Longshoremen's and Harbor Workers' Compensation Act, 33 U.S.C. §§ 901-950 (the Act). Crowell, the workers' compensation judge, ruled in favor of Knudsen. Benson (plaintiff) then filed suit in federal district court against Crowell (defendant), alleging the workers' compensation court did not have jurisdiction, because Knudsen was not employed by Benson at the time of the injury. Benson amended his complaint to allege that the Act was unconstitutional- DP and AIII challenges. - Questions of fact: The statute says the commissioners findings are final. Otherwise the entire purpose of the statute is undermined and would be essentially a dry run of trial. - Questions of Jurisdictional fact: AIII court gets de novo review whether the injury occurred on navigable water of the US - Question of Facts on which constitutional rights depend: AIII gets to review de novo - Questions of law: AIII court gets to review de novo.

Where the plaintiff is the United States or a Federal Agency

Leiter Minerals, Inc. v. United States, 352 U.S. 220 (1957), the Court recognized an additional exception for injunctions sought by the United States. Leiter sued lessees of the United States in state court, seeking a declaration that it owned certain mineral rights and an accounting. The United States subsequently brought a federal action against Leiter and others to quiet title to the mineral rights and sought to enjoin the state proceedings. the policy of preventing conflict between federal and state courts "is much more compelling" in litigation between private parties than "when the United States seeks a stay to prevent a threatened irreparable injury to a national interest. The frustration of superior federal interests from precluding the Federal Government from obtaining a stay of state court proceedings except under the severe restrictions of 28 U.S.C. § 2283 would be so great that we cannot reasonably impute such a purpose to Congress from the general language of § 2283 alone"

actions for Declaratory and Injunctive Relief Concerning State and Local Laws Alleged to Be Preempted by Federal Law

Litigants often sue state or local officials, claiming that a state statute, regulation, or other action is preempted by a federal statute or by the federal Constitution and seeking injunctive relief and if federal law gives a right of action (express or implied) to a plaintiff to bring such an action, there is no doubt that the lawsuit "arises under" federal law. The harder questions concern the source and scope of any federal right of action. Express: Sometimes a federal statute confers an express right to sue a state official for injunction against a state law that the plaintiff contends is preempted.

Questions of Coverage

Lynch v. Household Finance Corp. (1972), the Court held (6-3) that a prejudgment garnishment was not a "proceeding" in state court within the scope of § 2283, and hence could be enjoined by a federal court, even though the garnishment might be necessary to obtain satisfaction of any subsequent judgment obtained by the creditor. The opinion emphasized that the garnishment could be instituted by the creditor's attorney, without judicial order, before filing suit. County of Imperial v. Munoz (1980), the county obtained a state court injunction against a landowner barring him from selling water from a well on his property for use outside the county. Three persons who had agreements to buy water from the landowner for use in Mexico then sued the county in federal court, alleging that the state court injunction violated the Commerce Clause. The Supreme Court reversed a grant of preliminary injunctive relief, relying on Atlantic Coast Line in rejecting the view that the state court proceedings had terminated. In Munoz, the Court ruled that unless the federal plaintiffs were "strangers," the injunction they sought was barred by § 2283; the case was remanded for an appropriate determination. Hale v. Bimco Trading, Inc. (1939). There, after one party obtained a state court order requiring a state agency to enforce a state statute, a different person obtained a federal court injunction barring the agency's enforcement of the statute. The Hale opinion upheld that injunction, rejecting the view that the Anti-Injunction Act in effect bars federal suit by strangers to a state court proceeding who seek to enjoin a statute that was the subject of that proceeding.

Advice of Counsel.

Malley v. Briggs (1986): a police officer presented arrest warrants to a state judge, who approved and signed them. The officer was later sued under § 1983 for having caused the arrest of individuals without probable cause. The Supreme Court refused to hold that the officer was absolutely immune because he had relied "on the judgment of a judicial officer in finding that probable cause exist[ed] and hence issuing the warrant". Messerschmidt v. Millender (2012): the fact that a neutral magistrate has issued a warrant" as a legally relevant indicator that "the officers acted in an objectively reasonable manner". Chief Justice Roberts' majority opinion described Malley as establishing an "exception" to the principle that a magistrate's decision to issue a warrant normally establishes the objective legal reasonableness of its execution. That exception applied, he said, only when "the magistrate so obviously erred that any reasonable officer would have recognized the error."

The Standard for Congressional Divestment of § 1331 Jurisdiction:

Mims v. Arrow Financial Services, LLC, 132 S.Ct. 740 (2012) , the Court held that an explicit congressional grant of state court jurisdiction over a federal cause of action did not oust federal court jurisdiction under § 1331. The Telephone Consumer Protection Act of 1991 (TCPA) (codified as amended at 42 U.S.C. § 227 ), which prohibits some telemarketing practices, authorizes States to bring civil actions on behalf of their citizens when there is a pattern or practice of violation; the federal courts have exclusive jurisdiction over such actions. The TCPA also authorizes private parties to seek judicial redress "if otherwise permitted by the laws or rules of court of a State, * * * in an appropriate court of that State". Justice Ginsburg's opinion noted the deeply rooted presumption that an explicit grant of federal court jurisdiction does not oust concurrent state court jurisdiction—a presumption that can be overcome only " 'by an explicit statutory directive, by unmistakable implication from legislative history, or by a clear incompatibility between statecourt jurisdiction and federal interests' "

Gunn v. Minton (2013)

Minton's action for patent infringement, the federal district court found Minton's patent invalid, on the basis that the invention had been on sale more than a year prior to the patent application. In a motion for reconsideration, Gunn, the lawyer for Minton, raised for the first time an argument that the prior uses of the invention were "experimental" and thus, under the patent laws, did not render the patent invalid. Minton then sued Gunn for malpractice in Texas state court, complaining of Gunn's tardiness in raising the experimental-use argument. Applying the Gramble requirements, the Court concluded that the last two requirements were not satisfied. ***As to the third criterion, the Court said that an issue was not "substantial" just because it was important to the plaintiff's case; rather, the relevant question was "the importance of the issue to the federal system as a whole."*** Here, the federal question—whether timely presentation of the experimental-use argument would have changed the outcome of the patent infringement lawsuit—would not change the outcome of the patent action, nor would it threaten the uniformity of federal law. As to Grable's fourth requirement, the Court noted that it was the states' responsibility to maintain standards of practice for members of their bars.

prosecutorial immunity administrative tasks

Mitchell v. Forsyth (1985), the Court thus held with little discussion that former Attorney General John N. Mitchell, in authorizing a warrantless wiretap for reasons of national security, was not acting in a prosecutorial capacity and hence was not shielded by absolute immunity. Burns v. Reed (1991) the plaintiff alleged that the defendant had violated her constitutional rights (i) by improperly advising the police that they could question her under hypnosis and that they "probably had probable cause" to arrest her and (ii) by presenting false and misleading evidence at a court appearance in support of an application for a search warrant. The Supreme Court held that the defendant's court appearance and his presentation of evidence at the hearing on the search warrant were protected by absolute immunity, but that his acts of providing advice to the police were subject only to a qualified immunity defense. The court stressed the rationale behind absolute prosecutorial immunity—"to free the judicial process from the harassment and intimidation associated with litigation"

Mitchum v. Foster

Mitchum v. Foster: A Florida prosecutor filed suit in state court against Mitchum to close down Mitchum's bookstore as a public nuisance under Florida law. The state court entered a preliminary injunction that closed operation of the bookstore. When further state proceedings stalled, Mitchum filed a civil rights action against state officials in the United States District Court for the Northern District of Florida. The claim alleged violations of rights guaranteed by the First and Fourteenth Amendments to the United States Constitution and sought injunctive and declaratory relief under 42 U.S.C. § 1983 to stay the state proceedings. The district court denied the injunction, holding that the federal anti-injunction statute, 28 U.S.C. § 2283, prohibited a stay of the state proceeding, in accordance with Atlantic Coast Line R. Co. v. Brotherhood of Locomotive Engineers, 398 U.S. 281. The United States Supreme Court granted certiorari on that issue. To qualify under the "expressly authorized" exception, a federal law need not contain an express reference to the statute (§2283), and it need not expressly authorize an injunction of a state court proceeding. However, the Act of Congress must have created a specific and uniquely federal right or remedy, enforceable in a federal court of equity, that could be frustrated if the federal court were not empowered to enjoin a state court proceeding.

Murdock v. City of Memphis (1875)

Murdock sued the City of Memphis in Tennessee state court. The complaint alleged that Murdock's ancestors sold land to the city in 1844 for construction of a United States naval depot. The deed contained a clause that conveyed the property back to the grantors if it was not used as a depot. The same year, the city sold the land to the United States by warranty deed, without conditions or mention of the depot. The United States then made improvements to the property for use as a naval depot. In 1854, by act of Congress, the United States transferred the property back to the City of Memphis for the use and benefit of the city. Murdock alleged that the property instead should have reverted back to the grantors under the condition in the original deed to the city. The Tennessee Supreme Court affirmed, stating that the act of Congress in conveying the property to the city controlled the issue. The United States Supreme Court granted Murdock's writ of error. It cannot, therefore, be maintained that it is in any case necessary for the security of the rights claimed under the Constitution, laws, or treaties of the United States that the Supreme Court should examine and decide other questions not of a Federal character.

Public Rights

Murray's Lessee: there are matters, involving public rights, which may be presented in such form that the judicial power is capable of acting on them, and which are susceptible of judicial determination, but which Congress may or may not bring within the cognizance of the courts of the United States, as it may deem proper." Thus the Congress, in exercising the powers confided to it, may establish "legislative" courts (as distinguished from "constitutional courts in which the judicial power conferred by the Constitution can be deposited") Congress has power to determine whether certain matters between gov. and citizens come into cognizance of AIII courts (e.g., issues related to interstate and foreign commerce, taxation, immigration, public lands, public health, post office facilities, pensions, and payments to veterans)

Brown JAckson

My conclusion is that no lower federal court should entertain a petition except on the following conditions: (1) that the petition raises a jurisdictional question involving federal law on which the state law allowed no access to its courts, either by habeas corpus or appeal from the conviction, and that he therefore has no state remedy; or (2) that the petition shows that although the law allows a remedy, he was actually improperly obstructed from making a record upon which the question could be presented, so that his remedy by way of ultimate application to this Court for certiorari has been frustrated

Does SCOTUS have the authority to order the delivery of the these commissions?

NO; There is statutory language in his favor. However the court does not view him as a public minister and therefore is not within the original jurisdiction of the court. Congress cannot expand this jurisdiction which stands in Article III, which this act aimed to do. It is emphatically the province and duty of the judicial department to say what the law is-> Acts repugnant to the Constitution cannot become the law. What should the supreme court do? Cannot exercise the jurisdiction - this is where we get the basic principle that is now very important- a law repugnant to the constitution

Thole v. U.S. Babnk N.A

Named plaintiff James Thole and others brought a class action lawsuit against U.S. Bank and other over alleged mismanagement of a defined benefit pension plan between 2007 and 2010. The plaintiffs alleged that the defendants violated Section 404, 405, and 406 of the Employee Retirement Income Security Act of 1974 (ERISA) by breaching their fiduciary duties and causing the plan to engage in prohibited transactions with a subsidiary company. The plaintiffs argued that as a result of these prohibited transactions, the plan suffered significant losses and became underfunded in 2008. The plaintiffs lack Article III standing to sue in federal court because, win or lose this case, they would still receive the exact same monthly benefits they are already entitled to receive. Justice Brett Kavanaugh authored the opinion for the 5-4 majority. As participants in a defined-benefit plan (as opposed to a defined-contribution plan, such as a 401(k)), the plaintiff retirees receive a fixed payment each month, notwithstanding any changes to the value of the plan or the investment decisions of the plan's fiduciaries.

Ward v. Love County

Natives forced to pay tax despite SCOTUS ruling sought repayment. To say that the county could collect these unlawful taxes by coercive means and not incur any obligations to pay them back is nothing short of saying that it could take or appropriate the property of these Indian allottees arbitrarily and without due process of law. Of course this would be in contravention of the 14th.

Anticommandeering

New York v. United States: the Court made clear that, despite Garcia, it had not abandoned judicial enforcement of constitutional limits on Congress' power to regulate the states. The Court invalidated, as outside the commerce448power, a federal statute requiring states that failed to provide for disposal of internally generated radioactive waste by a certain date to take title to the waste and thereby to assume associated liabilities. Congress lacked power either to "commandeer" the states into regulating waste disposal or to require states to take title to waste; accordingly, Congress lacked power to offer the states a choice between those two options. Printz v. United States: Justice Scalia's opinion for the Court held that Congress overstepped constitutional bounds by directing local law enforcement officials to conduct background checks on would-be purchasers of handguns. The Court said that just as the federal government may not order the states to legislate, it may not "command the States' officers, or those of their political subdivisions, to administer or enforce a federal regulatory program". The Court in Printz distinguished Testa on the same basis that it had in New York v. United States.

O'Shea v. Littleton

Nineteen individual citizens filed a civil rights class-action suit seeking injunctive relief against various government officials of Cairo, Illinois, including a judge and a magistrate (defendants). The district court complaint alleged a historical and continuing pattern of discriminatory law enforcement against African Americans. The allegations against the judge and magistrate involved racial discrimination in determining bail bonds, sentencing, and jury-fee practices. The complaint gave examples of alleged misconduct against certain state officials, but made only general allegations against the judge and magistrate. The relief sought was an injunction to prevent defendants from depriving class members of their constitutional rights in the future. AIII prohibits federal courts from hearing cases where resolution would be premature and where the injury is speculative and hypothetical.

Other noticeable factors regarding municipality liability

No Qualified Immunity: Owen v. City of Independence, 445 U.S. 622, 638 (1980): a municipality sued under Monell for violations committed by its officials does not have a qualified immunity from damages liability under § 1983, even if it can show that the officials would themselves be entitled to such an immunity in a § 1983 action against them in their personal capacity. No punitive damages: City of Newport v. Fact Concerts, Inc. (1981), holding that municipalities may not be held liable under § 1983 for punitive damages.-> based on idea that this goes beyond the compensatory purpose.

State's potential responses to habeas proceeding include

Non-exhaustion - didn't present the claim to state courts Procedural default Stone v. Powell (4th amendment claims usually not permitted) State-court fact determination is presumed correct-highly deferential: Many petitions are frivolous, and more than 40% of non-capital petitions are dismissed without the state's even filing a response. A state's response, when filed, often establishes a basis for dismissal without further proceedings.

Bevins Concurrence

Notes that it would be at least anomalous to conclude that the federal judiciary—while competent to choose among the range of traditional judicial remedies to implement statutory and common-law policies, and even to generate substantive rules governing primary behavior in furtherance of broadly formulated policies articulated by statute or Constitution—is powerless to accord a damage remedy to vindicate social policies which, by virtue of their inclusion in the Constitution, are aimed predominantly at restraining the Government as an instrument of the popular will. **More importantly, the presumed availability of federal equitable relief against threatened invasions of constitutional interests appears entirely to negate the contention that the status of an interest as constitutionally protected divests federal courts of the power to grant damages absent express congressional authorization.-> Injunctive relief makes no sense here What about exclusion rule? What if there is nothing to exclude? Then the exclusionary rule would not operate. The question then, is, as I see it, whether compensatory relief is "necessary" or "appropriate" to the vindication of the interest asserted. In resolving that question, it seems to me that the range of policy considerations we may take into account are at least as broad as those a legislature would consider with respect to an express statutory authorization of a traditional remedy.

Courts to Adjudicate Public Rights Disputes

Numerous bodies fall within the elusive category of federal tribunals that adjudicate public rights. Current examples include the Court of Federal Claims, the Tax Court, and the United States Court of Appeals for Veterans Claims. The contours of the public rights doctrine have never been clearly defined, although most if not all public rights disputes involve civil claims by or against the government.

United States v. Kimbell Foods, Inc.

O.K. Super Markets (O.K.) took out a loan from Kimbell Foods, Inc.. The loan was secured by certain O.K. equipment and merchandise. O.K. then took out a second loan, secured by the same property. This second loan was guaranteed by the United States Small Business Administration. O.K. defaulted on both loans and eventually sold its equipment and inventory. Kimbell brought suit, claiming that its interest in O.K.'s sale proceeds was superior to the SBA's interest. The Small Business Act, which governed SBA loans, did not address lien priority. The SBA argued, based on the rationale behind applying federal law to federal tax liens, that federal law should apply to prioritize its interest because applying state law to the SBA's loan program would harm its ability to collect on the loans. On appeal before the United States Supreme Court, the Court considered the question of whether state or federal common law governed the question of whether Kimbell or the SBA had lien priority. Court finds that the program needs to be governed by federal law, but then embraces state law to fill the gap in the federal law.

Monroe v. Pape

On October 29, 1958, thirteen police officers, including Frank Pape, arrived at James Monroe's Chicago apartment at 5:45 A.M. The officers broke down the door, forced Monroe and his wife to stand naked in their living room, and ransacked the apartment. Afterwards, James Monroe was escorted to police quarters and held for ten hours on "open" charges while he was interrogated about a murder. The police did not have a warrant for the search or the arrest, and refused Monroe permission to call his attorney. Monroe brought a complaint against each of the Chicago police officers individually and against the City of Chicago. The City of Chicago moved to dismiss the complaint on the ground that it was not liable under the Civil Rights Act nor for acts committed in performance of governmental functions. All defendants moved to dismiss, arguing that there was no cause of action under the Civil Rights Acts. The district court dismissed the complaint. The complaint states a claim under section 1983 against the officers regardless of whether state law also bans and provides a remedy for their actions.

One Step instead of Cooper 2 -step?

One step: is there a strong federal interest and if so do any of the policies conflict with state rule- if no then apply it state law. Factors for strong federal interest: can include - US financial or contractual interests, Federal people: US servicemembers, officers, employees in course of their duties Policy of a federal statutory scheme Areas of national concern (eg, immigration, foreign relations Courts virtually never claim that as a matter of discretion, federal common law should be formulated but that they lack competence to do so. Does this suggest that the two-step analysis collapses into a single question whether federal law should apply? Ernie Young: Starting from the premise that any genuine link between federal common lawmaking and implied congressional authorization is "tenuous indeed", he argues that "competence" to make federal common law must come, if anywhere, "from the existence of a conflict between state law and some preexisting federal policy"—a circumstance that gives rise to preemption under the Supremacy Clause. leading cases now often assert that state law applies unless displaced by a conflict with federal policy or interest [J]udicial creation of a special federal rule * * * is limited to situations where there is a 'significant conflict between some federal policy or interest and the use of state law.' See the approach below in Boyle.

Relief for Habeas

Ordinarily the only remedy awarded is release from custody, but the remedy is tailored to the nature of the constitutional violation. For example, can be a conditional release- if it is fixable then they give them time to retry the case. Tailored to the kind of error found.

D'Oench, Duhme & Co. v. FDIC

P gave bank a promissory note. The FDIC insured the bank and acquirred the note. Sued to recover. None of the statutes expressly answers the question directly. Held: federal law governs whether D'Oench has a defense when sued on the note by the FDIC because the federal policy to protect the FDIC from fraud bars that defense. Jackson Concurrence: Although acknowledging that there was no "general common law," Jackson added that "this is not to say that wherever we have occasion to decide a federal question which cannot be answered from federal statutes alone we may not resort to all the source materials of the common law, or that when we have fashioned an answer it does not become a part of the federal non-statutory or common law". Congress in creating the FDIC- the purpose has to be served and courts couldn't do that if they were limited to what Congress said. Look at statutory purpose.-> implied delegation Many commentators read the Clearfield-D'Oench Duhme line of cases to rest on the idea that a statute establishing a federal program can be understood to include an implied delegation to judges to supply necessary omissions.

Standing Redressability

P must allege that a court is likely able to provide a redress the injury

Injury-in-fact

P must allege that he personally suffered an actual or threatened injury. The threatened injury must be certainly impending.

Standing Causation

P must allege that the injury is fairly traceable to D's conduct Linda R.S. v. Richard D. (1973): Class action brought by mother of a child born out of wedlock, against state officials whose policy it was to bring non-support prosecutions only against fathers of children born in wedlock Sought injunction requiring prosecution of fathers of children born out of wedlock the court held that the mothers did not have standing. Plaintiff failed to allege sufficient nexus between her injury and the gov't action -> Requested relief would result in jailing of child's father, and would only speculatively result in future payment of child support. Regents Univ. Cal. v. Bakke (1978): White plaintiff had standing to challenge university's affirmative action program for minority students applying to medical school. Some amici argued that Bakke could not have standing because he had not shown that his asserted injury—being denied admission—would be redressed by a favorable outcome, since he may not have been admitted anyway. Court affirmed standing, arguing that Bakke's asserted relief would redress the injury of having been deprived, due to his race, the chance to compete for a spot in the entering medical school class

Reynoldsville Casket Co. v. Hyde (1995)

P sued out of state D in state court > 2 years after the accident before SCOTUS in Bendix held a state tolling statute unconstitutional because it treated out of state Ds worse than in-state D's. State court held the action timely refusing to apply Bendix retroactively. Before SCOTUS P argued that the state court was merely denying remedy because of P's reliance. In retroactively applying an earlier precedent that had invalidated Ohio's statute of limitations, the Court emphasized, in particular, that Harper's nonretroactivity principle should not be avoidable simply by asserting that the denial of relief was "based on 'remedy' rather than 'non-retroactivity'. Supremacy Clause requires application of Bendix to P's case

Heckler v. Mathew

P, a man, had standing to challenge distribution of greater Social Security benefits to women than to men b/c P framed his injury as the right to receive benefits that weren't distributed according to improper classifications. If he had merely sued for an increased amount of benefits, he likely could not show redressability, b/c decision would have led to decreased benefits. Is this in tension with Allen v. Wright- posit the distinction- in Allen the claim of unequal treatment is more indirect- more attenuated chain. Court takes advantage of the fact that it is a nationwide class.

Parden v. Terminal Ry. (1964).

Parden rejected Alabama's claim of immunity from an employee's federal court negligence action under the Federal Employers' Liability Act (FELA) against a state-owned railway. The case presented two major questions that form the basis of sovereign immunity analysis: (1) Did Congress in enacting the FELA intend to subject a State to suit in these circumstances? (2) Did it have power to do so, as against the State's claim of immunity?" The court answered yes to both, but the question two the second was a bit confused. Its analysis confusingly conflated an abrogation theory ("imposition of the FELA right of action upon interstate railroads cannot be precluded by sovereign immunity") with a theory of waiver or implied state consent ("Alabama, when it began operation of an interstate railroad after enactment of the FELA] necessarily consented to such suit as was authorized by that Act" In the mix, suggesting some sort of waiver issue was the idea that Alabama started an operating an interstate railroad after the FELA so it knows and had implicitly waived the right to suit.

Patchak v. Zinke

Patchak sues under APA to challenge DOI authority to take particular land into trust for a particular native tribe. Litigation ensues over Patchak's ability to bring the suit. SCOTUS said he had standing. Congress then passes a statute to settle things. First provision said that disputed land was held in trust and affirmed Secretary's decision. Second provision (jurisdiction stripping) said federal courts could not adjudicate claims, including existing ones, related to the land. This case is like Bank Markazi b/c Congress validly exercised its legislative power to change the law Distinctive setting- rely on the fact that it is a case against the government- waived its sovereign immunity Dissent (Roberts, C.J.): Congress should not be able to accomplish impermissible objective (dictating the rule of decision) by passing such a narrow jurisdiction stripping statute that doesn't really change the law. Founders really wanted separation between legislature and judiciary; they saw negative consequences of state legislatures interfering.

First Exception Sentencing

Penry v. Lynaugh (1989), where the claim was that execution of a prisoner with the mental capacity of a seven-year old violated the Eighth Amendment. A unanimous Court ruled that the first exception extends to new rules "prohibiting a certain category of punishment for a class of defendants because of their status or offense. Caspari v. Bohlen (1994), the Court found the first exception inapplicable. There, the petitioner claimed that providing the state a chance, at resentencing, to prove prior convictions (for purposes of sentence enhancement) that the state had failed to prove at the initial sentencing violated the Double Jeopardy Clause. The Court said that the petitioner's primary conduct was not beyond punishment because he could be sentenced to prison (whether as a repeat offender or not) on the underlying convictions.

Skelly Oil Co. v. Phillips Petroleum Co. (1950)

Phillips sues Skelly in federal court under the Federal DJ Act arguing that the certificate specified in contract had issued prior to Skelly's attempted termination and seeking a DJ that the contract is still in effect. Elements of that kind of law suit: 1) offer; 2) acceptance 3) consideration 4) performance by Phillips 5) no adequate remedy at law. Anticipated defense by Skelly: the contract has been terminated so there is no contract. If but for the DJ procedural the federal question would arise only as a defense to a non-federal question claim then there is no federal question jurisdiction over the DJ action-> the Declaratory Judgment Act as not conferring jurisdiction over declaratory actions when the underlying dispute could not otherwise have been heard in federal court. If Phillips sought damages from petitioners or specific performance of their contracts, it could not bring suit in a United States District Court on the theory that it was asserting a federal right. First time it would come up is with Skelly's defense- everyone is operating under the premise that the contract claim would not need to preemptively rebut that there is some act against it. . Congress enlarged the range of remedies available in the federal courts but did not extend their jurisdiction.

Aetna Life Ins v. Haworth

Plaintiff Aetna Life Insurance Company issued five life insurance policies to defendant. Under the Federal Declaratory Judgment Act of 1934 (Act), 28 U.S.C.S. § 400, Aetna sought a decree that certain insurance policies be declared null and void by reason of lapse for nonpayment of premiums. The trial court dismissed Aetna's suit upon the ground that there was no justiciable controversy. The Court of Appeals for the Eighth Circuit affirmed the trial court's decision. The Declaratory Judgment act is constitutional. - The Declaratory Judgment Act of 1934, in its limitation to 'cases of actual controversy,' manifestly has regard to the constitutional provision and is operative only in respect to controversies which are such in the constitutional sense. -> Thus the operation of the Declaratory Judgment Act is procedural only. - It calls, not for an advisory opinion upon a hypothetical basis, but for an adjudication of present right upon established facts.

Bell v. Hood + The Substantiality of the Asserted Federal Cause of Action.

Plaintiff and others brought an action in federal court against Defendant and other FBI agents seeking more than $3000 for violations of the Fourth and Fifth Amendments to the Constitution. D motioned for a 12 b6 failure to state a claim and a motion for summary judgment. The district court dismissed the complaint as one not arising under the Constitution or federal law. Claims that fit squarely within the constitution are within the jurisdiction of federal courts. The carve outs here are: 1) where the alleged claim under the Constitution or federal statutes clearly appears to be immaterial and made solely for the purpose of obtaining jurisdiction 2) where such a claim is wholly insubstantial and frivolous. So there are three categories: 1) state valid claim under federal law 2) no valid claim but not frivolous enough 3) could not possibly succeed so no fed q jurisdiction. Why does it matter?

Standing Constitutional Requirements

Plaintiffs wishing to sue in federal court must be able to identify a concrete injury 1) that he or she has sustained 2) that the defendant has caused and 3) that a properly framed judicial decree can redress.

Habeas Availability of Counsel

Prisoners, the great majority of whom are indigent, generally have no constitutional right to appointed counsel in state or federal collateral attacks on their convictions. Capital Cases. In 1988, Congress conferred a right to appointed counsel upon indigent federal habeas petitioners attacking a capital sentence or conviction Non-Capital Cases. In non-capital cases, the § 2254 Rules require appointment of counsel for an indigent petitioner if an evidentiary hearing is warranted (Rule 8(c)) or when necessary to utilize effectively discovery authorized by the court- rare though

Clapper v. Amnesty Int'l (2013)

Pursuant to FISA, AG may seek an order from the FISC to allow targeted surveillance of persons reasonably believed to be located outside the US to acquire foreign intelligence information. Ps asserted that this statute violated First and Fourth Amendments. Ps were attorneys and human rights organizations that worked with clients and sources abroad, and had fear that the gov't would be surveilling their conversations. Court rejected Ps' argument that they faced a substantial risk of injury based on the fact that their work requires them to communicate by phone or email w/ individuals abroad believed to be terrorist groups. None of the Ps could show that their communications had been intercepted or were likely to be intercepted.

Ward Breakdown

Question of jurisdiction: The OK Supreme Court's decision rested on an apparently non-federal (likely general law ground considering the various sources they cited). However the SCOTUS held the finding that they payments were voluntary was unsupported which knocks out the ground for the decision below. What then replaces it? "It is a well settled rule that money got through imposition may be recovered back. If a county obtains the money or property of others without authority the law independent of any statute will compel restitution or compensation." But what law? General law was not jurisdiction conferring (remember we are in Swift v. Tyson regime.) **One can understand this is saying the 14th amendment gives a right and imposes a remedial obligation on the counties.

Using Implied Pre-Emption instead of express>

Recent decisions interpreting such clauses, however, have suggested that such clauses do not preclude consideration of implied preemption as well, and some decisions find implied preemption without even reaching the question of express preemption. The question then is how much mileage can you get out of an express pre-emption clause? Leave purpose aside? How you would do that is difficult to see, but you could make different arguments - not think about purpose. Why? Not any easier to determine the preemption clause in certain instances and sometimes when interpreting the express pre-emption clause, the court imputes purpose anyway. Ex: ERISA: has an express pre-emption provision. Shall supersede any and all State laws insofar as they now or hereafter relate to any employee benefit plan. Boggs v. Boggs: Decedent left her sons her community property interest in her husbands pension undistributed pension benefits. Husbands remarried. Held: invalid under ERISA. Court did not reach ERISA's express pre-emption and instead it relied on conflict pre-emption: upholding sons' claim would conflict with ERISAs purpose to stream of income to surviving spouse. Rutledge v. Pharmaceutical Care: majority applies precedents concerning express pre-emption clause, including by considering the purpose of the statute. Thomas- applying precedents correctly but don't think about statutory purpose when you apply.

Strict time limits for Pre-trial motions:

Reece v. Georgia (1955): D was appointed counsel the day after his indictment. His counsel moved to quash the indictment six days later. The GA court rejected this motion as untimely (b/c post-indictment). Application of the time-bar violated due process here b/c the state refused to provide D's counsel until after the indictment stage. Michel v. Louisiana (1955): The Michel case involved three defendants who, the Louisiana courts found, had failed to comply with a state rule requiring that objections to the composition of the grand jury be raised before the expiration of the third judicial day following the end of the grand jury's term, or before trial, whichever was earlier. one D's lawyer filed a late motion to suppress after being appointed late; another D's motion was late b/c D had fled; another D's motion was late b/c his lawyer was elderly. the time-bars were adequate and not unreasonable b/c all of the Ds had a "reasonable opportunity" to have the issue adjudicated.

Why Require Exhaustion?

Rose v. Lundy (1982), the Court said that the exhaustion requirement serves to "protect the state courts' role in the enforcement of federal law", to "prevent disruption of state judicial proceedings", and to " 'minimize friction between our federal and state systems of justice by allowing the State an initial opportunity to pass upon and correct alleged violations of prisoners' federal rights' Though it is often said that the exhaustion requirement merely postpones federal review until the state courts have had the chance to consider a federal claim, the requirement has always had broader effects. Most importantly today, exhaustion may lead to the generation of state court determinations of law or fact, to which federal courts are obliged, under § 2254(d), to defer.

Harlow Breakdown Absolute Immunity

Responding the defendants claim for absolute immunity, the court first gives a little background on the norms- government officials are entitled to some form of immunity from suits for damages, but for executive officials in general our cases make plain that qualified immunity represents the norm. It only extends absolute immunity in certain circumstances. The court then works through its existing, somewhat conflicting precedent to conclude that the defendants are not entitled to absolute immunity as presidential aides: Butz v. Economou - secretary of agriculture gets only qualified immunity not absolute immunity. Court uses this to say if the secretary cannot get absolute, then why would an aide Gravel v. US- applies speech and debate clause immunity to legislative acts by senator's aide. President too gets absolute immunity so the defendants argue that this should extend to presidential aides. But the court while acknowledging the force behind this argument concludes its extending to be too sweeping would not be reconcilable with Butz-> dissent focuses on Gravel Petitioners also assert an entitlement to immunity based on the "special functions" of White House aides. For aides entrusted with discretionary authority in such sensitive areas as national security or foreign policy, absolute immunity might well be justified to protect the unhesitating performance of functions vital to the national interest.

Heyburn's Case: PA

Review of judicial judgments by the legislature not consistent with the judicial power of the Constitution. Any such tasks thus fall beyond its bounds. The court could not enforce the Act. Judges may not act in a non-judicial manner + their judicial opinions cannot be subject to the revision of the legislature and an executive officer (separation-of-powers concern)

United States v. Richardson

Richardson, a taxpayer interested in activities of the Central Intelligence Agency, sued the government to provide records detailing the CIA's expenditures. The Court held that Richardson did not have standing to sue. Using the two-pronged standing test of Flast v. Cohen (1968), Chief Justice Burger found that there was no "logical nexus between the status asserted [by Richardson as a taxpayer] and the claim sought to be adjudicated." It was clear to Burger that Richardson was not "a proper and appropriate party to invoke federal judicial power" on this issue. Generalized grievance b/c impact on P was the same as on any citizen, and did not allege that as a taxpayer, he is in danger of suffering any concrete injury as a result of the operation of the statute Challenge of this subject matter is reserved for the political process, not the federal courts.

Spokeo v. Robins (2016)

Robins brought suit under the Fair Credit Reporting Act against Spokeo, a "people search engine." Robins asserted that Spokeo had incorrect information about his background and current employment status that made it difficult for him to secure a job for which he was searching. Robins brought suit on his own behalf and on behalf of a class of others similarly situated. Congress's role in identifying and elevating intangible harms does not mean a P automatically satisfies the injury in fact requirement whenever a statute grants a person a statutory right and purports to authorize that person to sue to vindicate a right. -> a bare procedural violation divorced from any concrete harm would not satisfy standing without some concrete interest that is affected by the deprivation. The Court found that intangible injuries (such as that occasioned by the publication of an inaccurate report) could be concrete, but not simply because Congress created a right to sue. Instead, the concreteness inquiry focuses on whether the individual has suffered a real or de facto injury that Congress has made cognizable by granting a right to sue for its invasion.

Staub v. City of Baxley

Rose Staub was convicted and fined for attempting to organize a branch of the International Ladies' Garment Workers Union at Hazlehurst Manufacturing Company. She violated an ordinance in the neighboring town of Baxley, where many of the Manufacturing Co. workers lived. That ordinance required anyone soliciting members for a union to apply for a permit from the mayor and city council. The mayor and city council had unlimited discretion to grant or deny the permits for any reason. Staub argued that the ordinance violated her constitutional right to free speech. The Court of Appeals of Georgia affirmed the conviction, but did not consider the constitutional question because Staub did not attempt to comply with the ordinance. They held that appellant's failure to attack "specific sections" of the ordinance rendered it unnecessary, under Georgia procedure, "to pass upon . . . the constitutionality of the ordinance, or any other phase of the case" If a procedural requirement unduly burdens a federal right, the state rule is very likely inadequate. Making D separately count off each allegedly violated section would be an "arid ritual of meaningless form." (The law was also unconstitutional; this may have added to its inadequacy)

Limitations of Stone v. Powll

Rose v. Mitchell (1979): involved an allegation of racial discrimination in selection of the grand jury. When, as in that case, an untainted trial jury finds the prisoner guilty beyond a reasonable doubt, discrimination in selection of the grand jury—which typically has to find only a prima facie case of guilt—is highly unlikely to have led to conviction of an innocent person. But Justice Blackmun, writing for the Court, reached the merits of the discrimination claim, confining Stone to "cases involving the judicially created exclusionary rule" and distinguishing grand jury discrimination from the exclusionary remedy on a number of grounds. Withrow v. Williams (1993), where the petitioner claimed a violation of the Miranda rules. Justice Souter's majority opinion argued that unlike the Fourth Amendment's exclusionary rule, the Miranda decision "safeguards 'a fundamental trial right' " that is not "necessarily divorced from the correct ascertainment of guilt." Most importantly, eliminating habeas review of Miranda claims would not significantly unburden the courts. A claim of that the state court failed to exclude a coerced confession remains cognizable in habeas even if the evidence (apart from the confession) overwhelmingly establishes the petitioner's guilt. This is a one off decision that does not extend to other constitutional concerns- just 4th amendment concerns.

Procedural Requirements: Raising the argument below

SCOTUS does not entertain federal claims that were not raised below, but new arguments in support of previously raised claims will be allowed. Claims may be waived; arguments in support of a claim may not be waived. Has to be raised and ruled on. Statutory basis - 1257

Frank v. Mangum

SCOTUS refuses to grant the writ reasoning Frank had the chance to raise his arguments in the GA Supreme Court. No interest in finding whether he was innocent or not Holmes (dissent): Mob law does not become due process of law by securing assent of a terrorized juror

Allen v. Cooper

Same with copyright In 1996, a private researcher hired petitioner Frederick Allen and his company, Nautilus Productions, LLC, to document the recently discovered shipwreck of Blackbeard's Queen Anne's Revenge, which ran aground at Beaufort, North Carolina, in 1718. Allen documented the shipwreck for nearly twenty years in photographs and videos and registered his works with the U.S. Copyright Office. At some point before October 2013, the state of North Carolina posted online various of the copyrighted works of Allen without his permission. Allen sued the state for copyright infringement, and the state moved to dismiss on the grounds of sovereign immunity under the Eleventh Amendment of the U.S. Constitution. Allen argued that the Copyright Remedy Clarification Act (CRCA)—which defines potential infringers of copyright to include "any State, any instrumentality of a State, and any officer of a State or instrumentality of a State acting in his or her official capacity"—abrogates state sovereign immunity for copyright infringement claims. Congress lacked the authority to abrogate state sovereign immunity from copyright infringement suits.

First Exception Substantive Rights v. Procedural Rights

Schriro v. Summerline (2004): Recharacterized the first Teague exception New substantive rules generally apply retroactively new procedural rules do not.. This includes: 1) Decisions that narrow the scope of a federal criminal statute by interpreting its terms; and 2) Constitutional determinations that place particular conduct or persons covered by the statute beyond the State's power to punish. Footnote: we have sometimes referred to rules of this latter type as falling under an exception to Teague's bar on retroactive application of procedural rules; they are more accurately characterized as substantive rules not subject to the bar. Montgomery v. Louisiana (2016): Miller's rule prohibiting under 8A automatic sentencing (without considering mitigating circumstances at sentencing) of juvenile to life imprisonment is substantive and applies retroactively on appeal.

The Relationship of § 2254(d)(1) to Teague

Section 2254(d)(1) does not by its terms override the judge-made doctrine of Teague v. Lane. Ordinarily, however, § 2254(d)(1)'s limitation on review will be the key question, as that limitation is broader than the limitation established in Teague. But unlike Teague, § 2254(d)(1) applies only when a habeas claim was adjudicated in state court. where there is neither a state court determination on the merits nor a procedural barrier to the exercise of habeas jurisdiction, the Teague doctrine (both its general bar on relying on "new" law and the exceptions to that bar) applies Teague - threshold issue- need petitioner to benefit from it Relevant time: when the judgment of conviction becomes final on direct review Section 2254- need a state court final decision - not a procedural default. Flexibility as to order of decision. Can figure out what the right answer is Date of last state court determination on merits of the claim Not clear if 2254d1 implicitly takes in the exceptions. Has not spoken directly to this issue.

Successive Petition? Not when prior petition dismissed for failure to exhaust, and petitioner then does exhaust

Slack v. McDaniel (2000), a habeas petition included some claims that had not yet been presented to the state courts. The "total exhaustion" doctrine prevents a federal court from entertaining a petition that includes both exhausted and unexhausted claims. Therefore, upon the prisoner's motion and without objection from the state, the district court dismissed the petition without prejudice; the order granted leave to refile after completing the exhaustion of state remedies. After exhaustion, Slack filed a new habeas corpus petition. The Supreme Court held, 7-2, that the second petition was not successive, stressing that "the complete exhaustion rule is not to 'trap the unwary pro se prisoner' "

Hernendez v. Mesa

Sergio Adrián Hernández Güereca, a 15-year-old Mexican national, was playing with friends in the cement culvert between El Paso, Texas, and Cuidad Juarez, Mexico. Border Patrol Agent Jesus Mesa, Jr. arrived on the scene and detained one of Hernández's friends on U.S. territory. Hernández ran into Mexican territory and stood by a pillar near the culvert. From U.S. territory, Mesa fired at least two shots across the border at Hernández, one of which struck Hernández in the face and killed him. Hernández's parents filed a lawsuit against the officer and various other defendants alleging violation of their son's Fourth and Fifth Amendment rights. The Fifth Circuit held that Hernández lacked Fourth Amendment rights, but his parents were entitled to a remedy under Bivens v. Six Unknown Named Agents. What if they were all on the US side of the border? Definitional question of whether this is a new context. Could wonder- Novelty is really in the eyes of the beholder- level of generality - Dissenter: the court in Bivens was trying to control rogue federal agents- this is not a new context then. Court identifies three different ways in which this is different: Foreign relations National security Congress has been hesitant to create claims based on allegedly tortious actions Dissent further found it signficant that unlike Ziglar the defendant had no other recourse Thomas and Gorsuch think the court should overrule Bivens- no analytical coherence so delete it

Mootness:

Settled Supreme Court case law suggests that a live controversy must exist throughout. Monaghan characterized mootness as "the doctrine of standing set in a time frame. The requisite personal interest that must exist at the commencement of the litigation (standing) must continue through its existence (mootness)." With few exceptions, a federal court will not entertain a suit if judicial relief would no longer be useful to the parties involved b/c the dispute is no longer concrete and definite

Implied Rights of Action to Enjoin Preempted State Regulation.

Shaw v. Delta Air Lines, Inc (1983) unanimously upheld the availability of injunctive relief against a state law banning discrimination on the basis of pregnancy, on the ground that it was preempted by ERISA. The Court simply declared that "[i]t is beyond dispute that federal courts have jurisdiction over suits to enjoin state officials from interfering with federal rights." If what you are trying to do is say there is a federal statute that pre-empts state law you can sue for an injunction even when the statute does not say you can. Based primarily on the supremacy clause so there is federal question jurisdiction- gives a right of action. Although there are separate questions they still seemed to find an implied right under the Supremacy clause Armstrong v. Exceptional Child Ctr. (2015): Supremacy Clause does not create a right of action for P seeking to enjoin enforcement of a state law alleged to be preempted by federal law. Judge-made remedy in Shaw that allows federal courts to grant injunctive relief against state officers who are violating or planning to violate federal law. Just an application of the idea that the federal courts may in some circumstances grant injunctive relief against state officers who are violating or planning to violate federal law.-> We did not find an implied right of action in shaw- this is a judge made remedy that does not rest upon an implied right of action contained in the Supremacy Clause

Preemption Through Agency Decision-making:

Should Chevron deference apply? Reminder: Chevron deference: as to a statute within the scope of agency that if the statute is ambiguous the courts will defer to the ordinary idea of the agency. - Forgoing Chevron deference to an agency's preemption decision is said to promote the political safeguards of federalism by ensuring that Congress does not displace state law through means that fall short of bicameralism and presentment - The Court has not answered this question, but probably would not get deference if agency determines that federal statute that it administers preempts state law Agencies probably lack expertise re: preemption, and agency deference operates in context of agency expertise Watters v. Wachovia Bank (2007), three dissenting Justices (Chief Justice Roberts and Justices Stevens and Scalia) indicated their willingness to find that Chevron does not apply.

Individual Officers: Official Capacity Suits.

Some damages actions under § 1983 are filed against an officer in the officer's "official" capacity. When an official is sued for damages in an "official" (as opposed to an "individual") capacity, the suit will be treated as one against the official's employer. Accordingly, damages actions pleaded against state officials in their "official capacity" will ordinarily be dismissed as barred by the state's sovereign immunity. When officials are properly sued in their official capacities, the government can be ordered to pay damages and attorney's fees, provided that it had adequate notice and opportunity to defend. When equitable relief is sought, the defendant official is ordinarily named in an official capacity. See, Hutto v. Finney, 437 U.S. 678, 693 (1978). Even in a suit thus captioned, the Eleventh Amendment ordinarily interposes no bar if the relief is deemed prospective in character-> Young

Katz Breakdown

Some theory that if the property is in rem and not in the possession of the state then no sovereign immunity in the first place. Need uniform system such that a discharge is a discharge Relied more broadly on the nature of the bankruptcy power vested in Congress under Article I: The question, he stated, was not whether Congress could "abrogate" state sovereign immunity in the Bankruptcy Act (as Congress had attempted to do), because the history and justification of the Bankruptcy Clause, as well as legislation enacted immediately following ratification, "demonstrate that [the Bankruptcy Clause] was intended not just as a grant of legislative authority to Congress, but also to authorize limited subordination of state sovereign immunity in the bankruptcy arena"

Federal Maritime Commission [FMC] v. South Carolina State Ports Authority

South Carolina Maritime Services, Inc. (Maritime Services), asked the South Carolina State Ports Authority (SCSPA) five times for permission to berth a cruise ship, the M/V Tropic Sea, at the SCSPA's port facilities in Charleston, South Carolina. Some cruises offered by Maritime Services would allow passengers to participate in gambling activities while on board. The SCSPA repeatedly denied Maritime Services' requests, contending that it had an established policy of denying berths in the Port of Charleston to vessels whose primary purpose was gambling. Maritime Services file a complaint with the Federal Maritime Commission (FMC), arguing that SCSPA violated the Shipping Act by its denials. The complaint was referred to an Administrative Law Judge (ALJ), who found that the SCSPA, as an arm of the State of South Carolina, was entitled to sovereign immunity and thus dismissed the complaint . State sovereign immunity precluded a federal administrative agency from adjudicating a private party's complaint against an unconsenting state.

Sprint Communications, Inc. v. Jacobs (2013):

Sprint Communications, Inc. sought from a state utility board an order enjoining a local telecommunications company from withholding services to Sprint because of a fee dispute between Sprint and the local company. After the state board ruled against Sprint, Sprint brought an official-capacity suit against the board members in federal court, arguing that the board's ruling was preempted by the federal Telecommunications Act of 1996 and seeking declaratory and injunctive relief. Finding an "important state interest in regulating and enforcing ... state utility rates," the Eighth Circuit agreed with the district court that Younger abstention was appropriate. Supreme Court reversed, holding that Younger abstention is appropriate in only three categories of cases. 1) The first category encompasses instances when there is an "ongoing state criminal prosecution[]." 2) Another category encompasses cases when a federal court is asked to interfere with "pending 'civil proceedings involving certain orders ... uniquely in furtherance of the state courts' ability to perform their judicial functions. Challenge to civil contempt order Challenge to requirement for posting bond pending appeal 3) One other category of case in which Younger abstention is warranted includes some actions for civil enforcement. The state proceeding to review the board decision in Sprint also failed to fall within this category: "It is not 'akin to a criminal prosecution.' Generally, state proceedings "akin to a criminal prosecution" in 'important respects"

Standing v. Ripeness

Standing focuses on the qualitative sufficient to fulfill the requirement of AIII and whether the P has personally suffered that harm, whereas ripeness occurs on the temporal fact of whether that harm has yet occurred

Tafflin Scalia Concurrence

State courts have jurisdiction over federal causes of action not because it is "conferred" upon them by the Congress; nor even because their inherent powers permit them to entertain transitory causes of action arising under the laws of foreign sovereigns but because "[t]he laws of the United States are laws in the several States, and just as much binding on the citizens and courts thereof as the State laws are. . . . The two together form one system of jurisprudence, which constitutes the law of the land for the State; and the courts of the two jurisdictions are not foreign to each other. As an original unqualified proposition, it would be eminently arguable that depriving state courts of their sovereign authority to adjudicate the law of the land must be done, if not with the utmost clarity, at least expressly. Assuming, however, that exclusion by implication is possible, surely what is required is implication in the text of the statute, and not merely, as the second part of the Gulf Offshore dictum would permit, through "unmistakable implication from legislative history."

State Procedural law:

State law procedural grounds, like state substantive law, can be independent and adequate to preclude SCOTUS review. While the Supreme Court defers toward review on state substantive law as an independent and adequate state ground, they are highly deferential toward the state when it comes to state procedural grunds E.g. - Long filed his appeal late- this is independent and adequate state grounds - SCOTUS decision wouldn't change anything. What if state court ignores procedural default: should SCOTUS say we have no jurisdiction because of state procedural rule despite the state avoiding it? As long as its clear state did this- doesn't feel like a vehicle problem. If they just rule on the appeal and not bring it up, might be a vehicle problem

Innocence based on the evidence actually presented in the trial?

State prisoners can seek federal habeas relief on the ground that upon the record evidence adduced at the trial no rational trier of fact could have found proof of guilt beyond a reasonable doubt. Jackson v. Virginia (1979), the Court held that the due process requirement of proof beyond a reasonable doubt means not only that a jury must be so instructed, but also that the question whether a properly instructed jury could reasonably have found the evidence in the record to establish guilt beyond a reasonable doubt is itself a federal constitutional question cognizable on habeas.

ASARCO v. Kadish

State taxpayers and association of public school teachers challenged state statute governing mineral leases on school lands as void under federal law State supreme court invalidated the statute and remanded for entry of declaratory judgment and injunctive relief. When a state court has issued a judgment in a case where P in the original action had no standing to sue in federal court, SCOTUS can exercise jurisdiction on certiorari if the judgment of the state court causes direct, specific, and concrete injury to the parties who petition for cert review, where requisites of a case or controversy are also met. If brought initially in federal court, P would lack standing because of pure speculation that relief sought would result in tax reductions or increase in public school teacher pay Given how proceeding unfolded, SCOTUS could review the state court decision that proceeded to final judgment under more lenient state standing requirements. Because the decision invalidated mineral leases held by private leaseholders who sought cert, produced the kind of injury to them that would be cognizable under federal law

States and State Agencies as Defendants:

States are not persons under 1983 and it does not abrogate their 11A immunity Quern v. Jordan: Congress did not clearly manifest an intention in § 1983 to override the states' Eleventh Amendment immunity-> Did not meet the clear statement rule that is needed to abrogate under the 14th A -> 11A still applies Will v. Michigan Department of State Police: neither a state nor a state official acting in an official capacity is a "person" within the meaning of § 1983, at least when sued for retrospective relief. Justice White's opinion for the Court stated that in common usage, the term "person" does not include the sovereign. Invoking the "clear statement" requirement developed in Eleventh Amendment cases, he insisted on application of a similar approach to the question presented and concluded that the language of § 1983 did not satisfy such a requirement. Given that a principal reason for enacting § 1983 was to provide a federal forum, the Court found it implausible that Congress meant to create a liability under § 1983 that was enforceable only in a state court-> of course this was before Alden v. Maine

AEDPA Statute of Limitations

Statute of limitations: AEDPA added a one-year limitations period running from the latest of four specified dates. - [1] Day "on which judgment became final by conclusion of direct review or the expiration of the time for seeking such review." 28 U.S.C. §2244(d)(1)(A) - [2] Impediment to filing, created by state action in violation of the Constitution or federal law, was removed - [3] SCOTUS initially recognized a new constitutional right, retroactively applicable to cases on collateral review - [4] Factual predicate of the claim could have been discovered with due diligence (§2244(d)) The Supreme Court has indicated that petitions filed more than one year after the dates specified in § 2244(d) are nonetheless not time-barred if the claim falls within the doctrine of equitable tolling or if the petitioner can satisfy an exacting standard of actual innocence Claims adjudicated on merits by state court Evidentiary hearings: 28 U.S.C. §2254: State custody; remedies in Federal courts A fast-track opt in process for capital cases

Successive Petition? Not when claim of incompetency to be executed was unripe at time of first petition, and then the claim becomes ripe:

Stewart v. Martinez-Villareal (1998), the prisoner's claim of incompetency, filed in his first petition, was dismissed as premature. Then, after the other grounds for relief were denied, the prisoner, fearing that § 2244(b) might foreclose a second petition raising his incompetency claim, moved to re-open the earlier petition. Rejecting the state's argument that the incompetency claim now constituted a second petition, the Court found that the prisoner had filed only one petition and that a district court should rule on each claim presented at the time it becomes ripe. Panetti v. Quarterman (2007), a death row inmate's first petition did not include an incompetency claim; after that petition was denied and an execution date was set, the prisoner filed a second petition raising that claim. Stating that it was "hesitant to construe a statute, implemented to further the principles of comity, finality, and federalism, in a manner that would require unripe claims to be raised as a mere formality, to the benefit of no party", the Court concluded that "[t]he statutory bar on 'second or successive' applications does not apply to [a claim that one is incompetent to be executed] brought in an application filed when the claim is first ripe."

Susan B. Anthony List v. Driehaus (2014)

Substantial risk of administrative proceedings against advocacy group, if backed by the additional threat of criminal prosecution, sufficed for standing. SBA List opposes abortion, and made statements about Ohio representative that his vote for the ACA constituted voting for taxpayer funding of abortions. Candidate lost and withdrew. Substantial risk of injury suffices to establish standing. Here, threat of enforcement of false statement statute amounted to Art. III injury in fact- > because of History of past enforcement plus Ps intended to continue doing the same things in future elections. Substantial risk of administrative proceedings, if backed by additional threat of criminal prosecution, sufficed to establish standing.

Dombrowski v. Pfister (1965)

Suit was brought by a civil rights group and affiliated individuals to enjoin state officials from prosecuting or threatening to prosecute the plaintiffs for alleged violations of two Louisiana statutes criminalizing subversive activities. The mere possibility of erroneous initial application of constitutional standards will usually not amount to the irreparable injury necessary to justify a disruption of orderly state proceedings". But in the Court's view, the plaintiffs' allegations that the statutes were overbroad, if true, would establish the threat of "irreparable injury" warranting relief. --We hold the abstention doctrine is inappropriate for cases such as the present one where, unlike Douglas v. City of Jeannette, statutes are justifiably attacked on their face as abridging free expression, or as applied for the purpose of discouraging protected activities

Refund Remedies for Unconstitutionally Exacted Taxes

Supreme Court took a major step in the direction of holding that "a taxpayer [has] a constitutional right to litigate the legality of a tax", but then took at least half a step back. - Reich v. Collins, 513 U.S. 106, 109-10 (1994), the Court stated that " 'a denial by a state court of a recovery of [state] taxes exacted in violation of the laws or Constitution of the United States by compulsion is itself in contravention of the Fourteenth Amendment,' the sovereign immunity States traditionally enjoy in their own courts notwithstanding" - Alden v. Maine, 527 U.S. 706, 740 (1999), the Court characterized Reich as having held that state sovereign immunity had to yield only because the state, in requiring taxpayers to pay first and litigate later, had effectively promised to provide a post-deprivation remedy and was bound by the Due Process Clause to satisfy its promise. The Court's shifting rationale highlights Professor Hart's observation that the doctrine of sovereign immunity is difficult to integrate with constitutional understandings that one has a right to a remedy for a constitutional violation.

Tafflin v. Levitt

Tafflin and other holders of unpaid certificates of deposit sued Levitt and other officials of the failed bank that did not pay the CDs. The plaintiffs brought suit in federal district court under the Racketeer Influenced and Corrupt Organizations Act (RICO). The defendants filed a motion to dismiss the case, due to pending state-court litigation based on the same claim. The district court dismissed the complaint, ruling that it should abstain from hearing the action in deference to the pending litigation in Maryland state court. The United States Court of Appeals for the Fourth Circuit affirmed. The United States Supreme Court granted certiorari. The presumption of concurrent jurisdiction can be rebutted by an explicit statutory directive, by unmistakable implication from legislative history, or by a clear incompatibility between state-court jurisdiction and federal interests.

Exceptions to the Patsy Rule

Tax cases: no damages remedy where state provides plain adequate and complete remedy. Fair Assessment in Real Estate Ass'n v. McNary (1981) the Court applied principles of comity to require federal courts to decline jurisdiction in suits seeking a damages remedy for state taxation whenever the state provides a plain, adequate, and complete remedy. They agreed that when the state courts would require exhaustion of administrative remedies before entertaining a challenge to state taxes in which monetary relief was sought, a federal court entertaining a § 1983 action should ordinarily do likewise. Actions by Prisoners. The Prison Litigation Reform Act of 1995 ("PLRA"), 110 Stat. 1321 (1996), requires the exhaustion of "such administrative remedies as are available" prior to the filing of federal suits by prisoners challenging prison conditions under § 1983 "or any other Federal law". - Booth v. Churner (2001), applied the exhaustion requirement to a prisoner seeking only money damages, despite the unavailability of monetary relief in the administrative forum. You cannot exercise equitable power of excusing non-compliance- Congress sets the rules and we can only set exceptions when Congress wants us to. Jones v. Bock (2007) the Court held that a court of appeals had overstepped its authority by erecting procedural barriers to prisoner suits that went beyond the requirements of the PLRA. More specifically, the Court rejected rulings that had required inmates to plead exhaustion in their complaints (rather than treating non-exhaustion as an affirmative defense), barred suits against a defendant who had not been specifically named in an administrative grievance, and mandated total dismissal of complaints in which petitioners had failed to exhaust some, but not all, of their claims. Cannot just add more barriers to what the inmate faces - special pleading requirements

Taylor v. Riojas (2020)

Taylor was first placed (for four days) in a cell where feces covered nearly all of the floor, ceiling, window, and walls, and was even "packed inside the water faucet." Out of fear of this environment, Taylor did not eat and drink during those four days. He was then placed (for the next two days) in a different cell which was extremely cold and had only a clogged floor drain in lieu of a toilet. When Taylor eventually had to relieve his bladder in that cell, the drain overflowed and spilled raw sewage on the floor. The cell lacked a bunk, Taylor lacked clothing, and he had to sleep naked on that floor. The Fifth Circuit had held that the record, taken in the light most favorable to Taylor, would support findings that the cell conditions "exposed him to a substantial risk of serious harm and denied him the minimal civilized measure of life's necessities" and that the defendants knew of that risk and failed to take reasonable measures to remedy it - i.e., the record would support a finding that the defendants violated Taylor's Eighth Amendment rights. But the Fifth Circuit held that the defendants had lacked fair warning of the unconstitutionality of their actions, and thus that they were entitled to qualified immunity. The Supreme Court applied Hope v. Pelzer and United States v. Lanier and concluded that the Fifth Circuit had erred in holding that the defendant correctional officers were entitled to qualified immunity on Taylor's Eighth Amendment conditions-of-confinement claim.

Boyle Second Question - conflict?

That the procurement of equipment by the United States is an area of uniquely federal interest does not, however, end the inquiry. Displacement will occur only where, as we have variously described, a "significant conflict" exists between an identifiable "federal policy or interest and the [operation] of state law," or the application of state law would "frustrate specific objectives" of federal legislation In some cases, for example where the federal interest requires a uniform rule, the entire body of state law applicable to the area conflicts and is replaced by federal rules. We agree with the scope of displacement adopted by the Fourth Circuit here. Liability for design defects in military equipment cannot be imposed, pursuant to state law, when (1) the United States approved reasonably precise specifications; (2) the equipment conformed to those specifications; and (3) the supplier warned the United States about the dangers in the use of the equipment that were known to the supplier but not to the United States.

Justifiability

a cluster of related issues that define the scope of federal judicial power through categories such as standing, ripeness, mootness and the political question doctrine. - Standing- question as to who constitutes a proper plaintiff to invoke federal judicial power - Ripeness: when is a matter sufficiently immediate and concrete to justify judicial consideration - Mootness: what should a court do when a crucial element of a live dispute goes away during the adjudication process - Political question doctrine: what sorts of legal disputes does the Constitution mark off for exclusive resolution by other branches.

Unreasonable application

The "unreasonable application" clause will typically be the critical one in determining whether § 2254(d)(1) bars relief. Lockyer v. Andrade, 538 U.S. 63 (2003) (5-4), where the prisoner claimed that the imposition, under California's "three strikes" law, of two consecutive sentences of 25 years to life for each of two thefts of a handful of videotapes violated the Cruel and Unusual Punishment Clause. Justice O'Connor's opinion stressed that the Court's decisions "in this area have not been a model of clarity" and "have not established a clear or consistent path for courts to follow." The only "clearly established" doctrine is a "gross disproportionality principle, the precise contours of which are unclear, applicable only in the 'exceedingly rare' and 'extreme' case." Finding that the sentence at issue fell between two Supreme Court precedents court did not allow. If the "precise contours" of an established principle must be clear, § 2254(d)(1) is especially likely to preclude habeas relief when the prisoner invokes constitutional doctrine framed as a standard rather than as a rule. The more general the standard, the less likely a state-court decision will be found unreasonable. Example: whether prisoner was in custody for Miranda purposes.

Procedure for Filing a Second or Successive Petition

The 1996 amendments require a prisoner, before filing a successive petition in district court, to first file a motion in the court of appeals, seeking authorization to file the petition in district court. the court of appeals is to act on the motion within 30 days, § 2244(b)(3)(D), and will grant the motion if the petition satisfies the criteria set forth in § 2244(b)(2). Without such authorization, a prisoner may not file a successive habeas petition in the district court. Section 2244(b)(3)(E) provides that the grant or denial of authorization "by a court of appeals shall not be appealable and shall not be the subject of a petition for rehearing or for a writ of certiorari." ---Felker v. Turpin (1996) the Court unanimously ruled that § 2244(b)(3)(E) did not preclude the prisoner whose request for authorization had been denied from filing a petition in the Supreme Court seeking an original writ of habeas corpus, but that the standards set forth in § 2244(b)(2) would inform the Court's decision whether to grant relief.

Initial Extension of the Bivens Approach

The Bivens opinion suggested that implication of a damages remedy might be inappropriate in a case presenting (i) "special factors counselling hesitation in the absence of affirmative action by Congress" or (ii) "an explicit congressional declaration that plaintiff should be remitted to another remedy, equally effective in the view of Congress." But the Court's next two decisions in the Bivens line suggested a narrow compass for those limitations on the Bivens remedy

CFTC v. Schor

The Commodity Futures Trading Commission regulates the trading of commodity futures. Disgruntled customers may seek reparations from the CFTC for a broker's violations of the Commodity Exchange Act or CFTC regulations. The CTFA promulgated a regulation that allowed it to adjudicate any counterclaims that the broker might assert against a consumer if such counterclaims arose out of the same transaction or series of transactions set forth in the customer's complaint. Such counterclaims were not mandatory; brokers could pursue their action against the customer in state court as a contract action. Mr. Schor, a customer, filed a complaint against a broker, Conti Commodity Services, Inc. (Conti), for numerous violations of the CEA. In addition to defending itself from Schor's claims, Conti filed a state based counterclaim against Schor in the CFTC reparations proceeding. The CFTC ruled in favor of Conti, and Schor sought judicial review. Individuals own right to a fair and unbiased adjudicator- not here; they waived it. Separation of powers concerns: not waivable; admittedly a private right set by state law but other factors render the scheme permissible. They are structural concerns- but its okay here. Functional not formal distinction here- not broad jurisdiction. Only reaches small little pieces of claims

Ex parte Young Breakdown Jurisdiction

The Court first concluded that the circuit court had "arising under" jurisdiction, as the suit raised several federal questions: (i) whether enforcement of the rates would take property without due process of law, (ii) whether the penalties for violation were so enormous as to deny equal protection and due process, and (iii) whether the legislation interfered with interstate commerce - What if they said this is like stealing? And Young says I have legal authorization to take it and the railroad says- that violates the constitution? Then this starts to look like Mottley and there would not be jurisdiction. What else could be going on? ---Young could be viewed as an implied right of action under the Constitution- directly under the 14th amendment- federal law creates their right of action so there is jurisdiction. There is no doubt that a long and important string of subsequent cases has built on the understanding that the Constitution of its own force gives rise to causes of action to enjoin state officials engaged in constitutional violations—including those involving school segregation and suppression of voting rights—against which state tort law would have provided no protection. - Furthermore, it could be viewed as an exercise of the court's equitable jurisdiction. Remember in Armstrong - implied rights of action under the supremacy clause do not exist, an instead it was implied right of action under the equitable powers of the federal courts. Douglas v. Independent Living Center of Southern California (2012), Chief Justice Roberts, in a dissenting opinion joined by three other Justices, maintained that no cause of action would lie under the Supremacy Clause. Quoting an earlier opinion that cited Professor Harrison's article, the Chief Justice distinguished Ex parte Young and similar subsequent cases on the ground that they "present[ed] quite different questions involving 'the pre-emptive assertion in equity of a defense that would otherwise have been available in the State's enforcement proceedings at law.' What about Smith Court saying that cases not created by federal law, but having an important component via federal law can get in

Special Factors:

The Court has also declined to recognize Bivens actions when "special factors [might] counsel[ ] hesitation [even] in the absence of affirmative action by Congress". The court has tended to hold that anything attempting to be an extension (which is debatable in and of itself) there are many reasons to hesitate. This breaks down into two separate steps: 1) is this a new context or new category of defendant which is another problem as well. 2) if so then do any special factors counsel hesitation before extending Bivens to the new context or type of defendant?

Actual Innocence

The Court has also ruled that a habeas court may hear a defaulted claim, even where cause and prejudice cannot be shown, in one other, narrowly defined circumstance—when the petitioner makes an adequate showing of "actual innocence" Two ways of showing innocence: 1. Gateway claim of innocence- a way around a Sykes procedural default 2. Innocence Simplicter- free standing innocence claim

Zschernig v. Miller

The Court invalidated—as "an intrusion by the State into the field of foreign affairs which the Constitution entrusts to the President and the Congress"—an Oregon statute that, as interpreted and administered, barred foreigners from inheriting if their country did not (a) grant U.S. citizens reciprocal rights to inherit and (b) permit foreign legatees or heirs to enjoy the inherited property without confiscation. "The statute as construed seems to make unavoidable judicial criticism of nations established on a more authoritarian basis than our own" and could "impair the effective exercise of the Nation's foreign policy."

Verlinden Breakdown

The Court stated that the "controlling decision" is Osborn, which "reflects a broad conception of 'arising under' jurisdiction, according to which Congress may confer on the federal courts jurisdiction over any case or controversy that might call for the application of federal law." We need not now resolve that issue or decide the precise boundaries of Art. III jurisdiction, however, since the present case does not involve a mere speculative possibility that a federal question may arise at some point in the proceeding. Rather, a suit against a foreign state under this Act necessarily raises questions of substantive federal law at the very outset, and hence clearly 'arises under' federal law, as that term is used in Art. III. "By reason of its authority over foreign commerce and foreign relations, Congress has the undisputed power to decide, as a matter of federal law, whether and under what circumstances foreign nations should be amenable to suit in the United States. Actions against foreign sovereigns in our courts raise sensitive issues concerning the foreign relations of the United States, and the primacy of federal concerns is evident.

Order of Decision in Qualified Immunity Cases. Current Approach

The Court unanimously reversed itself in Pearson v. Callahan (2009), which held that lower court judges "should be permitted to exercise their sound discretion in deciding which of the two prongs of the qualified immunity analysis should be addressed first in light of the particular circumstances in the particular case at hand." Indeed now the court has said that in general, courts should think hard, and then think hard again, before turning small cases into large ones" by ruling on the merits when a case could be easily dismissed on qualified immunity grounds.

Lujan v. Defenders of Wildlife

The Endangered Species Act of 1973 (S7(a)(2)) required federal agencies to consult with the Secretary of the Interior to ensure that any authorized actions did not jeopardize endangered or threatened species or critically destroy natural habitats. A 1986 amendment to the act limited it scope to actions in the United States or on the high seas. ESA- any person may sue no his own behalf to enjoin any person including the US and any other governmental. Defenders of Wildlife and other organizations dedicated to wildlife conservation filed an action seeking a declaratory judgment that the new amendment erred by providing for a geographic limit on the original law. Court for the first time rejected a congressional attempt to confer standing-> This law converted the undifferentiated public interest into an individual right, infringing the Take Care Clause of AII.

Thomas v. Union Carbide Agricultural Products Co

The Federal Insecticide, Fungicide, and Rodenticide Act (FIFRA) set forth procedures for the registration of pesticides. FIFRA required companies registering pesticides to submit data to the Environmental Protection Agency (EPA) in support of their registrations. FIFRA provided compensation for the EPA's use of data submitted by such companies, and required the amount of compensation be set by binding arbitration. Thirteen large firms in the business of developing and marketing chemicals used in pesticides (chemical firms) (plaintiffs) brought suit against the United States (defendant) in the United States District Court for the Southern District of New York challenging the constitutionality of FIFRA. The chemical firms argued the requirement of binding arbitration violated Article III of the Constitution, which vested judicial power in Article III courts. The District Court found in favor of the chemical firms. Thomas appealed. > Public Rights suits are not limited just to suits where the government is a party. Encroachment on judicial branch is limited- think about what we are really worried about- the separation of powers is the important consideration here.

Trevino v. Thaler

The Martinez rule also applies where the state framework makes it highly unlikely that a typical D will have a meaningful opportunity to raise ineffective-assistance-of-trial-counsel claim on direct appeal.

FCL Civil Actions Past Regime and the role of general common law:

The Rules of Decision Act: The laws of the several states, except where the Constitution or treaties of the United States or Acts of Congress otherwise require or provide, shall be regarded as rules of decision in civil actions in the courts of the United States, in cases where they apply Swift v. Tyson: Tyson makes a written promise to pay two men. They then endorse it to Swift who sues Tyson for payment. Does he take the promise to pay free of the underlying fraud defense? NY: possibly yes; Federal Courts would say no. Which rule should they apply? Justice Story- Federal courts do not have to do what the state court will do. RDA does not include court decisions-> creates the general common law- Not tied to a specific federal constitutional or statutory provision and therefore neither the states nor the federal government could be authoritative in it. Overtime the general common law expanded massively and Federal judges were beginning to favor big business- states the other way Recall that Erie announced that "[t]here is no federal general common law." It did not, however, hold that there is no federal common law at all. Rather, Erie held that there was no federal general common law of the sort applied in Swift v. Tyson

Seminole Tribe of Florida v. Florida (1996)

The Seminole Tribe brought suit against the State of Florida for violating the good faith negotiations requirement of the Indian Gaming Regulatory Act (IGRA). Under the IGRA, the Tribe may engage in gaming (i.e., casino gambling) activities subject to Florida's good faith regulations. Florida moved to dismiss the Tribe's action, alleging that the lawsuit violated Florida's sovereign immunity. On appeal from the District Court's denial of Florida's motion to dismiss the lawsuit, the Court of Appeals reversed, holding that the Eleventh Amendment shielded Florida from federal suit and that under Ex Parte Young, the Tribe may not enforce its right to good faith negotiations by naming Florida's governor as a party to the suit. Overrules Union Gas- Neither the Commerce Clause nor the Indian Commerce Clause allow Congress to abrogate state sovereign immunity.

Murdock Breakdown

The State courts are the appropriate tribunals, as this court has repeatedly held, for the decision of questions arising under their local law, whether statutory or otherwise. And it is not lightly to be presumed that Congress acted upon a principle which implies a distrust of their integrity or of their ability to construe those laws correctly. Our system has been based upon the fundamental principle that this jurisdiction was limited to the correction of errors relating solely to Federal law. When it appears that the Federal question was decided erroneously against the plaintiff in error, we must then reverse the case undoubtedly, if there are no other issues decided in it than that. If the federal issues are dispositive there can be no doubt that the judgment of the State court must be reversed, and under the new act this court can either render the final judgment or decree here, or remand the case to the State court for that purpose. But when we find that the State court has decided the Federal question erroneously, then to prevent a useless and profitless reversal, which can do the plaintiff in error no good, and can only embarrass and delay the defendant, we must so far look into the remainder of the record as to see whether the decision of the Federal question alone is sufficient to dispose of the case, or to require its reversal. whether there exist other matters in the record actually decided by the State court which are sufficient to maintain the judgment of that court, notwithstanding the error in deciding the Federal question.

St. Cyr Breakdown

The Supreme Court held that the 1996 amendments had not foreclosed jurisdiction under the pre-existing general grant of habeas corpus jurisdiction in 28 U.S.C. § 2241. In reaching that conclusion, the Court relied heavily on a series of interpretive presumptions: The strong presumption in favor of judicial review of administrative action The longstanding rule requiring a clear statement of congressional intent to repeal habeas jurisdiction" and Most important of all, the presumption that statutes should be construed, where possible, to avoid serious constitutional problems. [A]t the absolute minimum, the Suspension Clause protects the writ 'as it existed in 1789.' At its historical core, the writ of habeas corpus has served as a means of reviewing the legality of executive detention A serious suspension issue would be presented if we were to accept the INS submission that the 1996 statutes have withdrawn that power from federal judges and provided no adequate substitute for its exercise. -> avoidance based interpretation.

The Relationship Between the Remedy Under § 1983 and the Writ of Federal Habeas Corpus.

The Supreme Court, in Preiser v. Rodriguez, 411 U.S. 475 (1973), and Heck v. Humphrey, 512 U.S. 477 (1994), has effectively subordinated the § 1983 remedy to the writ of habeas corpus when the remedies would overlap (and to some extent, even when they do not), holding that § 1983 may not be resorted to if the direct or indirect effect of granting relief would be to invalidate an existing state court conviction.

Clearfield Trust Co. v. United States

The United States issued a check to an employee of the Works Progress Administration in April 1936. The check was mailed to the employee's home. At some point, the check was intercepted and stolen by an unknown party. The thief then brought the check to a J.C. Penney Co. (defendant) store where he forged the employee's signature as an endorsement and exchanged the check for merchandise. J.C. Penney then endorsed the check to Clearfield Trust Co. (defendant), who endorsed it and submitted it with several other checks to the Federal Reserve Bank of Philadelphia. Twelve days after the check was issued, in May 1936, the federal employee informed his supervisor that he had not yet received his check. His supervisor then informed other members of the government. In August 1937, the United States filed suit in United States District Court against Clearfield Trust, seeking to recover the lost funds. J.C. Penney eventually intervened as a defendant. The district court ruled that Pennsylvania, rather than federal law dictated the outcome of the case. The court then held for Clearfield, finding that under Pennsylvania law the government had unreasonably delayed in informing Clearfield Trust that the check was a forgery. The rights and duties of the United States on commercial paper which it issues are governed by federal rather than local law.

Allen v. Wright

The Wright family and other parents of African American public school children, brought a nationwide class action suit against the Secretary of the Treasury and the Commissioner of Internal Revenue at the Internal Revenue Service. The parents argued that the failure of the IRS to deny tax-exempt status to racially-segregated private schools caused injury to their children on two grounds. Firstly, the parents alleged that the IRS's failure to comply with desegregation laws caused them direct harm by creating a climate of stigma against their children. Secondly, the parents alleged that their children's ability to attend a desegregated school had been directly impaired because the IRS's failure to remove private schools' tax-exempt status effectively encouraged the continued segregation of schools. P's claims: IRS has not adopted sufficient stds and procedures- wanted forward looking relief- deny tax exemption. Article III standing requires that a plaintiff allege a harm directly traceable to specific action on the part of the defendant.

Smith v. Kansas City Title & Trust Co

The case involved a shareholder's suit to enjoin a trust company from investing in federal bonds issued by Federal Land Banks or Joint-Stock Land Banks under authority of an Act of Congress. That Act explicitly provided that these federal bonds constituted lawful investments for all fiduciary and trust funds. The shareholder claimed a right to relief on the ground that state law prohibited the trust company from investing in bonds not issued pursuant to a valid law, and that the federal statute under which the bonds were issued was invalid under the federal Constitution. Where it appears from the bill or statement of the plaintiff that the right to relief depends upon the construction or application of the Constitution or laws of the United States, and that such federal claim is not merely colorable, and rests upon a reasonable foundation, the District Court has jurisdiction under this provision." Smith's recognition of jurisdiction might seem to depend on the fact that the federal question there was one of constitutional law.3 The Grable opinion states that "constitutional questions may be the more likely ones to reach the level of substantiality that can justify federal jurisdiction.

Franchise Tax Board v. Construction Laborers Vacation Trust

The case involved a trust established to provide paid vacation benefits earned by construction employees in California. Some of the employee-beneficiaries of the trust were delinquent in paying their California state income taxes. The Franchise Tax Board, the California agency charged with enforcement of the income tax, filed a complaint in state court against the trust. The first claim sought to enforce three levies against the trust, which would require the trust to pay to the Board amounts equal to the tax delinquencies of employee beneficiaries of the trust. The second claim sought a declaration that the Board's regulatory authority was not preempted by ERISA. After the trust removed the case to federal court under 28 U.S.C. § 1441, the Board moved to remand. Endorses the broad interpretation of Skelly- Federal question exists only if either P or D could bring a coercive action that falls within Federal question jurisdiction.

Oshea breakdown

The complaint failed to satisfy the threshold requirement imposed by Art. III of the Constitution that those who seek to invoke the power of federal courts must allege an actual case or controversy. Plaintiffs "must allege some threatened or actual injury resulting from the putatively illegal action before a federal court may assume jurisdiction." Past exposure to illegal conduct does not in itself show a present case or controversy regarding injunctive relief, however, if unaccompanied by any continuing, present adverse effects. Neither the complaint nor respondents' counsel suggested that any of the named plaintiffs at the time the complaint was filed were themselves serving an allegedly illegal sentence or were on trial or awaiting trial before petitioners. it seems to us that attempting to anticipate whether and when these respondents will be charged with crime and will be made to appear before either petitioner takes us into the area of speculation and conjecture.

Seminole Tribe and Ex Parte Young

The court finds it inapplicable here: IGRA remedial scheme that renders Young remedy inappropriate. Allowing for an ex parte young remedy would make the remedy mere surplusage- people would file Ex parte young suits to get more complete remedy. That Congress chose to impose upon a state liability that is significantly more limited than would be imposed under Young, indicates that Congress did not want to create the latter under §2710 This suggests that there is a further limitation on Young - when Congress provides a remedial scheme we are less willing to allow a Young Suit- seems similar to the Bivens analysis. By contrast with this quite modest set of sanctions, an action brought against a state official under Ex parte Young would expose that official to the full remedial powers of a federal court, including, presumably, contempt sanctions. If § 2710(d)(3) could be enforced in a suit under Ex parte Young, § 2710(d)(7) would have been superfluous

Probabilistic Harm

The court has said that injury in fact must be concrete, imminent and not speculative. However, injury that a cause of action seeks to prevent may be a statistical probability of harm. Injury is potentially not speculative if the conduct creates a substantial risk of injury, but Court has not been clear about what "substantial risk" means. Not clear how much Court allows standing for probabilistic injuries. Seems like it has to be individual, not group odds (Earth Island Institute). At least substantially likely; maybe more. But maybe relaxed if the risk of injury is really severe. (Susan B. Anthony). Three tiers- 1) occurring 2) certainly impending 3) substantial risk.

Ex parte Young Breakdown

The court rejects the argument that Young should have been protected by the 11A: The act to be enforced is alleged to be unconstitutional, and if it be so, the use of the name of the State to enforce an unconstitutional act to the injury of complainants is a proceeding without the authority of and one which does not affect the State in its sovereign or governmental capacity. It is simply an illegal act upon the part of a state official in attempting by the use of the name of the State to enforce a legislative enactment which is void because unconstitutional. Does this make sense though? The court essentially says that he is a state as to have the 14th A apply (only applies to states, not individuals in their own capacity), but not a state for the 11th A to apply.

Hart and Remedy

The denial of any remedy is one thing—that raises the question we're postponing. But the denial of one remedy while another is left open, or the substitution of one for another, is very different. It must be plain that Congress necessarily has a wide choice in the selection of remedies, and that a complaint about action of this kind can rarely be of constitutional dimension. They have the right to a remedy for a tax- congress needs to provide some remedy for constitutional harm.

United States v. Munsingwear, Inc.

The established practice of the Court in dealing with a civil case from a court in the federal system which has become moot is to reverse or vacate the judgment below and remand with a direction to dismiss . Munsingwear remains the controlling authority for civil cases that—through happenstance, conduct not attributable to the parties, or the unilateral action of the prevailing party in the lower court—become moot either (i) on appeal, (ii) during the pendency of a petition for certiorari, or (iii) after the grant of such a petition but prior to decision by the Supreme Court.

Vendo Co Breakdown

The federal statute in question must create a federal right enforceable in equity that would be frustrated- can you apply the Mitchum requirements here? - Right or remedy? One could argue that this is - Enforceable in a federal court of equity? Seems to have the same language as 1983 - Would the right to injunction be frustrated if the federal court were barred from issuing injunctions against state court proceedings? They are making us not compete by going to state court and getting a 7 million To rule otherwise would "eviscerate" § 2283 "since the ultimate logic of this position can mean no less than that virtually all federal statutes authorizing injunctive relief are exceptions to § 2283". Justice Blackmun, joined by Chief Justice Burger, concurred, but on the very different theory that § 16 was an "expressly authorized" exception only in the "narrowly limited circumstances", not found in the present case, where state court proceedings "are themselves part of a 'pattern of baseless, repetitive claims' that are being used as an anticompetitive device"

Prominent justifications for the new federal common law

The first and broadest position—advanced in commentary but never adopted by the Court—states that federal courts possess general authority to craft common law whenever federal interests are at stake. - Objectios: ---The first objection rests on the premise that policy decisions generally should be made by politically accountable branches of government. ---A second objection sounds in federalism. Critics emphasize the ways in which the cumbersome lawmaking procedures prescribed by the Constitution—for example, bicameral enactment and either presidential assent or a legislative supermajority—protect state interest ---Contradicts the REA- Clearfield and numerous cases thereafter have fashioned federal common law while ignoring the Rules of Decision Act. A second justification treats the new federal common law as legitimate when traceable, in some degree, to a source of federal authority derived from a statute or the Constitution. aspects of federal common law may be justifiable, for example, as an implication from the constitutional structure, as the product of an implicit delegation of power from Congress to supply an apparent omission in a statutory scheme, or as a tool for addressing a conflict between state law and federal statutory policy.

Sentence Summary orders

The hurdle interposed by § 2254(d)(1) has proved to be extremely difficult for petitioners to overcome and applies even where there is a single sentence order. Harrington v. Richter, 562 U.S. 86 (2011), the Supreme Court considered the application of § 2254(d) to summary state court decisions that deny federal constitutional claims on the merits without explanation. After Richter's conviction became final on direct review, he petitioned directly to the California Supreme Court for a state writ of habeas corpus. That court denied the petition in a one sentence summary order. When Richter's federal habeas corpus case reached the Supreme Court, it ruled, without dissent, that the California Supreme Court's order was an adjudication on the merits within the meaning of § 2254(d)

Court develops rationale equally applicable to Planter's Bank.

The idea is that the court creates a rationale that encompasses any law suit with the bank- federal law ingredient rationale. Banks right to sue or contract is implicated. Bank of the United States v. Planters' Bank of Ga.,: the existence of the Bank of the United States as a federal corporation was only a relatively minor premise of the claim for relief. Osborn's reasoning was applied to uphold the constitutionality of a grant of jurisdiction allowing the removal to federal court of actions against federally chartered railroad corporations. Pacific Railroad Removal Cases: The Court said that "the corporations now before us not only derive their existence, but their powers, their functions, their duties, and a large portion of their resources, from [Acts of Congress], and by virtue thereof sustain important relations to the government of the United States." However, this is no longer good law, Congress overrode it. Unlike the Bank of the United States, these railroads were not federal instrumentalities primarily carrying out government policy, a difference that Justice Frankfurter viewed as significant.

Supplemental Jurisdiction.

The jurisdiction could be seen as a version of supplemental jurisdiction. There plainly is federal question jurisdiction over the federal law issues that arise in bankruptcy, and jurisdiction over the adverse claims between trustee and third parties could be considered to be sufficiently related to the federal law proceeding as to be a single case or controversy. Where in article III section 2 is supplemental jurisdiction? Seems like it has to come under the arising under grant- something beyond just what arises under federal law. Gibbs: Arising from the same set of facts. The idea is that it is an interpretation of the statutory grant - carries with it some degree of supplemental. The same appears to be true for the constitutional grant.

The Absolute Immunity of Legislators.

The only explicit source in the Constitution for official immunity of any kind appears in Article I, Section 6, which states that Senators and Representatives "shall in all Cases, except Treason, Felony, and Breach of Peace, be privileged from Arrest1046during their Attendance at the Session of their respective Houses, and in going to and returning from the same; and for any Speech or Debate in either House, they shall not be questioned in any other Place." Kilbourn v. Thompson (1880). Relying on the English tradition of parliamentary privilege, the Court held that federal legislators who had voted for a resolution ordering the plaintiff to be imprisoned for contempt of Congress were immune from damages liability in a suit for false imprisonment. Subsequent cases have interpreted the Speech or Debate Clause to shield all "legislative acts"—matters that are "an integral part of the deliberative and communicative processes by which Members participate in committee and House proceedings with respect to the consideration and passage or rejection of proposed legislation or with respect to other matters which the Constitution places within the jurisdiction of either Gravel extended this immunity as well to their legislative aides who were indispensable in them fulfilling their tasks.

Circumvent judicial immunity?

The only way to circumvent judicial immunity is to show that a judge was acting "in the clear absence of all jurisdiction" or was not performing a "judicial act" - Forrester v. White: the Supreme Court unanimously ruled that a judge was acting in an administrative rather than a judicial capacity, and hence was not entitled to absolute immunity, when he fired a probation officer, allegedly on account of her sex and thus in violation of the Fourteenth Amendment. However courts have been extremely stringent on finding that they acted outside of a judicial capacity, even where there was no docket or anything. - Stump v. Sparkman (1978): an Indiana judge had approved ex parte a petition filed by parents of a fifteen-year-old girl to have her sterilized without her knowledge. When she later sued the judge for damages under § 1983, the Supreme Court ruled that he was absolutely immune: since he presided over a court of general jurisdiction, he had not acted wholly outside his jurisdiction, and he did not lose his immunity simply because no state statute specifically authorized his conduct --In the Court's view, whether a judge's action is a "judicial act" depends on whether (i) it is a function normally performed by a judge, and (ii) the parties' expectations revealed that they were dealing with the judge in his judicial capacity.

Teague v. Lane

The petitioner, a black man, was convicted in an Illinois state court for three counts of attempted murder, two counts of armed robbery, and one count of aggravated battery by an all white jury. During jury selection, the prosecutor used all ten of his peremptory challenges to exclude blacks. The petitioner's attorney used one of his ten challenges to exclude a black woman married to a police officer. The petitioner's attorney moved for a mistrial after six blacks were struck, but the trial court denied the motion. Teague again moved for a mistrial after an additional four blacks were struck. The prosecutor defended the challenges by stating that he was trying to achieve a balance of men and women on the jury. The trial court again denied the motion. The petitioner appealed to the Illinois Appellate Court which rejected his claim that the challenges denied him the right to be tried by a jury representative of the community. The petitioner then filed a petition for writ of habeas corpus in United States District Court ("District Court") again arguing the fair cross section claim, and arguing that under Swain v. Alabama, a prosecutor could be questioned about his use of peremptory challenges once he volunteered an explanation. New constitutional rules do not apply retroactively on collateral review.

Idaho v. Coeur d'Alene Tribe (1997)

The plaintiffs (an Indian Tribe and several of its members) brought a federal court action against state officials and agencies, and the state itself, seeking declaratory and injunctive relief based on a claim of ownership of certain submerged and related lands. As the case came to the Supreme Court, only the state officers, who had been sued in their individual capacities, remained as defendants, but the majority held (5-4) that suit against them was barred by the Eleventh Amendment. Young- does not apply to a suit agaisnt state officials that would extinguish the state's control over submerged lands that the state claims as an integral part of its territory.-> the state's special concern—deeply rooted in English history—for its sovereign control of submerged lands and the extraordinarily intrusive effect on state interests that a judgment for the plaintiffs would have. This is like saying the state is losing control over its own territory- too big means no ex parte young? In what the Justices referred to as the "principal opinion", Justice Kennedy contended that over the years, the doctrine of Ex parte Young had become an essentially discretionary one in which the federal courts—in determining whether suit against an officer was permitted—looked to a variety of factors in striking an appropriate balance. But he has not gotten a majority to endorse this.

Schweiker v. Chilicky

The plaintiffs there had been improperly denied disability benefits under the Social Security Act, but subsequently were awarded, or had pending administrative applications for, full retroactive benefits. They sued federal and state policymaking officials, alleging that the defendants had denied them due process by adopting policies that resulted in the improper denials. The complaint sought equitable relief and damages for "emotional distress and for loss of food, shelter and other necessities proximately caused by [defendants'] denial of benefits''. The Supreme Court refused to permit a Bivens action for the alleged due process violation, stressing that the Social Security Act provided an elaborate administrative and judicial remedy. While acknowledging that the statutory remedy permitted only the restoration of improperly denied benefits and that a Bivens remedy would offer the prospect of recovering damages for emotional distress or other hardships caused by the delay in awarding benefits, the Act nonetheless "provide[d] meaningful safeguards or remedies for the rights of persons situated as [plaintiffs] were." Congress provided adequate even though it is not as good- again, compare against to Carlson What congress thinks is adequate.

Prosecutorial Immunity.

The purposes and scope of prosecutorial immunity are similar to those of judicial immunity. Permitting damages claims would allow a civil rights suit to retry the criminal case and make the prosecutor less zealous. Imbler v. Pachtman (1976): a § 1983 action alleging that a state prosecutor had knowingly introduced perjured testimony, the Court found that prosecutorial immunity was well-established at common law and that absolute rather than qualified immunity was appropriate. Otherwise, a criminal defendant could "transform his resentment into the ascription of improper and malicious actions to the State's advocate", and suits "could be expected with some frequency". In subsequent cases the Court has made clear that prosecutorial immunity extends only to prosecutorial functions related to courtroom advocacy and not to administrative tasks.

Relitigation Exception

The relitigation exception permits a federal court to enjoin a state court to respect the preclusive effect of a federal judgment. Questions to ask: 1) Is there a judgment from federal court that predates state court activity? 2) What are the bounds of that prior federal court judgment? 3) Would proceeding in state court actually contravene what was decided in federal court?

Ziglar v. Abbasi

The respondents in this case are a group of male, non-U.S. citizens, most of whom are Muslim of Middle Eastern origin who were detained after the September 11, 2001 attacks and treated as "of interest" in the government's investigation of these events. In their original claims, the plaintiffs alleged that they were detained without notice of the charges against them or information about how they were determined to be "of interest," that their access to counsel and the courts was interfered with, and that they were subjected to excessively harsh treatment during their detention. The plaintiffs sued a number of government officials and argued that the government used their status as non-citizens to detain them when the government's real purpose was to investigate whether they were terrorists and that the conditions of their confinement violated their Constitutional rights to due process and equal protection. While that appeal was pending, some of the plaintiffs settled their claims against the government and the U.S. Supreme Court decided Ashcroft v. Iqbal, which held that a complaint must allege sufficient facts to be plausible on its face and to allow a court to draw the reasonable inference that the defendant is liable for the claimed conduct. The court suggests that Bivens came at a time where they were more comfortable implying rights, but now that is extreme. The court suggested that the new question is whether the judiciary is well suited absent congressional action or instruction to consider and weight the costs and benefits of allowing damages actions. Seems like it was not a new context, but the court distinguishes on the basis that this was pursuant to a high level executive police created in light of a terrorist attack.-> sensitive issue of national security. Congress kept an eye on it but did not create a cause of action here - they expressed interest in this and required the DOJ to report, but did not create a right of action. Congress may want civil litigation- creates discovery

Ex parte Young

The state of Minnesota passed laws limiting what railroads could charge in that state and established severe penalties, including fines and jail for violators. Some shareholders of Northern Pacific Railway filed a lawsuit in the United States Circuit Court for the District of Minnesota asserting that the laws were unconstitutional as violating the Due Process Clause of the Fourteenth Amendment, as well as the Dormant Commerce Clause. The shareholders sued the railroads to prevent them from complying with the law. They also sued Edward T. Young, the Attorney General of Minnesota, to prevent him from enforcing the law. Young argued that the Eleventh Amendment, which prohibits states from being sued by citizens of other states, meant that the court did not have jurisdiction to hear the case. The federal circuit court still issued an injunction against Young enforcing the law. The following day, Young filed a proceeding in Minnesota state court to force the railroads to comply with the statute. The court ordered Young to explain his actions, and he reiterated his Eleventh Amendment claim, whereupon the court held Young in contempt. Young filed a petition with the United States Supreme Court for a writ of habeas corpus for his release. When an officer in proceeding under such enactment comes into conflict with the superior authority of that Constitution, he is in that case stripped of his official or representative character and is subjected in his person to the consequences of his individual conduct.

Osborn v. Bank of the United States

The state of Ohio levied taxes on each branch of the United States Bank in Ohio. Although the Court ruled in McCulloch v. Maryland that such taxes were unconstitutional, Ohio persisted in its enforcement of the tax. Defying a circuit court injunction, Ralph Osborn, the Ohio State Auditor, forcibly seized funds from the Bank. The circuit court then ordered Osborn and his colleagues to repay the amount seized. - What does the statute say: yes this a statutory grant, Congress intended to grant and did grant jurisdiction so long as it is constitutional. - Constitutionality of the grant: We think, then, that when a question to which the judicial power of the Union is extended by the constitution, forms an ingredient of the original cause, it is in the power of Congress to give the Circuit Courts jurisdiction of that cause, although other questions of fact or of law may be involved in it. It is not only itself the mere creature of a law, but all its actions and all its rights are dependent on the same law. Can a being, thus constituted, have a case which does not arise literally, as well as substantially, under the law This has already been settled and doesn't seem to have a huge impact on this particular case. However, the court doesn't seem bothered- this is a very generous test.

Individual Capacity v. Official Capacity. The "official" immunities apply only in suits in which officials are sued in their "individual" capacities, meaning that damages are at least nominally sought from the officials themselves, rather than from government treasuries. There are two kinds of immunity that can be offered for officials: qualified and absolute

There are two kinds of immunity that can be offered for officials: qualified and absolute Absolute immunity: Officials protected by absolute immunity—such as judges acting in a judicial capacity—cannot be held liable for damages under any circumstances, even if they intentionally or maliciously violate clearly established federal rights. Because an official with absolute immunity has no obligation to justify action taken, a suit barred by the doctrine can ordinarily be dismissed on a Rule 12(b)(6) motion. Absolute immunity thus eliminates nearly all of the possible burden, expense, and anxiety of litigation. Qualified immunity: will result in the dismissal of a lawsuit on a Rule 12(b)(6) motion only if the defendant is not alleged to have violated "clearly established statutory or constitutional rights of which a reasonable person would have known."

Where federal law (including federal common law)creates the right of action

There is federal question jurisdiction under 1331 unless: 1) The fed claim is clearly immaterial and made solely for purposes of creating jurisdiction or 2) Wholly insubstantial and frivolous

Jurisdiction Grants

There is one general grant of jurisdiction: 28 U.S. Code § 1331 - Federal Question: The district courts shall have original jurisdiction of all civil actions arising under the Constitution, laws, or treaties of the United States. There exists other statutory grants such as those involved in Osborn v. Bank of The United States;

Textile Workers Union v. Lincoln Mills

There, the union sued the employer to compel arbitration of grievances, as called for by the collective bargaining agreement. Section 301(a) of the Labor Management Relations Act of 1947 (the Taft-Hartley Act), 29 U.S.C. § 185, confers federal court jurisdiction on suits for violation of a collective bargaining agreement between union and employer. Section 301(b) provides a few slivers of substantive federal law: a union and employer shall be bound by the acts of their agents; a union may sue or be sued as an entity in a federal court; and any money judgment against a union in a federal district court is enforceable only against the organization, not against individual members. The Act does not indicate whose law governs the enforceability of collective bargaining agreements. Justice Douglas' opinion for the Court noted the holdings of a majority of lower courts that § 301(a) was more than jurisdictional—that it authorized federal courts to fashion federal common law to enforce collective bargaining agreements, including the promise to arbitrate. But Jurisdiction is likely not enough think diversity

Cannon Factor 3

Third, a private remedy should not be implied if it would frustrate the underlying purpose of the legislative scheme. On the other hand, when that remedy is necessary or at least helpful to the accomplishment of the statutory purpose, the Court is decidedly receptive to its implication under the statute. Title IX, like its model Title VI, sought to accomplish two related, but nevertheless somewhat different, objectives. First, Congress wanted to avoid the use of federal resources to support discriminatory practices; second, it wanted to provide individual citizens effective protection against those practices. The first purpose is generally served by the statutory procedure for the termination of federal financial support for institutions engaged in discriminatory practices. Plus the second individualized one can only be achieved via a private remedy.

Granfinanciera, S.A. v. Nordberg:

This 1989 decision was actually about whether there is a right to jury trial, in a bankruptcy proceeding, with respect to a fraudulent conveyance action, based on federal statutory law, filed on behalf of a bankruptcy estate against a noncreditor. Relied on reformulation of public-right doctrine offered in Thomas concurrence: "rejected view that 'a mater of public right must at minimum arise between gov. and others" Real question is whether Congress created a seemingly private right that is so closely integrated in to public regulatory scheme as to be a matter appropriate for agency resolution with limited involvement by AIII judiciary Until the Granfinanciera decision, the precedents suggested that the Seventh Amendment has little if any applicability to adjudication before administrative agencies.

Brown v. Allen

This matter involved the consolidated cases of three black defendants, Brown, Speller, and Daniels (defendants), who were sentenced to death in North Carolina for interracial rape or murder. Brown's and Speller's state appeals were denied on the merits by the Supreme Court of North Carolina, and certiorari was denied. Daniels' state appeal was denied as untimely. The defendants then filed habeas corpus petitions in federal district court, alleging racial discrimination in jury selection and various other claims. Daniels' petition was denied because the state court appeal was filed late. The federal district court reviewed the state trial court record in Brown's and Speller's cases and took additional evidence in the Speller case. The district court denied the petitions, ruling that the state court's federal constitutional rulings were supported by the evidence and should not be disturbed. The district court also ruled in Speller's case that the constitutional claims were unsubstantiated. Clearly ruled that a federal court should routinely relitigate the merits of federal constitutional issues that the state court had decided adversely to the state prisoner

Implied conflict preemption

This occurs when it is impossible to comply with both state and federal law, or when state law "stands as an obstacle to the accomplishment and execution of the full purposes and objectives of Congress. Because the "obstacle" component of conflict preemption rests on judicial attribution of legislative purpose to Congress, it can be viewed as depending on the assumption that such purposes can be ascertained, and as being in tension with more text-focused theories of statutory interpretation. Yet the Court, despite its increasing emphasis on textual interpretation, continues to invoke implied preemption principles to invalidate state legislation as contrary to federal purposes. However, there has been some pushback now against that.

Bacon v. Rutland R.R.

marked the limits of the Prentis doctrine and made its rationale unmistakable. There, in a suit to enjoin the Public Service Commission of Vermont from enforcing an order concerning a passenger station, the defendants invoked the Prentis case in objecting that the railroad had failed to appeal to the state supreme court. But the Court, speaking again through Justice Holmes, held that at the judicial stage the railroads had a right to resort to the federal courts at once. Finding that no legislative powers had been conferred upon the Supreme Court of Vermont, it sustained the federal court's jurisdiction.

Lockerty v. Phillips

To combat wartime inflation, the Emergency Price Control Act of 1942 created an Article III court called the Emergency Court of Appeals (ECA), consisting of three or more federal district or circuit judges. The ECA was given "the powers of a district court with respect to the jurisdiction conferred on it", except that it had no power to issue any temporary restraining order or interlocutory decree staying the effectiveness of any order, regulation, or price schedule issued under the Act. The Act permitted a challenge to any order, regulation, or price schedule by filing a protest with the Administrator. But the ECA lacked jurisdiction over other actions under the Act. In particular, federal district courts had jurisdiction over (i) actions by the Administrator to enjoin violations of the Act or secure an order directing compliance; (ii) treble damage actions by the Administrator or a buyer for342over-ceiling sales; and (iii) criminal prosecutions for willful violations. This case involved a suit by wholesale meat dealers in a federal district court to restrain the United States Attorney from prosecuting them for violations of certain price regulations. SCOTUS affirmed: The Congressional power to ordain and establish inferior courts includes the power 'of investing them with jurisdiction either limited, concurrent, or exclusive, and of withholding jurisdiction from them in the exact degrees and character which to Congress may seem proper for the public good. Congress can restrict equity jurisdiction and require recourse to administrative process. [I]t is plain that Congress has power to provide that the equity jurisdiction to restrain enforcement of the Act, or of regulations promulgated under it, be restricted to the Emergency Court, and, upon review of its decisions, to this Court.

Unmistakable implication from legislative history-

To rebut the presumption of concurrent jurisdiction, the question is not whether any intent at all may be divined from legislative silence on the issue, but whether Congress in its deliberations may be said to have affirmatively or unmistakably intended jurisdiction to be exclusively federal. [Tafflin] We don't know when people make statements whether people approve of that being included in the statute. This would suggest that the legislative history is pretty unreliable- don't know if people agreed with a particular statement. -> rely on the statute instead It's no longer clear that implied exclusion exists: Yellow Freight System, Inc. v. Donnelly: decided only a few months after Tafflin, a unanimous Court expressed stronger disfavor of implied exclusion. Holding that state courts have concurrent jurisdiction over private civil actions brought under Title VII of the 1964 Civil Rights Act, Justice Stevens' opinion said that the omission of any express provision making federal jurisdiction exclusive "is strong, and arguably sufficient, evidence that Congress had no such intent".

Post Cannon Developments: the court seemed to continue through with the Cannon concurrence and dissents thoughts and restrict even more so.

Touche Ross & Co. v. Redington (1979), in which the Court refused to imply a right of action under § 17(a) of the Securities Act of 1934—which imposes recordkeeping and reporting requirements on broker-dealers and others—against an accounting firm that had audited and prepared the required reports for a securities firm that became insolvent. Justice Rehnquist, for the majority, carried through on the warning he issued in Cannon: "Here, the statute by its terms grants no private rights to any identifiable class and proscribes no conduct as unlawful. And the legislative history of the 1934 Act simply does not speak to the issue of private remedies under § 17(a). At least in such a case as this, the inquiry ends there Herman & MacLean v. Huddleston, 459 U.S. 375 (1983), the Court reaffirmed the private right of action under § 10(b) of the Securities Exchange Act of 1934 for violation of Rule 10b-5, the anti-fraud regulation promulgated by the SEC pursuant to § 10(b). The Court has not, however, taken the step of overturning earlier decisions that had recognized a private right of action under a more liberal approach. **Merrill Lynch, Pierce, Fenner & Smith, Inc. v. Curran (1982), the Court, by a 5-4 vote, inferred a private cause of action under the Commodity Exchange Act, primarily on the theory that such a remedy was part of the "contemporary legal context" that was preserved when Congress undertook a comprehensive revision of the Act in 1974. Rested on the premise that courts should more easily recognize rights of action under statutes enacted during a period when Congress' expectations may have been influenced by the Borak line of decisions.-> look at the landscape on which Congress created the statute.*****

In re Davis

Troy Anthony Davis was convicted in Georgia state court of murdering a police officer and he was sentenced to death. Both his direct appeal and his state habeas petition failed. In 2001 Davis filed a federal habeas petition raising several constitutional claims. The district court denied relief and the court of appeals affirmed. After the state set an execution date in 2007, Davis moved in state court for a new trial on the basis of evidence that included recantations by eyewitnesses who had testified at trial; affidavits asserting that another man had confessed to the murder; affidavits by witnesses (at or near the murder scene) who had not testified at trial; expert testimony on ballistics and eyewitness identifications; and affidavits from jurors. The state court denied relief and the Georgia Supreme Court affirmed; the U.S. Supreme Court granted a stay of Davis's execution but that stay dissolved when the Court denied Davis's petition for certiorari. Over a dissent, the federal court of appeals denied Davis's application for leave to file a second or successive federal habeas petition. Pursuant to original writ in SCOTUS execution was stayed and sent to District of Georgia. To succeed the defendant have to show by clear and convincing evidence that no reasonable juror would find guilt beyond a reasonable doubt.

Kimbell Breakdown

Two different questions: 1) governed by federal law? 2) what is the federal principle- are there reasons to absorb state principles. Governed by Federal law: Since the [agency derives its] authority to effectuate loan transactions from specific Acts of Congress passed in the exercise of a "constitutional function or power," their rights, as well, should derive from a federal source. When Government activities "aris[e] from and bea[r] heavily upon a federal . . . program," the Constitution and Acts of Congress " 'require' otherwise than that state law govern of its own force. Is there reason to absorb the state law here or craft a distinctly federal purpose: Controversies directly affecting the operations of federal programs, although governed by federal law, do not inevitably require resort to uniform federal rules.

Allen v. Wright Breakdown

Two injuries: 1) Court characterizes it as stigmatic injury- does not count this as a cognizable injury. 2) injury to ability to attend desegregated public schools- court holds this is a cognizable injury, but causation is too attenuated-> IRS allows tax exemptions- makes private school less expensive- impedes public schools by letting people go there cheaper. Court says they won't recognize that- court finds the chain of causation Article III of the Constitution confines the federal courts to adjudicating actual "cases" and "controversies." , the "case or controversy" requirement defines with respect to the Judicial Branch the idea of separation of powers on which the Federal Government is founded.

Prospective Decisions: The question of prospectivity typically has arisen when a ruling of the Court overrules a past decision or departs significantly from settled understandings and establishes rights and obligations not previously recognized in our history.

Until fairly recently, the Court's tradition had been to give even novel rulings full retroactivity—applying them not only to the case before it but also to other cases pending on direct review. - The Warren Court broke with that tradition in criminal cases. In Linkletter v. Walker, 381 U.S. 618 (1965), the Court held that the rule in Mapp v. Ohio, 367 U.S. 643 (1961), though applied to the case in which it was announced, would not be applied in collateral review of a final state court conviction. - In Johnson v. New Jersey, 384 U.S. 719 (1966), the Court asserted power to make rules of criminal procedure nonretroactive, by which it meant that the new rule would be applied in the case before it but otherwise would not apply in cases pending on direct review. In civil cases, too, the Court also gestured toward very expansive judicial discretion to announce new rules prospectively.

Vendo Co. v. Lektro-Vend Corp. (1977)

Vendo sued Lektro-Vend (and others) in state court for breach of an agreement not to compete. Lektro-Vend countered with a federal court action against Vendo, alleging that the agreement violated the federal antitrust laws and that the state court suit was designed to stifle competition and to harass. After a state court judgment against Lektro-Vend for over $7 million was affirmed, the federal district court enjoined enforcement of the judgment, holding that § 16 of the Clayton Act, 15 U.S.C. § 26 (which authorizes private suits for injunctive relief against antitrust violations) was an "expressly authorized" exception to § 2283. The Seventh Circuit affirmed. A splintered Supreme Court reversed. Justice Rehnquist, for himself and Justices Stewart and Powell, argued that, unlike § 1983, § 16 of the Clayton Act could be given "its intended scope" without a stay of state court proceedings; there was no indication that Congress "was concerned with the possibility that state-court proceedings would be used to violate the Sherman or Clayton Acts". -> no majority for how broad

Boyle First Question- Federal interest

We have held that a few areas, involving "uniquely federal interests' are so committed by the Constitution and laws of the United States to federal control that state law is pre-empted and replaced, where necessary, by federal law of a content prescribed (absent explicit statutory directive) by the courts—so-called "federal common law." Case here borders on two areas. Obligations to, and rights of, the United States under its contracts are governed exclusively by federal law. Civil liability of federal officials for actions taken in the course of their duty also governed by federal law. It is true that where "litigation is purely between private parties and does not touch the rights and duties of the United States federal law does not govern. However, this impacts a federal interest- The imposition of liability on Government contractors will directly affect the terms of Government contracts: either the contractor will decline to manufacture the design specified by the Government, or it will raise its price.

Stern v. Marshall (2011)

Vickie Lynn Marshall sued Pierce Marshall in a Texas probate court, alleging that Pierce had fraudulently induced his father and Vickie's husband, to leave Vickie out of his will. Pierce was ultimately successful in that suit but not before Vickie filed a bankruptcy petition in California. Pierce filed a proof of claim in the bankruptcy court, alleging that Vickie had defamed him and seeking a declaration that the defamation claim was nondischargeable. Vickie counterclaimed that Pierce had tortiously interfered with J. Howard's intended gift to her. The bankruptcy court decided in favor of Vickie. The district court ruled upon Pierce's appeal after entry of the Texas judgment for Pierce. The court concluded that Vickie's counterclaim was not a "core proceeding" subject to adjudication by the bankruptcy court. Rather than give effect to the Texas judgment, however, the district court reviewed the case on the merits and ultimately awarded Vickie more than $88 million in damages. On remand, the Ninth Circuit concluded that Vickie's counterclaim was not a core proceeding and that the district court should have given preclusive effect to the Texas state court judgment. Issue: Vickie's state law counterclaim against Pierce- this counter claim is independent of federal bankruptcy law and won't necessarily be resolved by a ruling on Pierce's proof of claim for his defamation claim. Public Rights Factors of stern: 1) parties- is it the gov 2) source of the claim 3) tied in with a federal regulatory scheme?

How likely of recurrence?

Vitek v. Jones: Jones brought procedural challenge to his transfer from prison to mental hospital. Against the background of Jones' mental illness, it was not "absolutely clear" that the challenged wrong would not recur. Justice Stewart, for three dissenters, argued that the case was moot because there was "no demonstrated probability" of recurrence. City of Erie v. Pap's A.M.: the state court enjoined the enforcement of the city's anti-nudity ordinance. However, the nude dancing establishment that brought the challenge had closed by the time the case reached the Supreme Court, and the proprietor submitted an affidavit attesting that he did not intend to resume business. By a vote of 7-2, the Court held the case not moot. The city suffered "an ongoing injury because it [was] barred from enforcing" its public nudity prohibitions; Pap's was "still incorporated" and "could again decide to operate a nude dancing establishment in Erie" By the time case reached SCOTUS Pap's had closed and the owner said in affidavit that he didn't intend to resume business but the court says that they don't want litigants to manipulate the system.

Terry Williams Stevens

Was contrary to and an unreasonable application of Strickland. They misunderstood SCOTUS precedent + unreasonable because it did not consider the mitigation evidence. Stevens interpretation of the statute did not get a majority. He is saying that it was codifying Teague- It is perfectly clear that AEDPA codifies Teague to the extent that Teague requires federal habeas courts to deny relief that is contingent upon a rule of law not clearly established at the time the state conviction became final. When we apply this we should do the same thing. Congress wanted to set a mood of careful attention to State. We are not persuaded that the phrases define two mutually exclusive categories of questions. Most constitutional questions that arise in habeas corpus proceedings require the federal judge to apply a rule of law to a set of facts W]e think the phrase surely capacious enough to include a finding that the state-court "decision" is simply "erroneous" or wrong.

Correspondence of the Justices

Washington and Jefferson wanted the SCOTUS to opine on a few issues, but they said they would not: "the lines of separation drawn by the Constitution between the three departments of the government" - Separation of Powers: judiciary is a check on another branch and a court of last resort; advisory opinions violate both - Neither the constitutional text nor the discussions at the Constitutional Convention reflected any clear prohibition against advisory opinions. - Opinion Clause (AII, § 2, cl. 1): The President . . . may require the Opinion, in writing, of the principal Officer in each of the executive Departments, upon any Subject relating to the Duties of their respective Offices

What does concrete mean?

We care about history - English or American law- has to be close to something allowed Judgment of Congress- embrace the Lujan concurrence- can give it Needs to be great risk of harm- example that presumably wouldn't suffice

Non-Advisory

We do find that there are some decisions that are a bit questionable-> don't affect the rights sought: 1) Purely prospective decisions; 2) Harmless error; 3) Alternative holdings 4) Dictum; 5) Extrajudicial holdings; 6) Lectures

Constitutional avoidance round 2

We don't have that many instances of Congress's acts being struck down with this issue because The Supreme Court has ordinarily found ways to avoid squarely facing the question whether Congress may strip all courts of jurisdiction to entertain constitutional claims. Webster v. Doe: an action by a former CIA employee against the CIA Director alleging an unlawful discharge, the Court interpreted a statutory review provision as barring consideration of non-constitutional challenges but not constitutional challenges. In finding that the provision did not satisfy the "heightened showing" of intent required to construe it as barring judicial review of constitutional challenges, the Court noted the " 'serious constitutional question' that would arise if a federal statute were construed to deny any judicial forum for a colorable constitutional claim".

Moore v. Dempsey

We shall not say more concerning the corrective process afforded to the petitioners than that it does not seem to us sufficient to allow a judge to escape the duty of examining the fact for himself if true as alleged they make the trial absolutely void. Began the recognition of the role of the federal courts to review state cases. One explanation for the differing results is that in Frank, the state appellate court, in reviewing the trial proceedings, had already provided fair corrective process, whereas in Moore, the Court may have held that the state appellate review of the alleged mob domination had been so perfunctory as not to constitute an adequate state corrective process, therefore permitting federal habeas review.

Procedural Default

What happens when the relevant claim was not presented to the state court at all, or was not presented to the state court in accordance with the state's procedural requirements? Link to exhaustion: If a state process is still available for raising a non-exhausted claim the petitioner is required to exhaust that state remedy before seeking federal habeas relief on that claim If the state process is no longer available for a claim that the petitioner did not exhaust, then the question of procedural default arises Some habeas petitions include a claim that was not raised in state court at all or that was not raised in accordance with state procedural requirements. Typically, a state court will treat a procedural default as forfeiting the prisoner's right to an adjudication on the merits. And so long as the state procedural ground is "adequate", it will bar Supreme Court review of the state court judgment. The question, rather, is whether a procedural default that would preclude the Supreme Court from exercising appellate jurisdiction should also preclude the exercise of federal habeas jurisdiction. In dealing with this question, the Supreme Court has shifted ground more than once.

Statutory Purpose and the Legal Process School:

When Clearfield was decided, the approach of interpreting statutes to fulfill the overall statutory purpose was superseding the "plain meaning" approach as the dominant approach to interpretation. Courts applying the Legal Process approach could supply apparently omitted statutory terms by asking, in light of the available contextual evidence, how a "reasonable legislator" would have implemented the purposes underlying the statute. United States v. American Trucking Ass'ns, 310 U.S. 534, 543-44 (1940), the Court announced that when the literal or semantic meaning of a statute was " 'plainly at variance with the policy of the legislation as a whole' ", the Court's duty was to "follow[ ] that purpose, rather than the literal words. Hart and Sacks wrote that "[t]he idea of a statute without an intelligible purpose is foreign to the idea of law and inadmissible. They famously emphasized that, in doing so, judges "should assume, unless the contrary unmistakably appears, that the legislature was made up of reasonable persons pursuing reasonable purposes reasonably."

Mootness and appeal

When a case in the federal system becomes moot on appeal, the disposition depends on the nature of the events that mooted the dispute. Munsingwear is the general rule, Bancorp for settlement unless it is obvious they did not do it on purpose.

Jurisdictional Limits on Enforcement Courts

When a federal court has jurisdiction to determine whether a defendant has violated federal law and, if so, to impose a sanction, may Congress limit the issues that may be raised by way of defense? Congress can also limit the types of defenses that can be entered in particular AIII courts, as long as that limitation does not violate some external Constitutional constraint

Wright v. West

When a habeas petition rests on a well-established constitutional principle, but the application of that principle to the facts of the case is uncertain, does the petition rest on a "new rule? Wright v. West: the state characterized Teague as requiring deference to state court determinations of legal principles, and submitted that habeas courts should also defer to "reasonable" state court decisions of "mixed" questions of law and fact—that is, to reasonable applications of established legal rules to the facts as found. All the Justices voted to deny relief on the ground that the prisoner's claim lacked merit.

Inyo County v. Paiute-Shoshone Indians (2003)

a Native American tribe sought injunctive and declaratory relief under § 1983 against a county and certain county officials. The complaint asserted that because of the tribe's sovereign status, it was immune from the processes of the county—in particular, from execution of search warrants against the tribe and tribal property. The Supreme Court assumed without deciding "that Native American tribes, like States of the Union, are not subject to suit under § 1983". in the situation here presented [in which the tribe was not complaining of any conduct that would have violated the federal rights of a private individual or entity but rather was objecting to an interference with its sovereign status], the Tribe does not qualify as a 'person' who may sue under § 1983".

Can States consent via their conduct in litigation?

When a state files suit in federal court, it necessarily waives its sovereign immunity from the court's jurisdiction to determine the validity of its claims and of any defenses that might be asserted against those claims. Removal: it is not always the case that removal constitutes a waiver, but where the state does not have immunity in a state court proceeding and they agree to remove then that constitutes a valid waiver. Lapides v. Board of Regents (2002): Lapides had filed a state court damages action against state officials and the Board (a state agency), complaining of violations of both federal and state law. All defendants joined in removing the case to federal court, which dismissed the action against the individual defendants on grounds of qualified immunity and held that by removing the case to federal court, the state agency had waived any Eleventh Amendment immunity with respect to the claims against it. In the context of state-law claims, in respect to which the state has explicitly waived immunity from state-court proceedings", the "State's act of removing a lawsuit from state court to federal court" constitutes a waiver of any claim of Eleventh Amendment immunity. The "motive" for the removal was irrelevant, as was the question whether the state's attorney general, who had been responsible for the removal and who had authority to represent the state in civil litigation, had the further authority under state law to waive the state's Eleventh Amendment immunity. Counterclaims: The circuits are divided on whether a state's voluntary appearance as a plaintiff in federal court waives the state's sovereign immunity with respect to compulsory (or permissive) counterclaims, and if so whether the relief sought must be of the same kind and not in an amount greater than that sought by the state

Individual Officers: Personal Capacity Suits.

When damages are sought from a governmental official, the officer is ordinarily sued in a "personal" or "individual" capacity. This designation indicates that any judgment will be assessed against the officer personally, rather than against the government employer. The official not the government owes judgments and attorney's fees- not necessarily factually true and they have indemnified. The Eleventh Amendment is inapplicable, since the relief does not directly affect the state. But official immunity doctrine might limit relief- qualified immunity and absolute immunity Tenney v. Brandhove (1951) it has been clear that officials sued under § 1983 in their personal capacity may avail themselves of "official immunity" doctrines—which are wholly distinct from the sovereign immunity doctrines. NO SUPERVISORY LIABILITY -IQBAL

immediate review is precluded by the absence of a final judgment in the state court

When immediate review is precluded by the absence of a final judgment in the state court, the aggrieved party may seek leave from the Supreme Court to file a petition for mandamus. Deen v. Hickman (1958): the Supreme Court granted leave to file after determining that the Texas courts were treating as open an issue it considered foreclosed by its prior decision. The writ itself was not issued, however, since the Court assumed that the Texas court "will of course conform to the disposition we now make" General Atomic Co. v. Felter (1978): The Court determined that a state trial court "has again done precisely what we held [in a prior decision] that it lacked the power to do". The opinion invoked the principle that "a lower court" that fails to "give full effect" to the Court's mandate may be controlled by writ of mandamus

As-Applied Challenges

When the Court is looking at an as applied challenge they are not asking whether each case under the statute would abrogate sovereign immunity, just in particular instances -> as applied to this particular type of claim what has Congress done. This makes the action by Congress much smaller and easier to meet in terms of proportionality. Tennessee v. Lane (2004): rejected (5-4) a motion to dismiss a damages claim against Tennessee by two paraplegics, one a criminal defendant and the other a court reporter, alleging a denial of physical access to the state's courts in violation of Title II of the Americans with Disabilities Act (ADA) Justice Stevens found ample evidence of "pervasive unequal treatment" of the disabled in the administration of both state and local services and programs generally, and with respect to access to courthouses in particular In addition, it was significant that the constitutional claim asserted—the due process right of access to courts—triggered an elevated standard of review; in this respect the case was analogous to the Court's treatment of sex-based classifications in Hibb.-> access to court warrants heightened scrutiny. United States v. Georgia (2006): a paraplegic prison inmate sought damages under Title II of the ADA based on conduct that allegedly included violations of his right to be free from cruel and unusual punishment. A unanimous Court, per Justice Scalia, ruled that the allegations of constitutional violations differentiated this from "our other cases" involving Congress' § 5 power. According to Justice Scalia, "no one doubts" Congress' power under § 5 to create "private remedies against the States for actual violations of the Constitution."

The Obligation of State Courts to Enforce Federal Law

When the concurrent-jurisdiction presumption is not rebutted, state courts are typically obligated to hear federal claims. For more than a century, the Supreme Court has ruled that state courts may not discriminate against federal causes of action. McKnett v. St. Louis & S.F. Ry.: FELA case, where an Alabama statute for the first time opened the state courts to suits against foreign corporations based on an out-of-state accident, but only for suits under the laws of sister states, not for those under federal law. The Court held that the state statute unlawfully discriminated against federal rights: "While Congress has not attempted to compel states to provide courts for the enforcement of the [FELA], the Federal Constitution prohibits state courts of general jurisdiction from refusing to do so solely because the suit is brought under a federal law. * * * A state may not discriminate against rights arising under federal laws.

Jurisdiction Under § 1331 Based on the Presence of a Federal Element: What to look for:

Where state law creates the right of action usually no federal question jurisdiction under 1331 unless a federal question is 1) Necessarily raised (look at the elements of the claim); 2) Actually disputed; 3) Substantial (Smith); 4) Capable of resolution in federal court without disrupting the federal state balance (Merrell Dow)

Trafficante v. Metropolitan Life Ins. (1972)

White and African American tenants were determined to have standing under §810 of the Civil Rights Act, to seek injunctive relief and damages from landlord for discriminating against non-white applicants. Ps claimed damages for: 1) Lost social benefit of living in integrated community, 2) Lost business and professional advantages, and 3) Embarrassment and economic injury from being "stigmatized" as residents of "white ghetto". The court found standing, stressing the role of private attorney generals + Statute authorized grievance by "any person" aggrieved, defined as any person who was injured by discriminatory housing practice

Ex Parte McCardle:

William McCardle (defendant), a newspaper editor in Vicksburg, Mississippi, was arrested by federal government officials after he wrote a series of newspaper articles that were highly critical of the post-Civil War Reconstruction and resulting military rule of the South. The federal government justified McCardle's arrest on the ground that he violated several provisions of the Reconstruction Acts. McCardle sought a writ of habeas corpus from a federal court in Mississippi, but was ultimately unsuccessful in challenging his arrest. McCardle then sought appellate review of his habeas corpus petition in the United States Supreme Court, relying on an 1867 congressional statute that permitted the Supreme Court to have appellate jurisdiction over such matters. However, while the case was pending in the Supreme Court, Congress passed a new law repealing the part of the 1867 statute that permitted Supreme Court appellate review of writs of habeas corpus.

Finality of Judgments

Within case or controversy requirement of Art. III is the Court's insistence that federal courts have the capacity to enter final judgments not amendable by the other branches-> Congress may not vest review of AIII courts' decisions in Executive Branch or Legislative Branch officials. This is pretty strongly related to the advisory opinion component.

Can Marbury sue for his commission in court?

Yes Is it true - essence of civil liberty is that individual right to claim the protection of the laws whenever he receives an injury. The Government of the United States has been emphatically termed a government of laws, and not of men. It will certainly cease to deserve this high appellation if the laws furnish no remedy for the violation of a vested legal right. Nor is this a mere political act for which an individual has no remedy. That there is such cases is not to be questioned. But it depends on the nature of the act. If one of the heads of departments commits any illegal act under colour of his office by which an individual sustains an injury, it cannot be pretended that his office alone exempts him from being sued in the ordinary mode of proceeding, and being compelled to obey the judgment of the law.

Does Marbury have a right to receive the commission?

Yes- it was final when signed and sealed: since his commission was signed by the President and sealed by the Secretary of State, was appointed, and as the law creating the office gave the officer a right to hold for five years independent of the Executive, the appointment was not revocable, but vested in the officer legal rights which are protected by the laws of his country.

Three corollaries follow from the proposition that federal law is part of the law of the land in the state

[1] A state court may not deny a federal right, when the parties and controversy are properly before it, without a valid excuse. [2] An excuse that is inconsistent with or violates federal law is not a valid excuse: the Supremacy Clause forbids state courts to disassociate themselves from federal law b/c of disagreement with its content or a refusal to recognize the superior authority of its source. [3] When a state court refuses jurisdiction b/c of a neutral state rule on the administration of the courts, we must act with utmost caution before deciding that it's obligated to entertain the claim.

Daniels v. Allen:

a companion case to Brown v. Allen two prisoners had been sentenced to death for murder. At their trial they raised federal claims (concerning jury discrimination and the introduction of coerced confessions) similar to those raised in Brown v. Allen. But the North Carolina Supreme Court refused to consider the merits of their appeals, because their lawyer had been tardy in serving the "statement of the case on appeal" on the prosecutor by one day. Going to honor state court procedural requirements - we have equally strict ones ourselves. Justice Reed's opinion was not entirely clear whether the denial of relief rested on waiver, failure to exhaust state remedies, or the presence of an adequate state ground. If a state process is still available for raising a non-exhausted claim, The petitioner is required to exhaust that state remedy before seeking federal habeas relief on that claim If the state process is no longer available for a claim that the petitioner did not exhaust,-Then the question of procedural default arises A failure to use a state's available remedy, in the absence of some interference or incapacity * * * bars federal habeas corpus. The statute requires that the applicant exhaust available state remedies. To show that the time has passed for appeal is not enough to empower the Federal District Court to issue the writ.

Robinson v. California,

a divided Court held in a novel decision that a law making drug addiction a crime constituted "cruel and unusual punishment" in violation of the Eighth and Fourteenth Amendments. Subsequently, the state informed the Court that the defendant in the case had died ten days before the appeal to the Supreme Court was taken, and it petitioned for rehearing and abatement of the judgment. The petition was denied without opinion. Three justices dissented, arguing that the judgment should be vacated as moot.

Chick Kam Choo v. Exxon Corp. (1988)

a federal district court in Texas had dismissed plaintiff's wrongful death action, finding that (i) choice of law doctrine called for application of the law of Singapore rather than of Texas, and (ii) forum non conveniens called for dismissal of the suit so long as the defendants submitted to jurisdiction in Singapore. The plaintiff then filed suit in Texas state court, asserting claims under the laws of Texas and Singapore. The Supreme Court ruled that § 2283 did not preclude the injunction insofar as it barred re-litigation of the Texas law claim, which the federal court had previously held not to be actionable when it determined that the law of Singapore applied. But the Court overturned the injunction insofar as it barred state court litigation of the claim based on the law of Singapore: because federal and state forum non conveniens law might differ, the state court would not necessarily be asked to relitigate the federal court's forum non conveniens ruling

Empire Healthchoice Assurance, Inc. v. McVeigh

a federal statute authorized creation of a health insurance plan for federal employees. The government negotiated a master contract with insurance carriers. The agreement between one of the insurers and its enrollees required an enrollee who obtains a tort recovery to reimburse the insurer for payments made for the enrollee's medical care. In this case, after an enrollee's estate obtained a settlement of a state court tort action against a third party, the insurer brought a federal court action to enforce its right to reimbursement. The court said there is no jurisdiction- Grable emphasized that it takes more than a federal element 'to open the "arising under" door.' This case cannot be squeezed into the slim category Grable exemplifies" What law governed the reimbursement claim? Argument that there should be federal common law, but the court holds that state law applies because there is no reason to displace state law with federal law. If it is federal common law but they adopt state law is that giving jurisdiction? Yes More prevalent approach- is there a reason to displace state law? "This case is poles apart from Grable." The instant claim, by contrast, is "fact-bound and situation specific." The state court in which the personal injury suit was lodged is competent to apply federal law to the extent it is relevant and would seem best positioned

Maine v. Alden

a group of Maine probation officers filed suit against the state in a Maine court, alleging that the state had violated the overtime provisions of the Fair Labor Standards Act of 1938 (FLSA), et seq., and seeking money damages. (The plaintiffs had initially filed their action in federal district court; they turned to the state court only after the federal court, following the Supreme Court's decision in Seminole Tribe, had dismissed their case.) The state responded to the state court action by invoking its sovereign immunity from suit. (It did not dispute Alden's showing that Maine law allows suits against the state by state employees for wage claims based on state law even though it denies those same employees the ability to enforce wage claims based on federal law. The sole difference is that state law does not require overtime pay, while the FLSA does.) Congress cannot require states to be sued in their own courts. Justice Kennedy began by characterizing the immunity claimed by the state as an incident of the constitutional plan: Eleventh Amendment immunity' is convenient shorthand but something of a misnomer, for the sovereign immunity of the States neither derives from, nor is limited by, the terms of the Eleventh Amendment-> the state's immunity is based on the general structure of the Constitution, its history, and the authoritative interpretations by this Court; not tied at all in text of the 11th amendment. The text and history of the Eleventh Amendment also suggest that Congress acted not to change but to restore the original constitutional design.

Field pre-emption:

a particular area of law is reserved for federal activity alone. The difficult questions with respect to field preemption tend to be about the precise boundaries of the field. This can occur in two different situations: Federal government has occupied the field- pervasive comprehensive scheme of regulation no room for state legislation Federal interest is so dominant that state action is per se undesirable where Congress has preempted a field in which areas have been traditionally occupied by the states, congressional intent to supersede state laws "must be clear and manifest" Court has not hesitated to draw an inference of field pre-emption where it is supported by the federal statutory and regulatory schemes though.

McKesson Corp. v. Division of ABT (1990)

a state court action seeking a refund of state taxes paid under a discriminatory tax that was held to violate the dormant Commerce Clause. The state court had enjoined future enforcement of the tax but had refused, on the basis of "equitable considerations", to award a refund of taxes previously paid. A unanimous Supreme Court held that if (as in this case) a state requires taxpayers to pay first and obtain review of the tax's validity later, the Due Process Clause requires the state to afford a meaningful opportunity to secure post payment relief. A unanimous Supreme Court held that if (as in this case) a state requires taxpayers to pay first and obtain review of the tax's validity later, the Due Process Clause requires the state to afford a meaningful opportunity to secure postpayment relief. The Court held that the state must either refund to the complaining taxpayer the constitutionally excessive portion of the taxes paid, or (to the extent consistent with other constitutional restrictions) assess and collect back taxes from the taxpayer's competitors to eliminate the discrimination.-> If state requires taxpayers to pay first and challenge later, DP requires meaningful opportunity for post payment relief.

Hope v. Pelzer (2002)

a state prisoner brought an action against state officers under § 1983, alleging violations of his Eighth Amendment right not to be subjected to cruel and unusual punishment. More specifically, the plaintiff asserted that on two occasions, while working on a chain gang, he had been handcuffed to a hitching post for disruptive conduct. He further alleged that on the first occasion, he was handcuffed above shoulder height for a two-hour period during which the cuffs cut into his wrists, causing pain; on the second, he was made to take off his shirt, then hitched for seven hours in the hot sun, and was given only one or two water breaks and no bathroom breaks. The lower courts thought the action barred by qualified immunity because the facts of Hope's situation were not "materially similar" to those of previous cases. The court held that a plaintiff need not show "that the very action in question has previously been held unlawful" so long as "in the light of pre-existing law, the unlawfulness [was] apparent".

JI Case Co. v. Borak (1964)

a unanimous Court upheld a private party's right to sue under § 14(a) of the Securities Exchange Act of 1934, which prohibits fraud in the solicitation of proxy material. The Court adopted a broad, purposive rationale for embracing an implied right of action. Noting that "it is the duty of the courts to be alert to provide such remedies as are necessary to make effective the congressional purpose", the Court concluded that "[w]hile [§ 14(a)] makes no specific reference to a private right of action, among its chief purposes is 'the protection of investors,' which certainly implies the availability of judicial relief where necessary to achieve that result."

Amalgamated Clothing Workers v. Richman Brothers (1955)

a union sought to enjoin a state-court suit as preempted by the NLRB's exclusive jurisdiction. The Supreme Court affirmed the district court's refusal to issue an injunction: because no statute authorized the union to file the federal court suit, the federal injunction was not ancillary to an independently-based, ongoing proceeding; and the Court refused to permit an injunction merely because state court jurisdiction had allegedly been preempted If the union is right then the state court does not have jurisdiction - The court is not saying that the state court got control of the thing first- instead they are saying yes they lack jurisdiction, but your theory is that only the NLRB has jurisdiction, so federal court also lacks jurisdiction

United States v. Standard Oil Co. (1947),

after a Standard Oil truck injured a soldier, the government sued the company. The somewhat novel tort theory sought damages for the military's loss of the soldier's services while he was convalescing and for its expenses for his hospitalization—by analogy to "the master's rights of recovery for loss of the services of his servant or apprentice; the husband's similar action for interference with the marital relation, including loss of consortium as well as the wife's services; and the parent's right to indemnity for loss of a child's services, including his action for a daughter's seduction". The Supreme Court held that the United States was not entitled to recover applying the two step approach. - Is this governed by federal law? Yes- Relying on Clearfield and similar cases, Justice Rutledge's opinion began by declaring that "the creation or negation of such a liability is not a matter to be determined by state law". What should the principle be? When it came to crafting a federal rule, however, the Court concluded that the question was one of "fiscal policy" for determination by Congress rather than by a federal court-> too novel

Milliken v. Bradley (1977)

after the Supreme Court had disapproved an interdistrict busing remedy to desegregate the Detroit schools, the district court ordered the provision of remedial education for pupils and in-service training for teachers and administrators, as well as the hiring of more counselors. The state, which shared responsibility for the segregation, was ordered to pay half the cost of these programs; though the order ran only against state officials, it contemplated payment from the state treasury. The Supreme Court unanimously held that the decree "fits squarely within the prospective-compliance exception reaffirmed by Edelman.

Osborn v. Bank of the United States (1824)

after the federal circuit court enjoined state officials from collecting an unconstitutional tax from the plaintiff Bank, an official seized $100,000 from a Bank office. The Supreme Court upheld a second decree ordering one official to return the $98,000 held in his possession but credited to the state, and ordering two others to repay the remaining $2,000 (the location of which was not discussed in the opinion), all over defendants' objection that the order violated the Eleventh Amendment. In all cases where jurisdiction depends on the party, it is the party named in the record. Consequently, the 11th amendment is, of necessity, limited to those suits in which a State is a party on the record.

Reich v. Collins (1994)

also a state court action for a state tax refund, the Supreme Court unanimously stated the constitutional obligation more unqualifiedly. State can choose between pre- and post-deprivation remedy, but due process requires one or the other. State cannot change course mid-stream (aka no "bait and switch") It cited, inter alia, McKesson, Bennett, and Ward, as support for the proposition that due process requires that a state provide a "clear and certain" remedy for taxes collected in violation of federal law, and that while the state may choose between predeprivation and postdeprivation remedies, it must provide one or the other. Thus, exaction of taxes in violation of a federal statute or the Constitution, by compulsion, violated the Fourteenth Amendment.

Lauf v. E.G. Shinner & Co

an employer challenged the constitutionality of the Norris-LaGuardia Act insofar as it sharply restricted, but did not completely abolish, the authority of a federal court to enjoin allegedly unlawful picketing and related activities. As the Supreme Court noted, the substantive rights of the parties were governed by state law and federal jurisdiction rested on diversity. There can be no question," the Court said, "of the power of Congress thus to define and limit the jurisdiction of the inferior courts of the United States. Norris-LaGuardia Act of 1932, 29 U.S.C. §§ 101-115, which sharply restricts the jurisdiction of "courts of the United States" to issue temporary or permanent injunctions in "a case involving or growing out of a labor dispute". The term "court of the United States" is defined to mean "any court of the United States whose jurisdiction has been or may be conferred or defined or limited by Act of Congress" Restricts jurisdiction of Court of the United States. - Sheldon v. Sill doctrine- this seems to be a step beyond it because the employers are going to raise federal constitutional rights.

Long (Stevens Dissent)

argues that the Court should not review state-court judgments that uphold claims of federal right The nature of the case before us hardly compels a departure from tradition. These are not cases in which an American citizen has been deprived of a right secured by the United States Constitution or a federal statute. I believe that in reviewing the decisions of state courts, the primary role of this Court is to make sure that persons who seek to vindicate federal rights have been fairly heard. That belief resonates with statements in many of our prior cases

Wilkie v. Robbins:

claims brought by a rancher (Robbins) against federal Bureau of Land Management (BLM) officials. Robbins alleged that after he refused to re-grant an easement that the BLM had forfeited by failing to record it, the government retaliated against his property rights through selective enforcement of federal law and through various forms of harassment against him. After noting the existence of "administrative" and "ultimately judicial" remedies for the government's alleged wrongs, the Court stressed the "difficulty in defining a workable cause of action" to describe Robbins' claim-> judicial manageability. The Court in Wilkie v. Robbin characterized its approach to Bivens cases as one of "weighing reasons for and against the creation of a new cause of action, the way common law judges have always done.

Causes of Action Created by Federal Common Law

it is now settled that causes of action that are properly "implied" from federal statutes or from the federal Constitution, or that are otherwise recognized by federal common law, do "arise under" federal law within the meaning of § 1331. Illinois v. City of Milwaukee (1972): There, the Court denied a motion for leave to file an original action in the Supreme Court, seeking to enjoin pollution of Lake Michigan, on the ground that a district court would be a more appropriate forum. The Court determined that pollution of navigable interstate waters is governed by federal common law (as well as by statutes) and that the district courts had jurisdiction because actions asserting such common law rights arise under the "laws" of the United States.

Sandoval Breakdown

court instead treats 601 and 602 as distinct and says this must come through 602. Hand clearing- the question now is, does 602 alone create a cause of action? Like substantive federal law itself, private rights of action to enforce federal law must be created by Congress. The judicial task is to interpret the statute Congress has passed to determine whether it displays an intent to create not just a private right but also a private remedy. Statutory intent on this latter point is determinative. Without it, a cause of action does not exist and courts may not create one, no matter how desirable that might be as a policy matter, or how compatible with the statute. -> Swaps the intent from Cannon! Now looking to see if the intent affirmatively suggests that there was an intent rather than if it affirmatively rejected it. Neither as originally enacted nor as later amended does Title VI display an intent to create a freestanding private right of action to enforce regulations promulgated under § 602. We therefore hold that no such right of action exists. Nor do we agree with the Government that our cases interpreting statutes enacted prior to Cort v. Ash have given "dispositive weight" to the "expectations" that the enacting Congress had formed "in light of the 'contemporary legal context.'-> No longer consider the landscape outside of just providing context to the interpretation

American Ins. Ass'n v. Garamendi (2003)

dealt with executive agreement in 2000 between President Clinton and German Chancellor Schröder, in which Germany agreed to create a foundation to compensate victims of Nazi persecution, funded with 10 billion Deutsche marks contributed equally by the German Government and German companies. In agreeing to create the fund, Germany bargained for a measure of security against American lawsuits. Accordingly, the so-called Foundation Agreement called for the U.S. Government to submit, in any Holocaust-era claim against a German company in an American court, a statement that "it would be in the foreign policy interests of the United States for the Foundation to be the exclusive forum and remedy for the resolution of all asserted claims against German companies" arising from the Nazi regime and World War II and "that U.S. policy interests favor dismissal on any valid legal ground." Held that certain executive agreements with Germany and other European nations preempted California's Holocaust Victim Insurance Relief Act (HVIRA)->even executive agreements can preempt.

United States v. Mendoza-Lopez

distinguished Yakus in part on the basis that it rested on the exigencies of wartime, and proceeded to hold that an enforcement court could not predicate a finding of criminal violation on a previous administrative determination where there had been no meaningful opportunity to seek judicial review of the administrative ruling. Says Yakus was a war time decision. Needs a chance to challenge the determination: At a minimum the result of an administrative proceeding may not be used as a conclusive element of a criminal offense where the judicial review that legitimated such a practice in the first instance has effectively been denied

Seminole Tribe and Union Gas

does congress have the power to do so?- whether Congress has acted "pursuant to a valid exercise of power." Was the Act in question passed pursuant to a constitutional provision granting Congress the power to abrogate? The Court begins by clearing its hand- it overrules Union Gas thus finding that the Interstate Commerce Clause does not authorize Congress to abrogate state sovereign immunity. This appeared necessary for their ultimate conclusion- A lot of reason to think that Congress's power to legislate and abrogate under Indian Commerce Clause would be greater- We agree with the petitioner that the plurality opinion in Union Gas allows no principled distinction in favor of the States to be drawn between the Indian Commerce Clause and the Interstate Commerce Clause. What about Stare Decisis? The decision was only seven years old- Really just a plurality so there is no binding precedent - think its wrong. Furthermore, the plurality's rationale also deviated sharply from our established federalism jurisprudence and essentially eviscerated our decision in Hans. In overruling Union Gas today, we reconfirm that the background principle of state sovereign immunity embodied in the Eleventh Amendment is not so ephemeral as to dissipate when the subject of the suit is an area, like the regulation of Indian commerce, that is under the exclusive control of the Federal Government. Even when the Constitution vests in Congress complete law-making authority over a particular area, the Eleventh Amendment prevents congressional authorization of suits by private parties against unconsenting States

External impediment as cause:

essentially just when the Government either deliberately not disclosing Brady material Amadeo v. Zant (1988): while the defendant's direct appeal was pending, an independent voting rights lawsuit uncovered a handwritten memorandum from the District Attorney's Office to the jury commissioners, listing figures for the number of blacks and women to be placed on master jury lists. The document's apparent purpose was to ensure that these groups were under-represented but not so much as to give rise to a prima facie case of discrimination. The Georgia Supreme Court rejected it as untimely. On federal habeas corpus, the Supreme Court found cause for the default because the basis for the claim was "reasonably unknown" to the prisoner's lawyers as a result of "the 'objective factor' of 'some interference by officials' " Strickler v. Greene: the Court extended the "external impediment" concept to the possibly inadvertent withholding of information by the government. In his federal habeas petition, Strickler for the first time claimed that the government had not disclosed exculpatory information, in violation of the Due Process Clause. Justice Stevens' opinion found cause for Strickler's failure to have sought the information either at trial or in a state habeas proceeding. Writing for a unanimous Court on this point, he stressed that the prosecution had maintained at trial an "open files policy" toward discovery.

Fitzpatrick v. Bitzer (1976),

federal court action alleging that Connecticut's retirement plan discriminated against male employees in violation of Title VII of the 1964 Civil Rights Act. The Eleventh Amendment did not bar an award of retroactive retirement benefits and attorney's fees under Title VII, to be paid from the state treasury, as long as "the 'threshold fact of congressional authorization' is clearly present", as it was in this case.-> Abrogation via section 5 of the 14th Amendment is permissible. Justice Rehnquist emphasized that the 1972 amendment was enacted pursuant to § 5 of the Fourteenth Amendment. That Amendment as a whole represented a "shift in the federal- state balance [that] has been carried forward by more recent decisions of this Court," This power shifting emphasis appears necessary- just because the 14A came last in time does not mean that it was meant to alter other portions of the Bill of Rights. However, something fundamental happened in the reorientation of the governments under the Constitution- interpose the government between the state and the people to protect the people. When Congress acts pursuant to § 5, not only is it exercising legislative authority that is plenary within the terms of the constitutional grant, it is exercising that authority under one section of a constitutional Amendment whose other sections by their own terms embody limitations on state authority.

Innocence based on evidence not presented at the trial?

free standing claim of innocence after the fact is not enough without a constitutional violation. Herrera v. Collins,(1993), the evidence at Herrera's murder trial included two eyewitness identifications and a handwritten letter in which Herrera appeared to admit his guilt. Ten years after his conviction and death sentence, his habeas petition contended that newly discovered evidence (affidavits declaring that Herrera's now-dead brother had confessed to the murder) showed that Herrera was actually innocent and argued that his execution would violate the Eighth and Fourteenth Amendments. Claims of actual innocence based on newly discovered evidence have never been held to state a ground for federal habeas relief absent an independent constitutional violation occurring in the underlying state criminal proceeding. The function of habeas review, the Court submitted, was to redress constitutional violations, not to correct factual errors, and review of freestanding innocence claims would severely disrupt the strong state interest in finality. The Court distinguished the claim in Jackson v. Virginia because it (i) established an independent constitutional violation; (ii) could be adjudicated by reviewing the adequacy of the record evidence, without new factfinding; and (iii) asked only if the verdict of guilt was rational, not whether it was correct

Implied Rights of Contribution.

he Court has held that an express statutory right of action does not authorize judges to recognize an implied right of contribution, but that an implied primary right of action does carry with it that further authority. In Texas Industries, Inc. v. Radcliff Materials, Inc., (1981), the Court held that a company sued for conspiring to fix prices under the Sherman Act cannot seek contribution against other alleged participants in the scheme in the absence of express statutory recognition of such a right. In contrast, the Court was willing to fashion a right of contribution in Musick, Peeler & Garrett v. Employers Ins. of Wausau (1993): That suit was based on the implied private right of action, previously recognized by the Court, under § 10(b) of the Securities Exchange Act of 1934 and SEC Rule 10b-5. See Paragraph (1)©. The Court found Texas Industries to be distinguishable, because its central inquiries—whether Congress " 'expressly or by clear implication' envisioned" a right to contribution, or "whether Congress 'intended courts to have the power to alter or supplement the remedies enacted' "—are not helpful in the context of a judicially created private right of action like the one under Rule 10b-5. Rather, in that context, the Court must ask "how the 1934 Congress would have addressed the issue had the 10b-5 action been included as an express provision in the 1934 Act."

NLRB v. Jones & Laughlin Steel Corp.

he Seventh Amendment] has no application to cases where recovery of money damages is an incident to equitable relief even though damages might have been recovered in an action at law. It does not apply where the proceeding is not in the nature of a suit at common law. Several decades later, the Court said that Jones & Laughlin "merely stands for the proposition that the Seventh Amendment is generally inapplicable in administrative proceedings, where jury trials would be incompatible with the whole concept of administrative adjudication and would substantially interfere with the NLRB's role in the statutory scheme"

Complete Pre-Emption:

if the plaintiff's claim, though cast as resting on state law, is "really" a federal claim, removal will be permitted on the ground that the plaintiff should not, by artful pleading, be allowed to negate the defendant's removal rights. Only so far applies to 1) LMRA 2) ERISA and 3) the National Bank Act For traditional pre-emption it will not survive the WPC rule- Plaintiff brings state tort claim with no mention of federal law to which the defendant brings a defense of federal law pre-emption. Look at the plaintiff's claim -defenses alone do not count and ordinarily assertion that a federal law pre-empts a plaintiff's nonfederal claim is merely a defense. Skelly doesn't come into play because that only plays where there is a DJ.

Haywood v. Drown

involved a provision of New York's Correction Law that, responding to concerns about vexatious and frivolous prisoner lawsuits, stripped New York's trial courts of jurisdiction over suits by prisoners seeking damages from state corrections officials. For damages actions against corrections officials, the statute substituted an action directly against the state in the state's court of claims, although this remedy was hedged with substantive and procedural limitations. The state's jurisdiction stripping law applied equally to federal and state law claims, and so when Haywood, an inmate, brought a federal civil rights action against prison officials, seeking damages under 42 U.S.C. § 1983, the state trial court dismissed his action. A jurisdictional rule cannot be used as a device to undermine federal law, no matter how evenhanded it may appear. Justice Stevens' majority opinion stressed that states may in no case withhold jurisdiction over federal claims based on substantive disagreement with federal law, and stated: "It is principally on this basis that [the New York law] violates the Supremacy Clause."

Express Pre-emption

ongress explicitly states that certain state laws are to be preempted. Such a preemption clause will typically provide that no state shall maintain any requirement not identical to the requirements of federal law, or words to that effect. An explicit preemption clause will generally prevent state authority, in any form, from imposing consequences not identical to those under federal law E.g.: The National Traffic and Motor Vehicle Safety Act of 1966 - Its preemption clause provides that once a federal motor vehicle safety standard is in effect, no State or political subdivision "shall have any authority either to establish, or to continue in effect, with respect to any motor vehicle or item of motor vehicle equipment[,] any safety standard applicable to the same aspect of performance of such vehicle or item of equipment which is not identical to the Federal standard."

Grove Unified Sch. Dist. v. Newdow (2004)

ourt cited avoiding interference with family relations structured by state law as prudential grounds for denying a divorced father's standing to challenge his daughter's school's practice of saying the pledge of allegiance "under God" Father claimed that his interfered with his ability to teach his daughter atheism Mother opposed the action, and under CA law, Newdow could not bring the action as his daughter's "next friend," but only on his own behalf - Court emphasized the outer reaches of Newdow's rights and interests as a parent were governed by state law and could be adverse to those of his daughter Analogy to "domestic relations exception," whereby federal courts generally lack jurisdiction to issue divorce, child custody, or alimony decisions.

Fox Film Corp. v. Muller

ox Film Corporation (Fox) (plaintiff) sued Muller (defendant) in state court for breach of two contracts involving Muller's license to show movies belonging to Fox. Muller alleged the contracts were invalid under the federal Sherman Anti-trust Act and, alternatively, that the arbitration clause of the contracts was invalid and voided the contracts. Fox was a defendant in an earlier case involving contracts substantially similar to Muller's contracts in which the arbitration clause was held to be invalid. See United States v. Paramount Famous Lasky Corp., 34 F.2d 984, aff'd 282 U.S. 30. The lower state court dismissed the case, finding first that the arbitration clause was invalid and voided the contract, and second, that the contracts violated the Sherman Anti-trust Act. The Supreme Court of Minnesota affirmed on the grounds that the arbitration clause was invalid and tainted the entire contract, relying on United States v. Paramount. After reviewing the opinion and decree of the federal district court in the Paramount case, the lower court reached the conclusion that the holding of the federal court was that the entire contract was illegal; and upon that view and upon what it conceived to be the weight of authority, held the arbitration plan was inseparable from the other provisions of the contract. When the presence of an adequate and independent state ground as deprives the court of jurisdiction to review the state court judgment, the proper disposition is to dismiss for lack of jurisdiction, rather than (as Murdock suggested) to affirm It is now an accepted principle that the Supreme Court will not review a federal question when the state court's decision of a state law issue precludes the Court from altering the outcome below, no matter how the Court might resolve the federal question.

Parsons Steel, Inc. v. First Alabama Bank

plaintiffs sued the bank in separate actions in federal and state court. The federal action came to judgment first, with the bank prevailing. The bank's assertion in state court of res judicata defenses, based on the federal judgment, was rejected, leading to a $4 million state court verdict against the bank. The bank returned to federal court and obtained an injunction against the state court proceeding on the ground that the state court claims could have been raised as pendent claims in the prior federal action and thus should have been held by the state court to have been precluded by the federal judgment. Timing matters - Relitigation exception is limited to situations in which state court has not yet ruled on the merits of the res judicata issue. Once the state rejected the claim of res judicata, federal courts must turn to state law to determine the preclusive effect of the state court's decision

Beneficial Nat'l Bank v. Anderson:

plaintiffs' state court action alleged that a bank chartered under the National Bank Act had violated state usury law. The Court, with Justice Stevens writing, upheld removal on the complete preemption theory. the National Bank Act "unquestionably pre-empts any [state-law] rule that would treat * * * as usurious" rates that were lawful under the Act. But a federal preemption defense would not alone justify removal; "[o]nly if Congress intended [the pertinent provision of the federal Act] to provide the exclusive cause of action for usury claims against national banks would the statute be comparable to the provisions" in Avco Insofar as Avco and Metropolitan Life included language referring to the extraordinary preemptive effect of a statute or to an intent in the legislative history to permit removal, Beneficial National Bank seems to make clear that neither is required to invoke complete preemption.

State Law as a Distinct Basis for Relief.

ppose, for example, that a taxpayer contends, in a state-court refund action, that a state tax violates both the federal and state constitutions. (a) If the state supreme court invalidates the tax under the state constitution, without reaching the federal question, the Supreme Court lacks jurisdiction to review. (b) If the state supreme court holds the tax invalid under the U.S. Constitution and independently invalid under the state constitution, there is no jurisdiction to review. Do state courts have an obligation to avoid decision of the federal constitutional issue if possible? (c) If the state court holds the tax valid under both constitutions, the state ground cannot independently support the judgment, and the federal-law ruling is plainly subject to review. (d) If the state court invalidates the tax under the federal Constitution without reaching the state-law issue, the settled rule is that the judgment is reviewable: Supreme Court jurisdiction depends on the state court's actual grounds of decision rather than on possible grounds.

Dissent: Breyer

pragmatic approach: [The relevant] factors include (1) the nature of the claim to be adjudicated; (2) the nature of the non-Article III tribunal; (3) the extent to which Article III courts exercise control over the proceeding; (4) the presence or absence of the parties' consent; and (5) the nature and importance of the legislative purpose served by the grant of adjudicatory authority to a tribunal with judges who lack Article III's tenure and compensation protections.

Yakus v. United States

presented the question whether a defendant prosecuted for a violation of the Price Control Act may defend the criminal case by challenging the validity of the Act or of a regulation. The process for this challenge was brought before article III judges in the ECA but it was an entirely separate process. It is okay to set up separate process to challenge a regulation and criminalize violation and split the trial. The court validates the framework. Could raise the issue that the entire statutory scheme is unconstitutional- DP or trial by jury. On the due process issue, Chief Justice Stone's opinion for the Court treated as the central question whether the procedure for review in the ECA "affords to those affected a reasonable opportunity to be heard and present evidence"

Shaw v. Delta Air Lines, Inc.

private plaintiffs, alleging that ERISA preempted provisions of New York's Human Rights Law and Disability Benefits Law, sued state officials for declaratory and injunctive relief. In upholding federal question jurisdiction, the Court said "It is beyond dispute that federal courts have jurisdiction over suits to enjoin state officials from interfering with federal rights. A plaintiff who seeks injunctive relief from state regulation, on the ground that such regulation is pre-empted by a federal statute which, by virtue of the Supremacy Clause of the Constitution, must prevail, thus presents a federal question which the federal courts have jurisdiction under 28 U.S.C. § 1331 to resolve.

Chisholm v. Georgia Justice Iredell's lone dissent

proceeded from the premise that the Supreme Court could exercise only that jurisdiction conferred by Congress. Here, he argued it was not because he contended that the First Judiciary Act's grant of original jurisdiction should be interpreted in light of common law principles. English law would have permitted an action like Chisholm's only by petition of right, with the sovereign's consent. Is this relevant? It is a feudal doctrine - not applicable in this context? No one can contest that English common law is embedded in American law, but other justices say this departs where it conflicts with the Constitution.

Mitchum v. Foster Breakdown

proceedings except under special circumstances. If you could never get a federal court to enjoin a state court via 1983 then we would not need to go through any of that. Plus there are some exceptions to Younger they would need to overturn. We proceed, then, upon the understanding that in determining whether § 1983 comes within the "expressly authorized" exception of the anti-injunction statute, the criteria to be applied are those reflected in the Court's decisions prior to Toucey.->The test, rather, is whether an Act of Congress, clearly creating a federal right or remedy enforceable in a federal court of equity, could be given its intended scope only by the stay of a state court proceeding. When determining whether §1983 comes within the "expressly authorized" exception, decisions to be applied are those before Toucey, which §2283 overruled Legislative history of §1983 makes clear that Congress conceived that it was altering the relationship between the states and the federal gov't regarding federally protected rights. It was concerned that the states could not protect these rights The purpose of §1983 was to interpose federal courts between the states and the people as guardians of the people's federal rights The whole point of reconstruction was bad behavior by states - put the federal gov between the state and people including state courts who were among the bad actors. Court also focuses on the language of the statute that says that it is enforceable in a federal court of equity.

Created by 42 USC 1983

provides an express right of action to obtain relief against action taken under color of state law in violation of federal statutory or constitutional rights, can embrace actions by a federal rightholder contending that state or local regulation is preempted by federal law. It appears that § 1983 also contemplates the award of declaratory relief, for that statute states that an injunction against a judicial officer shall not be granted "unless a declaratory decree was violated or declaratory relief was unavailable." Insofar as § 1983 creates an express federal right of action to declaratory relief, the existence of federal jurisdiction over that right of action seems clear. Golden State Transit Corp. v. City of Los Angeles (1989): the Court ruled that 42 U.S.C. § 1983, which provides an express right of action to obtain relief against action taken under color of state law in violation of federal statutory or constitutional rights, can embrace actions by a federal rightholder contending that state or local regulation is preempted by federal law. Even after the Golden State decision, federal courts (including the Supreme Court) have continued to uphold their jurisdiction under § 1331 over actions to enjoin preempted state or local regulations without relying on § 1983.

House v. Bell (2006)

ruled that the prisoner was entitled to litigate claims of ineffective assistance of counsel and failure to disclose exculpatory evidence that would otherwise have been foreclosed because of his procedural default in state court. House's conviction of capital murder rested on circumstantial evidence, including testimony that a semen sample taken from the victim was consistent with House's semen and that blood stains on House's pants were consistent with the blood of the victim. In his federal habeas corpus proceeding, House presented new evidence (i) establishing that the semen was that of the victim's husband, (ii) suggesting that the victim's blood might have been spilled from autopsy samples onto House's pants before the pants were tested, and (iii) indicating that the victim's husband had a history of abuse as well as the opportunity to have caused his wife's death, and had told two people that he had accidentally done so. They found that House was entitled to relief if he could show that "it is more likely than not that no reasonable juror would have convicted". The Court concluded that House's was one of the "extraordinary" cases that meets this "demanding" standard. Rejected freestanding claim though. Fails the test for innocence simplicter - this is a higher standard

Bank Markazi

statute provides a new standard clarifying that if Iran owns certain assets the victims of Iran Sponsored terrorist attacks will be permitted to execute against those assets. How to apply existing law. The application of the facts are dictated but it is okay because the court is look at what Congress is saying but they changed the law and that is okay even if they want it applied retroactively to cases. Congress can't say in Smith v. Jones, Smith wins. But Congress can change law and make it retroactive to cases before the Court.

Floyd v. City of NY

stop and frisk litigation- Named P's allege on behalf of themselves and a putative class that Ds have engaged in a policy and or practice of unlawfully stopping and frisking people in violation of their 4th amendment right. To be free from unlawful search and seizure and their 14th amendment right to freedom from discrimination on the basis of race. District court found way around this and held that there was standing - Risk of injury was not based on a string of unlikely contingencies Defining the injury is crucial here - change its operation to reconceptualize the injury. Distinguished Lyons: This is a class, Lyons was not Named P was stopped and frisked several times in preceding years. Probabilistic evidence that risk of future injury would occurs, given frequency of stops; creates a likelihood of future injury sufficient to address any standing concerns Cannot avoid injury by following the law, b/c his stops did not depend on breaking the law

General Oil Co. v. Crain:

suit was brought in Tennessee state court to enjoin a state official from enforcing a tax alleged, inter alia, to burden interstate commerce. The claim that anticipatory injunctive relief was necessary was premised on the heavy penalties for violation, doubts that payments under protest could be recovered, and concern that a multiplicity of refund actions would be necessary in order to obtain complete recovery. The Supreme Court of Tennessee held that the state courts lacked jurisdiction to grant the injunction sought, holding that the suit was one against the state and thus could not be entertained. Think of these as addressing the relationship between thinking there is remedy and the existence of federal court jurisdiction. If there is jurisdiction then there is a right to challenge the unconstitutional act- then SCOTUS jurisdiction. Held: if suit against state officials could be precluded by 11 Amendment and may be forbidden by a state in its own courts, without SCOTUS review, then this could prevent the enforcement of various constitutional provisions "It being ... the right of a party to be protected against a law which violates a constitutional right, ... a decision which denies such protection gives effect to the law, and the decision is reviewable by this court."

Iowa-Des Moines Nat'l Bank v. Bennett (1931)

taxpayers, alleging a denial of equal protection, sought a refund of taxes levied on the plaintiffs' stock at a higher rate than was applied to the shares of competing domestic corporations. Without denying that systematic discrimination existed, the Supreme Court of Iowa affirmed a judgment denying relief, holding that the auditor had violated state law in reducing taxes on the competitors, and that the plaintiffs' remedy was to await (or to initiate proceedings to compel) collection of the higher tax from their competitors. SCOTUS says once the suit is filed in court it is not the role of the government to make the competitor equaled out.- EP violation entitles tax payers to refund in state court where the state has not properly collected the additional tax from competitors.

Cannon Factor 2

the Cort analysis requires consideration of legislative history. What kind of intent? Not needing to show that Congress intended to create right of action, but an explicit intent to deny would control We must recognize, however, that the legislative history of a statute that does not expressly create or deny a private remedy will typically be equally silent or ambiguous on the question. Therefore, in situations such as the present one "in which it is clear that federal law has granted a class of persons certain rights, it is not necessary to show an intention to create a private cause of action, although an explicit purpose to deny such cause of action would be controlling." The court looks at the legislative history and they look at the landscape of when Congress was creating the law. Here the backdrop was that Title VI was already interpreted to create the right of action: In 1972 when Title IX was enacted, the critical language in Title VI had already been construed as creating a private remedy.-> identical language replacing race to sex.

Chicago & Southern Airlines v. Watermen

the Court considered whether an order of the Civil Aeronautics Board denying to one American air carrier and granting to another a certificate of convenience and necessity for an overseas air route was subject to judicial review. The act required the President approve such an order and also allowed some judicial review. Court said that such orders could not be reviewed because they were matters of Presidential discretion as to political matters. President's discretion may not be subjected to judicial review. "Judgements, within the powers vested in courts by the Judiciary Article of the Constitution, may not lawfully be revised, overturned or refused faith and credit by another Department of Government."

Wood v. Georgia

the Court decided an issue that all the Justices acknowledged had not been raised in state court. There, three employees of "adult" establishments had been convicted of distributing obscene materials. Their sentence of probation was conditioned on their making installment payments of substantial fines. When those payments were not made, probation was revoked. The Supreme Court granted review to decide whether imprisonment of a probationer who is unable to make such payments denies equal protection. Noting that the employees had been represented by a lawyer paid by their employer, and that the employer had promised to pay any fines imposed, the Court found that there was a potential conflict of interest that might have denied the employees due process. The Court accordingly remanded the case so that the state courts could determine the nature and implications of any such conflict. The Court has jurisdiction to review plain error unchallenged in the state court when necessary to prevent fundamental unfairness.

Harper v. Virginia Dept. of Taxation (1993)

the Court determined that its decision in Davis v. Michigan Dept. of Treasury 1989)—which held unconstitutional a state tax on federal pension income—should be applied retroactively. VA supreme Court refused to apply Davis to taxes imposed before Davis was decided. Some language in Justice Thomas' majority opinion indicated that retroactive relief must be afforded even in situations in which the new constitutional rule had not already been applied retroactively in a prior case, reasoning that the Court lacks the constitutional authority, in effect, to disregard substantive law when deciding a case or controversy or to treat similarly situated litigants differently by denying a new rule retroactive effect. "When this Court applies a rule of federal law to the parties before it, that rule is the controlling interpretation of federal law and must be given full retroactive effect in all cases still open on direct review and as to all events, regardless of whether such events predate or postdate our announcement of the rule."

Governor of Georgia v. Madrazo (1828)

the Court dismissed a libel in admiralty brought in federal district court by Madrazo against the governor for possession of certain slaves (and the proceeds of the sale of others) seized under a state statute after the slaves had allegedly been brought to this country in violation of federal law. The Court refined the party of record rule holding that "[the demand made upon [the Governor], is not made personally, but officially" because he was sued "not by his name, but by his title". Therefore, "the state itself may be considered as a party on the record". Essentially because the governor was acting in his official capacity, it counted as having the state named the defendant. Ex parte Madrazzo (1833): Madrazo subsequently filed a libel in admiralty against the State of Georgia as an original action in the Supreme Court. In a one paragraph opinion Chief Justice Marshall dismissed the suit- "It is a mere personal suit against a state to recover proceeds in its possession, and in such a case no private person has a right to commence an original suit in this court against a state."

Metropolitan Life Ins. Co. v. Taylor

the Court extended the approach of the Avco decision to federal regulation of employee benefit plans under ERISA. ERISA's legislative history strongly indicated that Congress intended the preemptive sweep of the statute to replicate that of § 301, showing that Congress wished "to make § 502(a)(1)(B) suits brought by participants or beneficiaries federal questions for the purposes of federal court jurisdiction in like manner as § 301 of the [Taft-Hartley Act]."

Barclays Bank, PLC v. Franchise Tax Bd. (1994)

the Court found unmeritorious a federal constitutional challenge to California's method of apportioning taxes on multinational corporations. In rejecting arguments that the challenged method impaired national uniformity in international trade and was likely to provoke international retaliation, the Court stressed that the nuances of foreign policy " 'are much more the province of the Executive Branch and Congress than of this Court' " and that "[t]he judiciary is not vested with power to decide 'how to balance a particular risk of retaliation against the sovereign right of the United States as a whole to let the States tax as they please'

Davis v. Passman (1979)

the Court held that Davis could bring a Bivens action alleging that she had been fired from her job as administrative assistant to Congressman Passman because of her sex, in violation of the "equal protection" component of the Due Process Clause of the Fifth Amendment. Justice Brennan's opinion for the Court emphasized that "unless such rights are to become merely precatory," litigants with "no [other] effective means [to] enforce these rights must be able to invoke the existing jurisdiction of the courts for the protection of their justiciable constitutional rights." Because Passman was no longer in Congress, the Court concluded that "for Davis, as for Bivens, 'it is damages or nothing.'->Equal Protection component of 5th Amendment DP

Alvarez v. Smith

the Court made clear that the Bancorp exception to vacatur would not apply if the circumstances demonstrated that the parties had not arranged their affairs to render the case moot. - First, the Court found it significant that the parties did not settle the federal case itself; rather, "[t]he six individual cases proceeded through a different court system without any procedural link to the federal case before us." - Second, the plaintiffs had not raised their federal claims in the state forfeiture cases themselves. - Third, "[t]he disparate dates at which plaintiffs' forfeiture proceedings terminated—11, 14, 27, and 40 months after the seizures—indicate that the [state] did not coordinate the resolution of plaintiffs' state court cases, either with each other or with plaintiffs' federal civil rights case."

Yee v. City of Escondido

the Court reiterated the proposition that if a federal claim was properly raised in state court, a party can raise before the Supreme Court any argument in support of that claim, even if the argument was not raised in state court. The distinction between a new claim and a new argument is hardly clear-cut.

Hui v. Castaneda (2010)

the Court rejected a Bivens claim alleging that Public Health Service (PHS) officials had shown "deliberate indifference" to Casteneda's "serious medical needs," in violation of the Fifth, Eighth, and Fourteenth Amendments, while he was in the custody of U.S. Immigration and Customs Enforcement (ICE). In a unanimous opinion for the Court, Justice Sotomayor held that the Bivens action was precluded by 42 U.S.C. § 233(a), This provision provides: "The Federal Tort Claims [FTCA] remedy against the United States for damage for personal injury, including death, resulting from the performance of medical or related functions by any [PHS] commissioned officer or employee while acting within the scope of his office or employment, shall be exclusive of any other civil action or proceeding by reason of the same subject-matter against the officer or employee. Court reads the statute at face value There was a different way to reason this through- 9th circuit said it is not meant to eclipse the availability of a Bivens claim

Fay v. Noia

the Court rejected the Daniels rule and greatly expanded habeas review of defaulted claims. Noia had been convicted of a capital crime but he avoided a death sentence. He then chose not to appeal. His subsequent motion for postconviction relief in state court, asserting that his conviction was based on a coerced confession, was denied because he had failed to appeal from his conviction. Meanwhile, Noia's two co-defendants, who had appealed from their convictions, did obtain postconviction relief in state court on the ground that their confessions were coerced. When Noia's federal habeas petition raising the coerced confession claim reached the Supreme Court, it held that his failure to have appealed from his conviction did not preclude the exercise of federal habeas jurisdiction. Procedural default bars review only if the defendant himself intentionally circumvented state remedy. Then the district court has limited discretion to deny relief.

Greene v. Fisher

the Court ruled that "clearly established law", for purposes of § 2254(d), does not include Supreme Court decisions announced after the last decision on the merits in state court but before the defendant's conviction became final on direct review. A petitioner's claim that relies on such decisions would not be "new" under (and therefore would not be barred by) Teague v. Lane, but in Greene, the Court stressed that the inquiries under § 2254(d) and Teague are distinct and relied upon what it viewed as the plain meaning of § 2254(d)

Atlas Roofing Co. v. Occupational Safety & Health Review Comm'n

the Court upheld an agency's assessment of a civil money penalty for violation of workplace safety regulations. Relying on the distinction between public and private rights, the Court said: "At least in cases in which public rights are being litigated—e.g., cases in which the Government sues in its sovereign capacity to enforce public rights created by statutes within the power of Congress to enact—the Seventh Amendment does not prohibit Congress from assigning the factfinding function and initial adjudication to an administrative forum with which the jury would be incompatible"

Carlson v. Green (1980)

the Court upheld the availability of a damages remedy in an action alleging that the failure of federal prison officials to provide medical attention to plaintiff's deceased son constituted cruel and unusual punishment in violation of the Eighth Amendment. The Court, again per Justice Brennan, stated flatly that "the victims of a constitutional violation by a federal agent have a right to recover damages against the official in federal court despite the absence of any statute conferring such a right," unless (1) the defendant demonstrates "special factors counselling hesitation", or (2) "Congress has provided an alternative remedy which it explicitly declared to be a substitute for recovery directly under the Constitution and viewed as equally effective"-> Eighth Amendment denial of adequate medical care Important idea: Recognizes that remedy even though there is an FTCA- the FTCA claim is just not as good - Congress did not intend that as a substitute for Bivens. -> Not good enough to refuse to imply a claim under the 8th amendment The Court suggested four ways in which the Bivens remedy might be more effective than the FTCA remedy: (a) an action against the individual wrongdoer is a more effective deterrent than an action against the government; (b) unlike the FTCA, the Bivens remedy permits punitive damages; (c) a Bivens plaintiff can opt for jury trial, unavailable under the FTCA; and (d) the FTCA applies only to conduct that would be actionable under state law if committed by a private person, whereas uniform federal rules govern the extent of Bivens liability.

Florida Prepaid Postsecondary Education Expense Board v. College Savings Bank (1999)

the Court, by 5-4, held unconstitutional provisions of the Patent Remedy Act. Chief Justice Rehnquist cited Boerne as establishing that for Congress to invoke § 5, "it must identify conduct transgressing the Fourteenth Amendment's substantive provisions, and must tailor its legislative scheme to remedying or preventing such conduct". He then concluded that this standard had not been met. Congress had "identified no pattern of patent infringement by the States, let alone a pattern of constitutional violations" Also note the case that every time there is a patent infringment- only when it is a due process violation. There is daylight between patent infringement and due process- a state could provide some other remedy or if they did it accidentally- so the court wants to see a record a record of widespread demonstration of the deprivation of rights

Coleman v. Ct. of Appeals of Maryland

the Court, without a majority opinion, held that Congress had no authority under the Fourteenth Amendment to abrogate the states' immunity from suit under a provision of the FMLA that required employers to provide up to twelve weeks of unpaid leave per year for an employee's own health issues. (Hibbs had upheld provisions of the FMLA regarding the health issues of an employee's family member.) Writing for a four-Justice plurality, Justice Kennedy found that Congress had not adequately linked the provision to a pattern of constitutional violations and, in particular, had not shown it to be a response to sex discrimination in sick leave policies. Self-care provision was not justified by the history. Writing for a four-Justice plurality, Justice Kennedy found that Congress had not adequately linked the provision to a pattern of constitutional violations and, in particular, had not shown it to be a response to sex discrimination in sick leave policies.

In re Ayers (1887)

the Eleventh Amendment barred an award of injunctive relief "the object of which is to compel the specific performance of a contract." The Young Court distinguished Ayers as follows: "the suit was one against the state, to enforce its alleged contract. It was not stated that the suit or the injunction was necessarily confined to a case of a threatened direct trespass upon or injury to property." What we can take away from this case is that it made clear that the caption test was no longer going to apply- the state was not a party to the suit and yet the court found that the 11A applied.

United States Bancorp Mortgage Co. v. Bonner Mall Partnership

the Supreme Court granted certiorari to resolve a question under the Bankruptcy Code, and the parties thereafter reached a settlement that mooted the case. Relying on Munsingwear, Bancorp, the losing party in the court of appeals, asked the Supreme Court to vacate the judgment below as well as dismissing the writ. "Mootness by reason of settlement does not [ordinarily] justify vacatur of a judgment under review." - Although mootness removes a court's authority to pronounce on the merits, the court retains authority to take such ancillary action as justice requires. - Judicial precedents are presumptively correct and valuable to the legal community as a whole. They are not merely the property of private litigants and should stand unless a court concludes that the public interest would be served by vacatur."

Prejudice:

the Supreme Court's subsequent decisions dealing with procedural default have not elaborated on just what constitutes a showing of prejudice. The most pertinent statement came in United States v. Frady (1982), a collateral postconviction attack brought by a federal prisoner where, the Court said that to establish prejudice, the prisoner must show that errors at trial "worked to his actual and substantial disadvantage, infecting his entire trial with error of constitutional dimensions." Show it is a violation of Brady means you are showing prejudice for Sykes- reasonable probability of a different result?

Felder v. Casey (1988)

the Wisconsin Supreme Court had dismissed a state court § 1983 suit because of plaintiff's noncompliance with the state's notice-of-claim statute, which required, as a condition of bringing suit in state court, provision of written notice, within 120 days of the injury, of any claim against state or local governments (or their officials). The Supreme Court reversed, reasoning that "[g]iven the evil at which the federal civil rights legislation was aimed, there is simply no reason to suppose that Congress contemplated that those who sought to vindicate their federal rights in state courts could be required to seek redress in the first instance from the very state officials whose hostility to those rights precipitated their injuries"

Nevada Dep't of Human Resources v. Hibbs

the abrogation of state sovereign immunity in the Family and Medical Leave Act of 1993 (FMLA). This Act, inter alia, entitles eligible employees (including certain employees of public agencies) to up to twelve weeks of unpaid leave annually for any of several reasons, including the onset of a "serious health condition" in the employee's spouse, child, or parent. The Act also creates a private right of action "against any employer (including a public agency)" for equitable and monetary relief. The Court, per Chief Justice Rehnquist, held that Congress had met its burden under § 5: it had satisfied the requirement that the statement of abrogation be clearly made and, in light of the relevant evidence of gender-based discrimination in the legislative record, had sufficient evidence of a "pattern of constitutional violations on the part of the States in this area" to warrant the provision of the FMLA's private rights and remedies as "appropriate prophylactic legislation". Congress did have sufficient evidence of discrimination + this was gender based discrimination which gets heightened scrutiny.

Avco Corp. v. Aero Lodge No. 735 (1968):

the company sued the union in state court, alleging that the union had violated the terms of a collective bargaining agreement, under which the union had agreed to submit all grievances to binding arbitration and not to cause or sanction any "work stoppages, strikes, or slowdowns." Although the company pled only a violation of state contract law, the union removed the case to federal court. On review, the Supreme Court held that the company's claim, although labeled a state law contract claim in the complaint, necessarily arose under § 301 of the federal Taft-Hartley Act, and thus, because it arose under federal law, was removable. "the pre-emptive force of § 301 is so powerful as to displace entirely any state cause of action [for breach of contract. if a federal cause of action completely preempts a state cause of action, any complaint that comes within the scope of the federal cause of action necessarily 'arises under' federal law."

Vachon v. New Hampshire (1974)

the defendant had been convicted of willfully contributing to the delinquency of a minor, for having sold a 14-year-old girl a button containing a sexual slogan. On appeal to the state supreme court, he had unsuccessfully challenged the sufficiency of the evidence of willfulness. The Supreme Court, in a brief per curiam opinion, reversed, relying on the federal constitutional principle that due process is denied when there is "no evidence" to support one element of a crime (here, willfulness). The Court relied on provisions in the Supreme Court's Rules stating that questions raised in the brief that were not presented in the jurisdictional papers "will be disregarded, save as the court, at its option, may notice a plain error not presented."

Hutto v. Finney (1978)

the district court—after finding that conditions in the Arkansas penal system constituted cruel and unusual punishment and issuing various injunctive orders over the course of seven years—had ruled that defendant officials had acted in bad faith and ordered them to pay $20,000 "out of Department of Correction funds" to plaintiffs' attorneys. In upholding that fee award, the Supreme Court stressed the importance of enforcing federal court orders, and held that "[t]he power to impose a fine is properly treated as ancillary to the federal court's power to impose injunctive relief. In this case, the award of attorney's fees for bad faith served the same purpose as a remedial fine imposed for civil contempt. It vindicated the District Court's authority over a recalcitrant litigant. Essentially what the court was saying here is that they need to be able to enforce the injunction and attorney's fees is a good way to make sure it can.

Prentis v. Atlantic Coast Line Co.

the plaintiffs asked a federal court to enjoin the enforcement of a rate order of the Virginia State Corporation Commission. On appeal from a lower court decision granting the injunction, the appellants argued that under state law the commission had the characteristics and powers of a court and that the Anti-Injunction Act, now 28 U.S.C. § 2283, forbade a federal injunction. The Court, speaking through Justice Holmes, dismissed that objection to the exercise of federal jurisdiction-> A plaintiff may not sue in federal court for redress of allegedly unlawful state action without first invoking or "exhausting" available state administrative (as opposed to judicial) remedies.

Trump v. NY: Census

the president took the view that in doing the census, the policy of apportionment base would exclude aliens who are not in a lawful immigration status. NY and others sued to change this policy. Three judge DC held that - 1) plaintiffs have standing 2) plaintiffs entitled to an injunction against inclusion of the immigration-status information in the census report. Standing because: the policy was chilling undocumented and their families from responding to the census thereby degrading the quality of the census data used to allocate federal funds and forcing some plaintiffs to divert resources to combat the chilling effect. SCOTUS finds no standing here- Standing must not depend on "contingent future events that may not occur as anticipated, or indeed may not occur at all." Chilling effect on the census- this seems to be moot Funding: Although the President clearly expressed his desire to exclude unlawfully present noncitizens from the apportionment base "to the extent practicable," it remains mere conjecture whether and how the Executive Branch might eventually implement this general statement of policy.

Mesa v. California (1989)

two Postal Service employees faced state criminal prosecutions arising out of traffic violations committed in connection with their jobs. The prosecutions were removed to federal court under 28 U.S.C. § 1442(a)(1), which authorizes removal of any civil or criminal action against an officer of the United States for any act "under color of such office". The government, relying in part on its interpretation of Tennessee v. Davis, argued that removal was proper even though the defendants asserted no colorable federal defense to the state charges. The Supreme Court rejected that view, ruling instead that § 1442(a) permits federal officer removal only when the defendant-officer avers a federal defense. Statute seems to fit- individual being sued for an act under their office- but the SCOTUS interprets it to kick in only when they say I have a federal law defense. Why? No protective jurisdiction: idea might be that under the DOJ's interpretation - they might say they don't want the federal official subjected to state courts. Some sort of protective jurisdiction. However, SCOTUS says it does not work- statute requires an actual federal question/defense to be implicated- don't want to go there and make the statute so broad. We have, in the past, not found the need to adopt a theory of 'protective jurisdiction' to support Art. III 'arising under' jurisdiction, and we do not see any need for doing so here because we do not recognize any federal interests that are not protected by limiting removal to situations in which a federal defense is alleged. In these prosecutions, no state court hostility or interference has even been alleged by petitioners If so then we have to confront the harder question of whether it is constitutional to grant jurisdiction on this basis of protective jurisdiction.

Merrell Dow Pharmaceuticals Inc. v. Thompson

two state law tort actions, the plaintiffs alleged that the defendants had misbranded a drug in violation of the Federal Food, Drug, and Cosmetic Act (FDCA). Under Ohio law, such a violation created a rebuttable presumption of negligence. Thus, embedded within a state law claim was a federal issue about the meaning and application of the FDCA. The defendant removed the actions from state to federal court. Court finds removal improper in large part because there is no cause of action under the FDCA. In exploring the outer reaches of § 1331, determinations about federal jurisdiction require sensitive judgments about congressional intent, judicial power, and the federal system."-not really giving a good rule- what do you pull out? No express right of action- pre- Sandoval so it would use the Cort factors. It is even worse for the plaintiff that they are using Cort- one of the factors is did Congress intend to not create an action. Not finding a cause of action means they did not intend a right of action. Court also noted the importance of the federal interest at play in determining if there is jurisdiction.

Yazoo County Indus. Dev. Corp. v. Suthoff (1982)(Rehnquist)

urged reexamination of the approach set forth in Bell v. Hood, which, he said, requires a "three tiered analysis" of motions to dismiss: a complaint may (i) be so wanting that it is dismissed for want of jurisdiction, (ii) establish jurisdiction but be dismissed for failure to state a claim, or (iii) state a proper claim not subject to dismissal. He suggested that Bell was wrong to treat a frivolous claim as falling outside the district courts' jurisdiction, and favored recognizing only two tiers: complaints whose allegations state a good claim (category (iii)) and those that do not (categories (i) and (ii)). Sum: under Bell v. Hood, if the federal claim is so wanting as to be "wholly frivolous and insubstantial", then no jurisdiction exists, precluding the exercise of supplemental jurisdiction over any state claim averred in the complaint; Justice Rehnquist's approach, by contrast, would not categorically preclude supplemental jurisdiction.

Martin v. Hunter Lessee (1816)

uring the American Revolution, Virginia created laws allowing the state to seize property of Loyalists. In 1781, Denny Martin, a British subject, inherited land from his uncle, a Loyalist. The following year, the Virginia legislature voided the land grant and transferred the land back to Virginia. Virginia granted a portion of this land to David Hunter. A federal treaty dictated that Lord Fairfax was entitled to the property. The Virginia Supreme Court upheld Virginia's law permitting the confiscation of property, even though it conflicted with the federal treaty. The U.S. Supreme Court reversed and remanded, holding that the treaty superseded state law under the Supremacy Clause of Article VI. On remand, the Virginia Court of Appeals declined to follow the ruling and argued that the law granting the Supreme Court appellate review over state court decisions, section 25 of the Judiciary Act (the Act), was unconstitutional. SCOTUS appellate jurisdiction encompasses cases coming from state courts. SCOTUS - appellate jurisdiction everything not within original jurisdiction- Art III S 2 cl 2 differentiates by subject matter not by court in which it is appealed. Otherwise it would throw off the role of state courts. If state courts could hear federal issues but no SCOTUS review then you would have federal law without a federal court which he already said you cannot have.

Novelty

very hard standard to meet: At the time of appeal, counsel could not reasonably have been expected to know his client had a constitutional argument. -> not just new in the Teague sense Reed v. Ross (1984): the jury instructions at Ross' trial in North Carolina placed on him the burden of proving provocation (which would mitigate murder to manslaughter). Ross did not challenge those instructions in his appeal, which was decided one year before the Supreme Court held that due process requires "proof beyond a reasonable doubt of every fact necessary to constitute the crime with which [a defendant] is charged." After the ruling in Hankerson v. North Carolina that North Carolina's assignment of the burden of proof of provocation was unconstitutional under Winship, Ross sought federal habeas relief. The Court, with Justice Brennan writing, found that "the cause requirement may be satisfied under certain circumstances when a procedural failure is not attributable to an intentional decision by counsel made in pursuit of his client's interests." Possible conflict with Teague - if you have a novel argument as to avoid Sykes, then you have a new rule. In order to actually get review, you need to then fall into one of Teague's narrow exceptions.-> makes novelty not a large help.

Explicit statutory directive

veryone agrees that this would rebut, but it is not present in this case. What does it need to say to be explicit? Will usually just say "federal courts shall have exclusive jurisdiction." Once the court adopts a rule like it did in Gulf Offshore then it is clear for Congress to uses this language; if it doesn't then there is a stronger presumption it did not intend exclusive federal jurisdiction. Permissive—rather than mandatory—language does not defeat the presumption. Example: 28 U.S.C. § 1338(a): "No state court shall have jurisdiction over any claim for relief arising under Act of Congress relating to patents . . . or copyrights"

Verlinden B.V. v. Central Bank of Nigeria (1983

was a rare case requiring the Supreme Court to consider a federal statutory grant of jurisdiction that seemed to approach the outer limits of Article III. The Verlinden case involved a federal court action by a Dutch corporation against an instrumentality of the Government of Nigeria for breach of a contract. Statutory jurisdiction over the action was provided by the Foreign Sovereign Immunities Act (FSIA), which confers on federal district courts original jurisdiction over "any nonjury civil action against a foreign state as to any claim with respect to which the foreign state is not entitled to immunity" under either the FSIA or any applicable international agreement. The main thrust of the FSIA is to establish substantive standards and procedural rules governing suits brought against foreign nations in the federal and state courts. The Act details when foreign states are immune from suit and specifies a number of instances in which such immunity does not exist; in the Verlinden case, the plaintiff alleged that Nigeria's conduct caused effects within the United States, within the meaning of one of the FSIA's provisions recognizing an exception to foreign sovereign immunity. the defendant moved to dismiss the complaint, for want of subject matter jurisdiction. The court of appeals held that the Act was unconstitutional insofar as it purported to authorize the federal courts to entertain an action when the substantive claim is not based on federal law and when diversity jurisdiction was absent. FSIA's grant of federal jurisdiction is constitutional- under Osborn- an action against a foreign sovereign arises under federal law, for purposes of Art. III jurisdiction.

Chisholm v. Georgia (1793)

when a South Carolina citizen filed an assumpsit claim against the State of Georgia as an original action in the Supreme Court. Jurisdiction was predicated on a statute authorizing the Court to entertain actions against a state by citizens of another state pursuant to the customs and usages of law. Rejecting Georgia's protest that an unconsenting state was immune from suit, the Court upheld its jurisdiction as consistent with Article III's grant of judicial power over controversies "between a State and Citizens of another State." Four of the Justices found that there was no issue of an unconsenting state being sued in federal court - Justices Blair and Cushing both relied on the clear language of Article III, noting that its grant of jurisdiction over controversies between two states contemplated that an unconsenting state could be a defendant. - Chief Justice Jay likewise relied on Article III's language, but argued in addition that the "feudal" doctrine of sovereign immunity was incompatible with popular sovereignty. Troubled, though, that this argument implied that the United States itself could be sued, notwithstanding the difficulty of enforcing a judgment against it, he ultimately left that question open. - Justice Wilson argued most fully that the doctrine of sovereign immunity was incompatible with principles of public law and with a republican form of government, reasoning that a state was no more sovereign, and no less subject to the law, than a free man.

"Antecedent" Versus "Distinct" State Law Grounds

where a state law ruling serves as an antecedent for determining whether a federal right has been violated, some review of the basis for the state court's determination of the state-law question is essential if the federal right is to be protected against evasion and discrimination—as Martin itself exemplifies. In Martin v. Hunter's Lessee, to obtain the relief he sought, Martin had to prevail on both the non-federal issue (that his chain of title had not been divested at the time of the Treaty of Peace) and the federal law issue (that the Treaty protected that title against the Act of Compromise and other efforts to divest it). In Murdock, the non-federal issue of trust law was logically (and functionally) quite distinct from any issue of federal law; answering the state law question was not a necessary antecedent to any question of federal law.

Douglas v. New York

where the New York courts had dismissed an FELA action by a Connecticut resident against a Connecticut corporation based on an accident in Connecticut. A New York statute permitted actions by a nonresident against a foreign corporation only in certain cases, of which this was not one. On appeal, Justice Holmes said that the FELA "does not purport to require State Courts to entertain suits arising under it, but only to empower them to do so, so far as the authority of the United States is concerned. It may very well be that if the Supreme Court of New York were given no discretion, being otherwise competent, it would be subject to a duty. But there is nothing in the Act of Congress that purports to force a duty upon such Courts as against an otherwise valid excuse.

Federalist 82

writing that the states "will retain all pre-existing authorities" that are not exclusively delegated to the national government, either by (i) an express grant of exclusivity to the national government, (ii) a grant of authority to the union where "the exercise of a like authority is prohibited to the States," or (iii) an authority granted to the Union, with which the exercise of state authority would be utterly incompatible. He then said that Article III's language—in particular, that "[t]he JUDICIAL POWER of the United States shall be vested in one Supreme Court, and in such inferior courts as the Congress shall from time to time ordain and establish"—should not suggest that the federal judicial power is exclusive, as that would "amount to an alienation of State power by implication." Instead, he preferred interpreting the quoted language as merely identifying the courts that will exercise the judicial power. he then said that state courts would have concurrent jurisdiction not only of pre-existing causes but also of causes of action to which federal acts of Congress "may give birth."


Related study sets

Texas Promulgated Contract Forms Chapter 7

View Set

Psychology 100 LC 7/8/9/10/11/12/13/14/15

View Set

Ch. 27 - The Child with Genitourinary Dysfunction

View Set

Indian Independence and Partition

View Set